You are on page 1of 207

1. Calimutan v.

People 482 SCRA 47

Facts: Victim Cantre and Saano, together with two other companions had a drinking spree in a
videoke bar at ten oclock in the morning of February 4, 1996. Thereafter, they decided to part
ways and went to their respective houses. On their way home, Cantre and Sanano met the
petitioner and Michael Bulalacao. Cantre suddenly punched Bulalacao because he is suspecting
the latter as the one responsible for throwing stones at his house on previous night. After being
hit, Bulalacao ran away. Petitioner picked-up a stone which is as big as mans fist, ran toward
Cantre, and threw it to the latter, hitting him at the left side of his back. When Cantre turned his
attention to the petitioner, Sanano tried pacify the two. Both Cantre and petitioner calmed down
and went to their houses. When Cantre arrived at his house, he complained of the pain in the left
side of his back which was hit by the stone. At that night, he again complained of backache and
also of stomachache. Hes condition immediately became worst, and at around three oclock in
the following morning, Cantre died.

Right after his death, Cantre was examined by Dr. Conchita S. Ulanday, the Municipal Health
Officer and made a finding that the cause of death was cardio-respiratory arrest due to suspected
food poisoning. Unsatisfied, the Cantre family requested for an exhumation and autopsy of the
body of the victim by the NBI. Dr. Mendez conducted an exhumation and autopsy and reported
that the cause of the death was traumatic injury of the abdomen. The victim suffered from an
internal hemorrhage and there was massive accumulation of blood in his abdominal cavity due to
his lacerated spleen caused by any blunt instrument, such as a stone.

Petitioner alleged that he only attempted to pacify the victim but the latter refused and pulled out
eight-inch Balisong. When he saw the victim was about to stab Bulalacao, he picked up a stone
and threw it at the victim Cantre. He was able to hit the victim. He contended that the throwing
of the stone was in defense of his companion.

The RTC rendered a decision, which was later affirmed by the CA, holding that petitioner was
criminally liable for homicide and that the act of throwing a stone from behind was a treacherous
one and the accused committed a felony which caused the death of the victim and held that the
accused is criminally liable for all the direct and natural consequences of this unlawful act even
if the ultimate result had not been intended. Hence, these case.

Issue: Whether or not the petitioner has the intent to kill the victim and thus liable for homicide?

Decision: While the Supreme Court is in accord with the factual findings of the RTC and the CA
and affirms that there is ample evidence proving that the death of the victim Cantre was caused
by his lacerated spleen which is the result by the stone thrown at him by petitioner Calimutan, it
nonetheless, is at variance with the RTC and the CA as to the determination of the appropriate
crime or offense for which the petitioner should have been convicted for.

Article 3 of the Revised Penal Code classifies felonies according to the means by which they are
committed, in particular: (1) intentional felonies, and (2) culpable felonies. These two types of
felonies are distinguished from each other by the existence or absence of malicious intent of the
offender.

In intentional felonies, the act or omission of the offender is malicious. In the language of Art. 3,
the act is performed with deliberate intent (with malice). The offender, in performing the act or in
incurring the omission, has the intention to cause an injury to another. In culpable felonies, the
act or omission of the offender is not malicious. The injury caused by the offender to another
person is "unintentional, it being simply the incident of another act performed without malice."
(People vs. Sara, 55 Phil. 939). As stated in Art. 3, the wrongful act results from imprudence,
negligence, lack of foresight or lack of skill.

In the Petition at bar, this Court cannot, in good conscience, attribute to petitioner any malicious
intent to injure, much less to kill, the victim Cantre; and in the absence of such intent, this Court
cannot sustain the conviction of petitioner Calimutan for the intentional crime of homicide, as
rendered by the RTC and affirmed by the Court of Appeals. Instead, this Court finds petitioner
Calimutan guilty beyond reasonable doubt of the culpable felony of reckless imprudence
resulting in homicide under Article 365 of the Revised Penal Code. The prosecution did not
establish that petitioner Calimutan threw the stone at the victim Cantre with the specific intent of
killing, or at the very least, of harming the victim Cantre. What is obvious to this Court was
petitioner Calimutans intention to drive away the attacker who was, at that point, the victim
Cantre, and to protect his helper Bulalacao who was, as earlier described, much younger and
smaller in built than the victim Cantre.

2. Manuel v. People 476 SCRA 461

Facts: This is a case filed against Eduardo Manuel for bigamy by Tina B. Gandalera.
Complainant allege that she met the petitioner in Dagupan City sometime in January 1996. When
he visited her in Baguio, as one thing led to another, they went to a motel where, Eduardo
succeeded in having his way with her. Petitioner proposed marriage and even brought his parents
to assure that he is single. Tina finally accepted the marriage proposal and they were married on
April 22, 1996. In their marriage contract, it appeared that Eduardo is single. However, their
happy relationship turns into a disaster, Manuel started making himself scarce and went to their
house only twice or thrice a year. One day, petitioner took all of his cloths, left and never
returned. Out of curiousity, Tina went to NSO in Manila where she found out that petitioner had
been previously married to Rubylus Gaa. She was so embarrassed and humiliated when she
learned that Eduardo was in fact already married when they exchanged their own vows.

For his part, Eduardo testified that he informed Tina of his previous marriage, but she
nevertheless agreed to marry him. He abandoned her when he noticed that she had a "love-bite"
on her neck, suspecting it that it come from another man. Eduardo further testified that he
declared he was "single" in his marriage contract with Tina because he believed in good faith that
his first marriage was invalid. He did not know that he had to go to court to seek for the
nullification of his first marriage before marrying Tina.. Rubylus was charged with estafa in 1975
and thereafter imprisoned. He visited her in jail after three months and never saw her again. He
insisted that he married Tina believing that his first marriage was no longer valid because he had
not heard from Rubylus for more than 20 years. After trial, the court rendered judgment finding
Eduardo guilty beyond reasonable doubt of bigamy. It declared that Eduardos belief, that his
first marriage had been dissolved because of his first wifes 20-year absence, even if true, did not
exculpate him from liability for bigamy and that even if the private complainant had known that
Eduardo had been previously married, the latter would still be criminally liable for bigamy.
Eduardo appealed the decision to the CA maintaining his contentions. He insisted that
conformably to Article 3 of the Revised Penal Code, there must be malice for one to be
criminally liable for a felony. He was not motivated by malice in marrying the private
complainant because he did so only out of his overwhelming desire to have a fruitful marriage.
Hence, these case.

Issue: Whether or not the petitioner has criminal intent to contract on the second marriage to be
liable for bigamy?

Decision: The Supreme Court ruled that the prosecution proved that the petitioner was married to
Gaa in 1975, and such marriage was not judicially declared a nullity; hence, the marriage is
presumed to subsist. The prosecution also proved that the petitioner married the private
complainant in 1996, long after the effectivity of the Family Code. The petitioner is presumed to
have acted with malice or evil intent when he married the private complainant. As a general rule,
mistake of fact or good faith of the accused is a valid defense in a prosecution for a felony
by dolo; such defense negates malice or criminal intent. However, ignorance of the law is not an
excuse because everyone is presumed to know the law. Ignorantia legis neminem excusat. It was
the burden of the petitioner to prove his defense that when he married the private complainant in
1996, he was of the well-grounded belief that his first wife was already dead, as he had not
heard from her for more than 20 years since 1975. He should have adduced in evidence a
decision of a competent court declaring the presumptive death of his first wife as required by
Article 349 of the Revised Penal Code, in relation to Article 41 of the Family Code. Such judicial
declaration also constitutes proof that the petitioner acted in good faith, and would negate
criminal intent on his part when he married the private complainant and, as a consequence, he
could not be held guilty of bigamy in such case. The petitioner, however, failed to discharge his
burden.

Article 3, paragraph 2 of the Revised Penal Code provides that there is deceit when the act is
performed with deliberate intent. Indeed, a felony cannot exist without intent. Since a felony
by dolo is classified as an intentional felony, it is deemed voluntary. Although the words "with
malice" do not appear in Article 3 of the Revised Penal Code, such phrase is included in the word
"voluntary."

Malice is a mental state or condition prompting the doing of an overt act without legal excuse or
justification from which another suffers injury. When the act or omission defined by law as a
felony is proved to have been done or committed by the accused, the law presumes it to have
been intentional. Indeed, it is a legal presumption of law that every man intends the natural or
probable consequence of his voluntary act in the absence of proof to the contrary, and such
presumption must prevail unless a reasonable doubt exists from a consideration of the whole
evidence.

3. U.S. v. Ah Chong (15 Phil. 488)

Facts: Defendant herein a chinese man named Ah Chong is employed us a cook at Fort
Mckinley. At that time there were rumours and accounts of frequent robbing of homes in the
area.

On the night of the killing, Ah chong before going to bed, and afraid of the rumoured robberies
taking place in the vicinity locked himself in their room by placing wooden blocks and chairs for
the purpose of thwarting robbers in case they tried to rob him.

After having gone to bed, he was awakened by the noise of someone trying to open the door. Ah
Chong for his part called out twice, Who is there, but to no avail. Fearing that the person trying
to enter was robber Ah Chong leaped from his bed and shouted If you enter the room I will kill
you. But at that precise moment, he was suddenly struck by the chair that he had placed in the
door, and believing that he was being attacked he seized a knife and struck it on the supposed
assailant/robber, who was killed by the blow. However the deceased was not a robber not
intruder it turned out that the person was his roommate, trying to enter their room.

Issue:Whether or not Ah Chong is criminally liable?

Held: NO. Ah Chong must be acquitted on the basis of honest mistake of fact. Where the facts
been as Ah Chong perceived them to be, he would have been justified in killing the intruder
under Article 11 of the Revised, par. 1 of the Revised Penal Code, which provides for a valid
self-defense of his person. If the intruder was indeed a robber, forcing his way to enter the room,
unlawful aggression would be present. Also the necessity means to avoid or to repel the attack
would be reasonable. Using the knife to defend himself. And lastly Ah Chong gave no
provocation at all to warrant such aggression. The Supreme Court Held that there is nothing
unlawful in the intention as well in the act of Ah Chong, his act would not have been a felony if
the real scenario was the facts he believed them to be.

4. People vs. De Fernando49 Phil. 75

FACTS: The accused, a policeman, was informed that three convicts had escaped. In the dark, he
saw a person going up the stairs of a house, carrying a bolo andcalling for someone inside. The
daughter of the owner of the house was at that time with the accused who fired a shot in the air.

As the unknown person continued to ascend the stairs and believing that he was one of the
escaped convicts, the accused fired directly at the man who turned out to be the nephewof the
owner of the house.

ISSUE: Whether or not the appellant is exempt from criminal liability due to mistake of fact.

HELD: An agent of the law, to whom notice had been given of the presence of suspicious
looking persons, who might be escaped prisoners from a nearby penitentiary, prowling around
the vicinity, and who enters a house to keep watch, and later in the evening sees a person with a
bolo in hand, approaching the house in the attitude of going up the stairs, who does not answer
the challenge of the officer of the law, and continues his advance notwithstanding that the latter
had fired a shot into the air, and the said agent of the law considering that the said stranger has
not been recognized by any person in the household, and thinking him to be an evil-doer, shoots
and kills him, is not guilty of murder or homicide. Taking into consideration the state of the mind
of the accused at the time, andthe meaning that he gave to the attitude of the unknown person, in
shooting the latter, he felt that he was performing his duty by defending the owners of the house
against an unexpected attack, and such act cannot constitute the crime of murder, but only that of
a simple homicide. He cannot be held guilty, however as principal, with malicious intent,
because he thought at the time that he was justified in acting as he did, and he is guilty only
because he failed to exercise the ordinary diligence which, under the circumstances, he should
have by investigating whether or not the unknown man was really what he thought him to be. In
firing the shot, without first exercising reasonable diligence, he acted with reckless negligence.
The crime committed by the accused, therefore is homicide though reckless negligence defined
and punished in Article 568, in relation with Art. 404, of the Penal Code

5. Diego V Judge Castillo 436 SCRA 67


Facts: Complainant herein alleges that the decision rendered by the respondent Judge is
manifestly against the law and contrary to the evidence. He questions the evidentiary weight and
admissibility of the divorce decree as a basis for the finding of good faith. In addition,
complainant stresses that the evidence on record negates respondent Judges finding of good
faith on the part of the accused. Thus, complainant urges this Court to impose sanctions upon
respondent Judge as, according to complainant, these acts amount to knowingly rendering an
unjust judgment and/or gross ignorance of the law.

In his comment, respondent Judge explains that what was in issue was the criminal culpability of
the accused under Article 349 of the Revised Penal Code. Respondent Judge does not dispute
that the second marriage was bigamous because at the time it was contracted, the first marriage
was still subsisting since divorce is not recognized in our country and because the accuseds first
husband was still alive. Respondent Judge, however, maintains that what was controlling was
whether by virtue of the divorce decree the accused honestly believed, albeit mistakenly, that her
first marriage had been severed and she could marry again. According to respondent Judge, the
same is a state of mind personal to the accused. He further stressed that knowledge of the law
should not be exacted strictly from the accused since she is a lay person, and that ineptitude
should not be confused with criminal intent.
Issue: Whether or not Judge Castillo is guilty of Mistake of fact.
Held: A careful study of the disputed decision reveals that respondent Judge had been less than
circumspect in his study of the law and jurisprudence applicable to the bigamy case.
In his comment, respondent Judge stated: That the accused married Manuel P. Diego in the
honest belief that she was free to do so by virtue of the decree of divorce is a mistake of fact.
This Court, in People v. Bitdu, carefully distinguished between a mistake of fact, which could be
a basis for the defense of good faith in a bigamy case, from a mistake of law, which does not
excuse a person, even a lay person, from liability. Bitdu held that even if the accused, who had
obtained a divorce under the Mohammedan custom, honestly believed that in contracting her
second marriage she was not committing any violation of the law, and that she had no criminal
intent, the same does not justify her act. This Court further stated therein that with respect to the
contention that the accused acted in good faith in contracting the second marriage, believing that
she had been validly divorced from her first husband, it is sufficient to say that everyone is
presumed to know the law, and the fact that one does not know that his act constitutes a violation
of the law does not exempt him from the consequences thereof.
Moreover, squarely applicable to the criminal case for bigamy, is People v.
Schneckenburger, where it was held that the accused who secured a foreign divorce, and later
remarried in the Philippines, in the belief that the foreign divorce was valid, is liable for bigamy.
These findings notwithstanding, the issue before us is whether or not respondent Judge should be
held administratively liable for knowingly rendering an unjust judgment and/or gross ignorance
of the law.
Knowingly rendering an unjust judgment is a criminal offense defined and penalized under
Article 204 of the Revised Penal Code. For conviction to lie, it must be proved that the judgment
is unjust and that the judge knows that it is unjust. Knowingly means consciously, intelligently,
willfully or intentionally. It is firmly established in this jurisdiction that for a judge to be held
liable for knowingly rendering an unjust judgment, it must be shown that the judgment is unjust

as it is contrary to law or is not supported by the evidence, and that the same was made with
conscious and deliberate intent to do an injustice.
The law requires that (a) the offender is a judge; (b) he renders a judgment in a case submitted to
him for decision; (c) the judgment is unjust; (d) he knew that said judgment is unjust. This Court
reiterates that in order to hold a judge liable, it must be shown that the judgment is unjust and
that it was made with conscious and deliberate intent to do an injustice. That good faith is a
defense to the charge of knowingly rendering an unjust judgment remains the law.
As held in Alforte v. Santos, even assuming that a judge erred in acquitting an accused, she still
cannot be administratively charged lacking the element of bad faith, malice or corrupt
purpose. Malice or bad faith on the part of the judge in rendering an unjust decision must still be
proved and failure on the part of the complainant to prove the same warrants the dismissal of the
administrative complaint.
There is, therefore, no basis for the charge of knowingly rendering an unjust judgment.

6. Estrada v. Sandiganbayan 369 SCRA 394

Facts: Petitioner Former President Joseph Estrada was prosecuted for a crime of violation of RA
7080 (An Act Defining and Penalizing the Crime of Plunder), as amended by RA 7659. Thus, he
questions the constitutionality of the said Law. One of the issues that was raised in the petition is
whether Plunder as defined in RA 7080 is a malum prohibitum, and if so, whether it is within the
power of Congress to so classify it.

Issue: Whether or not Plunder is a crime malum prohibitum?

Decision: The legislative declaration in R.A. No.7659 that plunder is a heinous offense implies
that it is a malum in se. For when the acts punished are inherently immoral or inherently wrong,
they are mala in se and it does not matter that such acts are punished in a special law, especially
since in the case of plunder the predicate crimes are mainly mala in se. Indeed, it would be
absurd to treat prosecutions for plunder as though they are mere prosecutions for violations of
the Bouncing Check Law (B.P. BIg. 22) or of an ordinance against jaywalking, without regard to
the inherent wrongness of the acts.

7. People v. Go Shiu Ling 251 SCRA 379

Facts: The Regional Trial Court of Pasay City finds accused-appellant Antonio Comia guilty of
conspiring with four others to import regulated drugs in violation of Art. III, Section 14 in
relation to Article IV, Section 21 of the Dangerous Drugs Act (Rep. Act No. 6425, as amended).

Issue: Whether or not a crime for violation of Dangerous Drugs Act is a crime malum
prohibitum?

Held: Even granting that Comia acted in good faith, he cannot escape criminal responsibility.
The crime with which he is charged is a malum prohibitum. Lack of criminal intent and good
faith are not exempting circumstances. As held inPeople v. Lo Ho Wing:

Moreover, the act of transporting a prohibited drug is a "malum prohibitum" because it is


punished as an offense under a special law. It is a wrong because it is prohibited by law. Without
the law punishing the act, it cannot be considered a wrong. As such, the mere commission of said
act is what constitutes the offense punished and suffices to validly charge and convict an
individual caught committing the act so punished, regardless of criminal intent.

Likewise, in People v. Bayona, it was held:


The rule is that in acts mala in se there must be a criminal intent, but in those mala prohibita it is
sufficient if the prohibited act was intentionally done. "Care must be exercised in distinguishing
the difference between the intent to commit the crime and the intent to perpetrate to act."

8. People V. Bayona 61 SCRA 181

Facts: Jose E. Desiderio, a representative of the Department of the Interior, and Major Agdamag
of the Philippine Constabulary, who had been designated to supervise the elections in the
Province of Capiz, testified positively that the defendant was within the fence surrounding the
polling place when Desiderio took possession of the revolver the defendant was carrying. This
also disposes of that part of the argument under the second assignment of error based on the

theory that the defendant was in a public road, where he had a right to be, when he was arrested.
The latter part of the argument under the second assignment of error is that if it be conceded that
the defendant went inside of the fence, he is nevertheless not guilty of a violation of the Election
Law, because he was called by a friend and merely approached him to find out what he wanted
and had no interest in the election; that there were many people in the public road in front of the
polling place, and the defendant could not leave his revolver in his automobile, which he himself
was driving, without running the risk of losing it and thereby incurring in a violation of the law.
Held: We cannot accept the reasons advanced by the Solicitor-General for the acquittal of the
defendant. The law which the defendant violated is a statutory provision, and the intent with
which he violated it is immaterial. It may be conceded that the defendant did not intend to
intimidate any elector or to violate the law in any other way, but when he got out of his
automobile and carried his revolver inside of the fence surrounding the polling place, he
committed the act complained of, and he committed it willfully. The act prohibited by the
Election Law was complete. The intention to intimidate the voters or to interfere otherwise with
the election is not made an essential element of the offense. Unless such an offender actually
makes use of his revolver, it would be extremely difficult, if not impossible, to prove that he
intended to intimidate the voters.
The rule is that in acts mala in se there must be a criminal intent, but in those mala prohibita it is
sufficient if the prohibited act was intentionally done. "Care must be exercised in distinguishing
the difference between the intent to commit the crime and the intent to perpetrate the act. ..."
(U.S. vs. Go Chico, 14 Phil., 128.)

9. U.S. V Mallari 29 PHIL 19

Facts: On the morning of September 25, 1913, in the barrio of Batasan, municipality of
Macabebe, Province of Pampanga, before going to his work, the defendant Maximo Mallari went
to the house of the married couple, Vicente Sunga and Canuta Flores, and from the shed outside
asked Vicente Sunga to cure his wife of a sickness from which she had been suffering for several
days, and which he thought was due to enchantment on the part of the said Vicente. As the latter
refused, averring that he was not a wizard and that he had not caused the illness of defendants
wife, the former became enraged and insulted the said spouses. Threatening to kill them, he
ascended the stairway carrying in his hand a thin, sharp bolo. At his wifes suggestion Vicente
Sunga tried to get out to report the matter to the teniente of the barrio, who lived at some paces

from their house, but as he met the defendant on the stairway, he immediately went back inside
and jumped out of a window. He was straightway pursued by the defendant and on arriving
almost in front of the house of the teniente saw that the defendant was following closely behind
him. He therefore turned to face his pursuer and defend himself as well as he could with his
hands. Thereupon the defendant with a single slash of the bolo wounded Vicente Sunga in the
abdomen, so that his intestines protruded therefrom. In this condition the victim sat down,
endeavoring with his hands to keep his intestines from falling out, while his assailant took to
flight.

The justice of the peace of Macabebe arrived on the scene a few moments later and in his
presence the wounded man declared that his assailant was Mallari, who had been in his house,
and who had inflicted the serious wound he had in the abdomen. As a consequence of this wound
he died three days later.

Issue: Whether Mallari should still be liable for the death of the victim even though it was also
proven that had there been sufficient medical intervention the victim would have survived.

Held: With reference to the classification of the criminal act, it does not appear in the case that
this was erroneous, for in spite of the statement of the health officer that the deceased might have
been saved if the wound had been aseptically treated from the first, its seriousness and fatal
character being due to lack of antiseptics, still the person inflicting it is responsible for all the
consequences of his criminal action, and therefore for the death that occurred some days after the
deceased received the wound.

10. Bataclan V. Medina 102 SCRA 181

Facts: Pass-midnight in September 1952, Juan Bataclan rode a bus owned by Mariano Medina
from Cavite to Pasay. While on its way, the driver of the bus was driving fast and when he
applied the brakes it cause the bus to be overturned. The driver, the conductor, and some
passengers were able to free themselves from the bus except Bataclan and 3 others. The
passengers called the help of the villagers and as it was dark, the villagers brought torch with
them. The driver and the conductor failed to warn the would-be helpers of the fact that gasoline
has spilled from the overturned bus so a huge fire ensued which engulfed the bus thereby killing
the 4 passengers trapped inside. It was also found later in trial that the tires of the bus were old.

ISSUE: Whether or not the proximate cause of the death of Bataclan et al was their burning by
reason of the torches which ignited the gasoline.

HELD: No. The proximate cause was the overturning of the bus which was caused by the
negligence of the driver because he was speeding and also he was already advised by Medina to
change the tires yet he did not. Such negligence resulted to the overturning of the bus. The
torches carried by the would-be helpers are not to be blamed. It is just but natural for the
villagers to respond to the call for help from the passengers and since it is a rural area which did
not have flashlights, torches are the natural source of lighting. Further, the smell of gas could
have been all over the place yet the driver and the conductor failed to provide warning about said
fact to the villagers.

Proximate cause is that cause, which, in natural and continuous sequence, unbroken by any
efficient intervening cause, produces the injury, and without which the result would not have
occurred.

And more comprehensively, the proximate legal cause is that acting first and producing the
injury, either immediately or by setting other events in motion, all constituting a natural and
continuous chain of events, each having a close causal connection with its immediate
predecessor, the final event in the chain immediately effecting the injury as a natural and
probable result of the cause which first acted, under such circumstances that the person
responsible for the first event should, as an ordinary prudent and intelligent person, have
reasonable ground to expect at the moment of his act or default that an injury to some person
might probably result therefrom.

11. Quinto V. Andres 453 SCRA 511

Facts: At around 7:30 a.m. on November 13, 1995, eleven-year-old Edison Garcia, a Grade 4
elementary school pupil, and his playmate, Wilson Quinto, who was also about eleven years old,
were at Barangay San Rafael, Tarlac, Tarlac. They saw respondents Dante Andres and Randyver
Pacheco by the mouth of a drainage culvert. Andres and Pacheco invited Wilson to go fishing
with them inside the drainage culvert.[1] Wilson assented. When Garcia saw that it was dark

inside, he opted to remain seated in a grassy area about two meters from the entrance of the
drainage system.
Respondent Pacheco had a flashlight. He, along with respondent Andres and Wilson, entered the
drainage system which was covered by concrete culvert about a meter high and a meter wide,
with water about a foot deep. After a while, respondent Pacheco, who was holding a fish, came
out of the drainage system and left without saying a word. Respondent Andres also came out,
went back inside, and emerged again, this time, carrying Wilson who was already dead.
Respondent Andres laid the boys lifeless body down in the grassy area.[5]Shocked at the sudden
turn of events, Garcia fled from the scene. For his part, respondent Andres went to the house of
petitioner Melba Quinto, Wilsons mother, and informed her that her son had died. Melba
Quinto rushed to the drainage culvert while respondent Andres followed her.
Two weeks thereafter, or on November 28, 1995, National Bureau of Investigation (NBI)
investigators took the sworn statements of respondent Pacheco, Garcia and petitioner
Quinto. Respondent Pacheco alleged that he had never been to the drainage system catching fish
with respondent Andres and Wilson. He also declared that he saw Wilson already dead when he
passed by the drainage system while riding on his carabao.

Issue: Whether or not evidence exist to hold respondents civilly liable for the death of Wilson
Quinto.
Held: In the present case, we rule that, as held by the trial court and the CA, the prosecution
failed to adduce preponderant evidence to prove the facts on which the civil liability of the
respondents rest, i.e., that the petitioner has a cause of action against the respondents for
damages.
It bears stressing that the prosecution relied solely on the collective testimonies of Garcia, who
was not an eyewitness, and Dr. Aguda.
We agree with the petitioner that, as evidenced by the Necropsy Report of Dr. Dominic Aguda,
the deceased sustained a 14x7-centimeter hematoma on the scalp. But as to how the deceased
sustained the injury, Dr. Aguda was equivocal. He presented two possibilities: (a) that the
deceased could have been hit by a blunt object or instrument applied with full force; or (b) the
deceased could have slipped, fell hard and his head hit a hard object.

12. People V Pilola 405 SCRA 134

FACTS: February 5, 1988 11:30 pm: Elisa Rolan was inside their store waiting for her husband
to arrive. Joselito Capa and Julian Azul, Jr. were drinking beer. Although already drunk, Edmar
Aguilos and Odilon Lagliba joined them. Edmar had a heated argument with Julian. Elisa
pacified Edmar and advised them to go home as she was already going to close up. Edmar and
Odilon left then returned to block Joselito and Julian. Edmar took off his eyeglasses and
punched Julian in the face. Elisa shouted: Tama na. Tama na but she was ignored as they
continued until they reached the end of the street. Odilon positioned himself on top of a pile of
hollow blocks and watched as Edmar and Julian swapped punches. As Joselito tried to stop the
fight, Odilon pulled out his knife with his right hand and stepped down from his perch. He
placed his left arm around Joselitos neck, and stabbed him. Ronnie and Rene Gayot Pilola, who
were across the street, saw their gangmate Odilon stabbing the victim and decided to join the
fray. Ronnie took a knife from the kitchen of Teresita and rushed together with Pilola to the
scene and stabbed Joselito. As Joselito was stabbed 11 times (6 fatal stab wounds), he fell in the
canal. Odilon and Pilola fled while Ronnie went after Julian who ran dear life. When Julian
noticed that Ronnie was no longer running after him, he looked back and saw Ronnie pick up a
piece of hollow block and bashed Joselitos head. Then, Ronnie got a piece of broken bottle and
struck Joselito once more before fleing from the scene. Joselito died on the spot. Elisa rushed to
Joselitos house and informed his wife and brother of the incident.

ISSUE: Whether or not Pilola is guilty of murder.

Held: Yes. The identity of the person who hit the victim with a hollow block is of de minimis
importance. Elisas testimony is corroborated by the autopsy report of Dr. Bienvenido Muoz.
No showing of any improper motive on the part of a witness to testify falsely against the accused
or to falsely implicate the latter in the commission of the crime. The trial court gave credence
and full probative weight to Elisas testimony.

Even if two or more offenders do not conspire to commit homicide or murder, they may be held
criminally liable as principals by direct participation if they perform overt acts which mediately
or immediately cause or accelerate the death of the victim. Art. 4. Criminal liability. Criminal
liability shall be incurred:

1. By any person committing a felony (delito) although the wrongful act done be different from
that which he intended.

Odilon all by himself initially decided to stab the victim. However, while Odilon was stabbing
the victim, the appellant and Ronnie agreed to join. All the overt acts of Odilon, Ronnie and the
Pilola before, during, and after the stabbing incident indubitably show that they conspired to kill
the victim. Since the victim is not yet dead, the crime is not yet consummated so Pilola is a
principal by direct participation.

13. People V Balmores 85 PHIL 493

Facts: Balmores was found guilty of attempted estafa through falsification of a government
obligation. He attempted to cash in a sweepstakes ticket that was obviously falsified (the ticket
was split into, and the winning ticket number written in ink at the bottom left part of the halved
ticket). He presented his falsified ticket to a PCSO booth. The PCSO employee manning the
booth saw that the ticket was obviously falsified, and had Balmores arrested. Balmores waived
the right to counsel, and pleaded guilty to the crime of attempted estafa.

Issue: Whether or not Balmores committed an impossible crime.

Held: No. The recklessness and clumsiness of the act of falsification did not make the crime an
impossible one under Paragraph 2 Article 4 of the RPC.1
The alteration of a losing sweepstakes ticket would constitute a crime only if an attempt to cash
it were done, which is what occurred in this case.

14. Intod V. CA 215 SCRA 52

Facts: Intod and company were tasked to kill Palang-pangan due to land dispute. They fired at
her room. However,she was in another city then thus they hit no one.

Issue: WON he is liable for attempted murder?

Held: No. They are only guilty of an impossible crime. In the Philippines, Article 4(2)
provides and punishes an impossible crimean act which, were it not aimed at something quite
impossible or carried out with means which prove inadequate would constitute a felony against
person or family. Its purpose is to punish criminal tendencies. There must either be (1 )legal
responsibility, or (2) physical impossibility of accomplishing the intended act in order to qualify
the act as an impossible crime. Legal impossibility occurs where the intended acts even if
completed, would not amount to a crime. Thus: Legal impossibility would apply to those
circumstances where:(1) The motive, desire and expectation is to perform an act in violation
of the law;(2) There is no intention to perform the physical act;(3) There is a performance of
the intended physical act; and(4) The consequence resulting from the intended act does not
amount to a crime. Factual impossibility occurs when extraneous circumstances unknown to
actor or beyond control prevent consummation of intended crime. Factual impossibility of the
commission of the crime is not a defense. If the crime could have been committed had the
circumstances been as the defendant believed them to be, it is no defense that in reality, the
crime was impossible of commission. Legal impossibility on the other hand is a defense which
can be invoked to avoid criminal liability for an attempt. The factual situation in the case at bar
presents a physical impossibility which rendered the intended crime impossible of
accomplishment. And under Article 4, paragraph 2 of the Revised Penal Code, such is sufficient
to make the act an impossible crime.

15. Jacinto V. People G.R. 162540

Facts: Jacinto along with Valencia and Capitle was charged with qualified theft for having stolen
and deposited a check with an amount of 10,000 php. Such check was issued by Baby Aquino for
payment of her purchases from Mega Foam, but the check bounced. Dyhengco found out about
the theft and filed a complaint with the NBI. An entrapment operation was conducted, with the
use of marked bills. The entrapment was a success and the petitioner along with her co-accused
was arrested.

Issue: Whether this can constitute as an impossible crime and not as qualified theft

Held: This constitutes as an impossible crime. The requisites of an impossible crime are:

1. That the act performed would be an offense against persons or property;


2. That the act was done with evil intent;
3. That its accomplishment was inherently impossible or the means employed was either
inadequate or ineffectual or the extraneous circumstance that constituted it as a factual
impossibility.

Legal impossibility occurs where the intended acts, even if completed, would not amount to a
crime. (Impossibility of killing a dead person)

From the time the petitioner took possession of the check meant for Mega Foam, she had
performed all the acts to consummate the crime of theft, had it not been impossible of
accomplishment in this case. Replacement for the check was no longer necessary for
the consummation of the crime since the crime of theft is not a continuing offense, petitioners act
of receiving the cash replacement should not be considered as a continuation of the theft. The
fact that the petitioner was caught receiving the marked money was merely corroborating
evidence to strengthen proof of her intent to gain.

16. Valenzuela v. People 525 SCRA 306

Facts: On 19 May1994, Valenzuela and Calderon were seen outside the Super Sale Club inside
the SM Complex along North Edsa by Lorenzo Lago, SM Security Guard. Valenzuela was
hauling a push cart with cases of Tide detergent and unloaded these cases of Tide in an open
parking space where Calderon was waiting. Valenzuela went inside the supermarket again and
came back with more cases of detergent.

Thereafter, Valenzuela left the parking lot and haled a taxi. He boarded the cab and told the
driver to go to the area where Calderon was waiting. Calderon loaded the cases of Tide and
boarded the taxi. Lago saw all of this and proceeded to stop the taxi. When Lago asked for a
receipt of the merchandise, the two accused reacted by fleeing on foot. Valenzuela and Calderon
were apprehended at the scene.

Valenzuela and Calderon were both convicted by the trial court of consummated theft.

It was only Valenzuela who filed an appeal with the Court of Appeals. Petitioner contends that he
should only be convicted of frustrated theft since at the time he was apprehended, he was never
placed in a position to freely dispose of the articles stolen. The Court of Appeals rejected this
contention, hence, this Petition for Review.

Issue: Whether or not petitioner Valenzuela is guilty only of frustrated theft?

Decision: Petition dismissed. Under the statutory definition of theft, free disposal of the stolen
items is not a constitutive element of theft.

Under Article 308 of the Revised Penal Code, the crime of theft is defined as follows. Theft is
committed by any person who, with intent to gain but without force or violence against or
intimidation of persons nor force upon things, shall take the property of another without the
latters consent xxx

On the face of the definition, there is only one operative act of execution by the actor involved in
theft the taking of personal property of another. It is also clear from the definition that in order
such taking may be qualified as theft, there must further be present the descriptive circumstances
that the taking was with intent to gain; without force upon things or violence against or
intimidation of persons; and is was without the consent of the owner of the property.

For the purpose of ascertaining whether theft is susceptible of commissions in the frustrated
stage, the question is again, when is the crime of theft produced? There would be all but certain
unanimity in the position that theft is produced when there is deprivation of personal property
due to to its taking by one with intent to gain. Viewed from that perspective, it is immaterial to
the product of the felony that the offender, once having committed all the acts of execution for
theft, is able or unable to freely disposed of the property stolen since the deprivation from the
owner alone has already ensued from such acts of execution.

Indeed the SC, after all, held that unlawful taking is deemed complete from the moment of the
offender gains possession of the thing even if he has no opportunity to dispose of the same.

17. People V. Palaganas 501 SCRA 533

FACTS:January 16, 1998 8pm: Brothers Servillano, Melton and Michael Ferrer were on a
drinking spree in their house because Melton visited his brothers in Pangasinan all the way from
San Fernando, La Union. The brothers decided to go to Tidbits Videoke bar to continue their
drinking spree and to sing. They were the only customers at around10:30 pm, Jaime Palaganas,
Ferdinand Palaganas and Virgilio Bautista arrived and they occupied a different table. When
Jaime sang My Way, Melton sang along. But, Jaime resented this, approached the brother and
said in Pangasinan dialect "As if you are tough guys. You are already insulting me in that way."
Jaime struck Servillanos head with the microphone and a fight ensued. Virgilio Bautista did not
joined in and just left. During the rumble, Ferdinand went out of the bar. Michael was about to
pursue him but was stopped by Servillano. They went back to continue to fight with Jaime.
Edith Palaganas, sister of Jaime and the owner of the bar, arrived and pacified them. Servillano
noticed that his wristwatch was missing. Since the brothers could not locate it inside the bar,
they went outside. They saw Ferdinand at them and said to Rujjeric Palaganas "Oraratan paltog
mo lara" meaning "They are the ones, shoot them." Rujjeric shot Servillano first at the left
side of the abdomen penetrating his large intestine and urinary bladder causing him to fall on the
ground then Melton with a fatal shot on the head and on the right thigh. When Servillano noticed
that Melton was no longer moving, he told Michael "Bato, bato and they threw stones at
Rujjeric
and
Ferdinand.
Michael
was
hit
on
the
right
shoulder.
Several cases where filed against Rujjeric among them

Criminal Case No. U-9608: Shooting Servillano with unlicensed firearm

Criminal Case No. U-9609: Shooting Melton with unlicensed firearm

Criminal Case No. U-9610: Shooting Michael with unlicensed firearm


Criminal Case No. U-9634: using a caliber .38 without first securing the necessary
permit/license
in
violation
to
Comelec
Res.
2958
Rujjeric and Ferdinand entered separate pleas of "Not Guilty" Upon motion of Ferdinand, the
four
cases
were
consolidated.
RTC: Rujjeric was guilty of the crime of Homicide and 2 counts of Frustrated Homicide but
acquitted of the charge of Violation of COMELEC Resolution No. 2958 in relation to Section
261 of the Omnibus Election Code while Ferdinand was acquitted of all the charges against him.

CA
Affirmed
Rujjeric argued that all the elements of a valid self-defense are present in the instant case and,
thus, his acquittal on all the charges is proper; that when he fired his gun, he was then a victim of
an unlawful aggression perpetrated by the Ferrer brothers; that he, in fact, sustained an injury in

his left leg and left shoulder caused by the stones thrown by the Ferrer brothers

ISSUE: W/N Rujjeric was guilty of the crime of Homicide and 2 counts of Frustrated
Homicide.
HELD: YES. And is affirmed with the following modifications:

Criminal Case No. U-9608: Shooting Servillano with unlicensed firearm - attempted homicide.
There being a special aggravating circumstance of the use of an unlicensed firearm and applying
the Indeterminate Sentence of Law, the penalty now becomes four (4) years and two (2) months
of arresto mayor as minimum period to six (6) years of prision correccional as maximum period
Criminal Case No. U-9609: Shooting Melton with unlicensed firearm - homicide is reclusion
temporal - There being a special aggravating circumstance of the use of an unlicensed firearm
and applying the Indeterminate Sentence Law, the penalty now is twelve (12) years of prision
mayor as minimum period to twenty (20) years of reclusion temporal as maximum period
Criminal Case No. U-9610: Shooting Michael with unlicensed firearm - frustrated homicide.
There being a special aggravating circumstance of the use of an unlicensed firearm and applying
the Indeterminate Sentence Law, the penalty now becomes six (6) years of prision correccional
as minimum period to twelve (12) years of prision mayor as maximum period.
Petitioner argued that all the elements of a valid self-defense are present in the instant
case and, thus, his acquittal on all the charges is proper; that when he fired his gun on that fateful
night, he was then a victim of an unlawful aggression perpetrated by the Ferrer brothers; that he,
in fact, sustained an injury in his left leg and left shoulder caused by the stones thrown by the
Ferrer
brothers.
ART. 11. Justifying circumstances. The following do not incur any criminal liability:
1. Anyone who acts in defense of his person or rights, provided that the following
circumstances concur;

First.

Unlawful
aggression;
No unlawful aggression on the part of the Ferrer brothers that justified the act of
petitioner in shooting them. Ferrer brothers then were merely standing outside the videoke bar

and were not carrying any weapon. When the Ferrer brothers started throwing stones, petitioner
was not in a state of actual or imminent danger considering the wide distance (4-5 meters) of the
latter from the location of the former. He was still capable of avoiding the stones by running
away or by taking cover. He could have also called or proceeded to the proper authorities for
help.

Second. Reasonable necessity of the means employed to prevent or repel it;


the gun was far deadlier compared to the stones thrown by the Ferrer brothers.
Third. Lack of sufficient provocation on the part of the person defending himself.
Unlawful aggression is a primordial element in self-defense. It is an essential and indispensable
requisite, for without unlawful aggression on the part of the victim.
As the burden of evidence is shifted on the accused to prove all the elements of self-defense, he
must rely on the strength of his own evidence and not on the weakness of the prosecution.
1.) In frustrated felony, the offender has performed all the acts of execution which
should produce the felony as a consequence; whereas in attempted felony, the offender merely
commences the commission of a felony directly by overt acts and does not perform all the acts of
execution.
2.) In frustrated felony, the reason for the non-accomplishment of the crime is some cause
independent of the will of the perpetrator; on the other hand, in attempted felony, the reason for
the non-fulfillment of the crime is a cause or accident other than the offender's own spontaneous
desistance.
When the accused intended to kill his victim, as manifested by his use of a deadly weapon in his
assault, and his victim sustained fatal or mortal wound/s but did not die because of timely
medical assistance, the crime committed is frustrated murder or frustrated homicide depending
on whether or not any of the qualifying circumstances under Article 249 of the Revised Penal
Code are present. However, if the wound/s sustained by the victim in such a case were not fatal
or mortal, then the crime committed is only attempted murder or attempted homicide.
If there was no intent to kill on the part of the accused and the wound/s sustained by the victim
were not fatal, the crime committed may be serious, less serious or slight physical injury

18. Rivera V. People 480 SCRA 188

Facts: As the victim, Ruben Rodil, went to a nearby store to buy food, accused Edgardo Rivera
mocked him for being jobless and dependent on his wife for support. Ruben resented the rebuke
and thereafter, a heated exchange of words ensued. In the evening of the following day, when
Ruben and his three-year-old daughter went to the store to buy food, Edgardo, together with his
brother Esmeraldo Rivera and Ismael Rivera, emerged from their house and ganged up on
him. Esmeraldo and Ismael mauled Ruben with fist blows. And as he fell to the ground, Edgardo
hit him three times with a hollow block on the parietal area. Esmeraldo, Ismael and Edgardo fled
to their house only when the policemen arrived. Ruben sustained injuries and was brought to the
hospital. The doctor declared that the wounds were slight and superficial, though the victim
could have been killed had the police not promptly intervened. The trial court found the accused
guilty of the crime of frustrated murder. An appeal was made by the accused, but the Court of
Appeals affirmed the trial courts decision with modification, changing the crime to attempted
murder and imposed an indeterminate penalty of 2 years of prision correccional as minimum to 6
years and 1 day of prision mayor as maximum.

Issues: Whether or not the Court of Appeals was correct in modifying the crime from frustrated
to
attempted
murder.
Held: Yes. Article 6 of the Revised Penal Code provides that there is an attempt when the
offender commences the commission of a felony directly by overt acts, and does not perform all
the acts of execution which should produce the felony by reason of some cause or accident other
than his own spontaneous desistance. Although the wounds sustained by the victim were merely
superficial and could not have produced his death, it does not negate criminal liability of the
accused for attempted murder. The intent to kill was already presumed based on the overt acts of
the accused. In fact, victim could have been killed had the police not promptly intervened.

19. People V. Almazan 365 SCRA 373

On 28 September 1996, at about 4:00 o'clock in the afternoon, Vicente Madriaga and a certain
Allan played chess in front of the former's house at Pag-asa, Camarin, Caloocan City. Spectators
were Vicente's son Noli, who was carrying his 2-year old daughter, Vicente's grandson Noel, and
a neighbor named Angel Soliva. While the game was underway, Henry Almazan unexpectedly
arrived and brandished a .38 caliber revolver in front of the group. Almazan's fighting cocks had

just been stolen and he suspected Angel, one of the spectators, to be the culprit. Thus he said,
"manos-manos na lang tayo,"[2] aimed his gun at Angel and pulled the trigger. It did not
fire. He tried again, but again it failed.
At this juncture, Vicente Madriaga stood up and tried to calm down Henry, but the latter refused
to be pacified ("ayaw paawat"). Angel ran away and Henry aimed his gun instead at Noli. Noli
cried for mercy, for his life and that of his daughter, but to no avail. Henry shot Noli at the left
side of his stomach sending him immediately to the ground. His daughter, unscathed, held on to
Noli, crying. Henry then turned on Noel and shot him on the left thigh. Noel managed to walk
lamely ("paika-ika") but only to eventually fall to the ground. Thereafter, Vicente Madriaga
called on his neighbors who brought Noli and Noel to the hospital. Noli however died before
reaching the hospital, while Noel survived his injuries.
Dr. Ma. Cristina Freyra of the PNP Crime Laboratory Service conducted an autopsy on the body
of Noli which revealed that the cause of the victim's death was a gunshot at the trunk from a .38
caliber revolver. Dr. Misael Jonathan Ticman, attending physician of Noel, in turn declared that
the gunshot wound on the left thigh of Noel was a minor injury that would heal in a week. Noel
was never admitted in the hospital as his doctor sent him home the same day. On crossexamination, Dr. Ticman testified that if not medically treated the wound might get infected or
lead to the victim's death.

Issue: whether or not the accused should be liable only for physical injuries and not frustrated
murder in Crim. Case No. C-51277.

Held: The SC found that the accused-appellant should be held liable for attempted murder, not
frustrated murder. For the charge of frustrated murder to flourish, the victim should sustain a
fatal wound that could have caused his death were it not for timely medical assistance. The
court anchored its ruling on the statement of Dr. Ticman on cross-examination that the wound of
Noel could catch infection or lead to his death if not timely and properly treated. However, in
his direct testimony, Dr. Ticman declared that the wound was a mere minor injury for which
Noel, after undergoing treatment, was immediately advised to go home. He even referred to the
wound as a slight physical injury that would heal within a week and for which the victim was in
no danger of dying. Clear as the statement is, coupled with the fact that Noel was indeed
immediately advised to go home as he was not in any danger of death, we have no reason to
doubt the meaning and implications of Dr. Ticman's statement. His statement that Noel could
catch infection was based on pure speculation rather than on the actual nature of the wound
which was a mere minor injury, hence, not fatal. According to jurisprudence, if the victim was
wounded with an injury that was not fatal, and could not cause his death, the crime would only
be attempted. The observation that the conviction should be for slight physical injuries only is

likewise improper as the accused-appellant was motivated by the same impetus and intent, i.e., to
exact vengeance and even kill, if necessary, when he shot Noel Madriaga. The fact that the
wound was merely a minor injury which could heal in a week becomes inconsequential.

20. People V. Campuhan 329 SCRA 270

FACTS: April 25, 1996 4 pm: Ma. Corazon P. Pamintuan, mother of 4-year old Crysthel
Pamintuan, went to the ground floor of their house to prepare Milo chocolate drinks for her 2
children. There she met Primo Campuhan, helper of Conrado Plata Jr., brother of Corazon, who
was then busy filling small plastic bags with water to be frozen into ice in the freezer located at
the second floor.

Then she heard Crysthel cry, "Ayo'ko, ayo'ko!" so she went upstairs and saw Primo Campuhan
inside her children's room kneeling before Crysthel whose pajamas or "jogging pants" and panty
were already removed, while his short pants were down to his knees and his hands holding his
penis with his right hand. Horrified, she cursed "P - t - ng ina mo, anak ko iyan!" and boxed him
several times. He evaded her blows and pulled up his pants. He pushed Corazon aside who she
tried to block his path. Corazon then ran out and shouted for help thus prompting Vicente, her
brother, a cousin and an uncle who were living within their compound, to chase the Campuhan
who was apprehended.

However, the accused kept his innocence and contested that Pamintuans statements were not
credible
for
the
latter
has
ill
will
against
him.
ISSUE:

W/N

it

was

consummated

statutory

rape

Held: The records reviewed failed to show the proof whether Primos penis was able to penetrate
Chrystels vagina. Failure to prove such penetration, even the slightest one, cannot be considered
consummated rape, however, only attempted rape, if not acts of lasciviousness. Also, there were
no physical signs of injuries on the witness body to conclude a medical perspective that a
penetration has taken place. In rape cases, it is important that a valid testimony and medical
certificate complements each other, for relying alone on testimonial evidence may create
unwarranted or mischievous results. It is necessary to carefully establish a proof that the penis, in
reality, entered the labial threshold of the female organ to accurately conclude that the rape was

consummated. The decision of the court on convicting Campuhan guilty of statutory rape is
modified. Hence, he was convicted of attempted rape instead.

21. People V. Anticamara 651 SCRA 489

Facts:Lando, Al, Dick Taedo (Dick), Roberto Taedo (Bert), Marvin Lim (Marvin), Necitas
Ordeiza-Taedo (Cita), and Fred Doe are charged with the crimes of Murder and of
Kidnapping/Serious Illegal Detention in two separate Informations, for Murder and for
Kidnapping with serious illegal detention.
The charge for murder reads:

That on or about the early morning of May 7, 2002, in Sitio Rosalia,Brgy. San Bartolome,
Municipality of Rosales, Province of Pangasinan, and within the jurisdiction of this Honorable
Court, the above-named accused, being then armed with a hand gun, conspiring, confederating
and mutually helping one another, with intent to kill, with treachery, evident premeditation and
superior strength, did then and there, willfully, unlawfully and feloniously take Sulpacio Abad,
driver of the Estrellas, hog tied (sic) him, brought (sic) to a secluded place, shoot and bury in a
shallow grave, to the damage and prejudice of the heirs of the victim.

While the charge for Kidnapping with serious illegal detention reads as:

That on or about the 7th day of May 2002, more or less 3:00 oclockin the early morning, at the
Estrella Compound, Brgy. Carmen East, Municipality of Rosales, Province of Pangasinan, and
within the jurisdiction of this Honorable Court, the above-named accused, who are private
persons, conspiring, confederating and mutually helping
one another, armed with firearms, did then and there willfully, unlawfully and feloniously kidnap
Sulpacio Abad and AAA, both employees of the Estrellas, thereby depriving them of their
liberty, all against their will for a period of twenty-seven (27) days. That in the course of the
kidnapping, Sulpacio Abad was killed and buried in Brgy. Carmen, Rosales, Pangasinan and
AAA was raped for several times by her abductors. Contrary to Article 267 of the Revised Penal
Code.

In his defense, Lando denied having committed the crimes charged and interposed alibi as a
defense. He claims that at the time of the incident on May 7, 2002, he was in Barangay
Maligaya, San Miguel, Tarlac, with his family. He denied ever going to the Estrella farm in Sitio
Rosalia, Barangay San Bartolome, Rosales, Pangasinan.

Al claimed that he acted as a lookout and was tasked to report to his companions if any person or
vehicle would approach the house of the Estrellas. He said that he was forced to follow what was
ordered of him and did not report the matter to the police because he was threatened to be killed,
including the members of his family who were in Cebu.

Issue: Whether or not the prosecution failed to prove that he was a co-conspirator in the
kidnapping charge and that he did not participate in the rape of the victim.

Held: To be a conspirator, one need not participate in every detail of the execution; he need not
even take part in every act or need not even know the exact part to be performed by the others in
the execution of the conspiracy. Each conspirator may be assigned separate and different tasks
which may appear unrelated to one another but, in fact, constitute a whole collective effort to
achieve their common criminal objective. Once conspiracy is shown, the act of one is the act of
all the conspirators. The precise extent or modality of participation of each of them becomes
secondary, since all the conspirators are principals

Appellant Al attempts to evade criminal liability by alleging that he was only forced to
participate in the commission of the crime because he and his family were threatened to be
killed. Als defense fails to impress us. Under Article 12 of the Revised Penal Code, a person is
exempt from criminal liability if heacts under the compulsion of an irresistible force, or under the
impulse of an uncontrollable fear of equal or greater injury, because such person does not act
with freedom. To avail of this exempting circumstance, the evidence must establish: (1) the
existence of an uncontrollable fear; (2) that the fear must be real and imminent; and (3) the fear
of an injury is greater than, or at least equal to, that committed. For such defense to prosper, the
duress, force, fear or intimidation must be present, imminent and impending, and of such nature
as to induce a well-grounded apprehension of death or serious bodily harm if the act be done. A
threat of future injury is not enough.

There is nothing in the records to substantiate appellant Als insistence that he was under duress
from his co-accused while participating in the crime that would suffice to exempt himfrom
incurring criminal liability. The evidence shows that Al was tasked to act as a lookout and
directed to station himselfacross the house of the Estrellas. Al was there from 7:30 p.m. to 1:00
a.m.21 of the following day, while the rest of the group was waiting in the landing field. Thus,
while all alone, Al had every opportunity to escape since he was no longer subjected to a real,
imminent or reasonable fear. However, he opted to stay across the house of the Estrellas for
almost six (6) hours,22 and thereafter returned to the landing field where the group was waiting
for his report. Subsequently, the group proceeded to the Estrellas house. When the group entered
the house, Al stayed for almost one (1) hour outside to wait for his companions. Later, when the
group left the house aboard a vehicle, Al rode with them in going to Sitio Rosalia, Brgy. San
Bartolome, Rosales, Pangasinan, bringing with them Sulpacio and AAA. Clearly, appellant Al
had ample opportunity to escape if he wished to, but he never did. Neither did he request for
assistance from the authorities or any person passing by the house of the Estrellas during the
period he was stationed there. Clearly, Al did not make any effort to perform an overt act to
dissociate or detach himself from the conspiracy to commit the felony and prevent the
commission thereof that would exempt himself from criminal liability.24 Therefore, it is obvious
that he willingly agreed to be a part of the conspiracy.

22. People V. Baharan 639 SCRA 157

Facts: On 14 February 2005, an RRCG bus was plying its usual southbound route, from its
Navotas bus terminal towards its Alabang bus terminal via Epifanio de los Santos Avenue
(EDSA). Around 6:30 to 7:30 in the evening, while they were about to move out of the
Guadalupe-EDSA southbound bus stop, the bus conductor noticed two men running after the
bus. The two insisted on getting on the bus, so the conductor obliged and let them in.

According to Elmer Andales, the bus conductor, he immediately became wary of the two men,
because, even if they got on the bus together, the two sat away from each other one sat two
seats behind the driver, while the other sat at the back of the bus. At the time, there were only 15
passengers inside the bus. He also noticed that the eyes of one of the men were reddish. When he
approached the person near the driver and asked him whether he was paying for two passengers,
the latter looked dumb struck by the question. He then stuttered and said he was paying for two
and gave PhP20. Andales grew more concerned when the other man seated at the back also paid
for both passengers. At this point, Andales said he became more certain that the two were up to
no good, and that there might be a holdup.

Afterwards, Andales said he became more suspicious because both men kept on asking him if the
bus was going to stop at Ayala Avenue. The witness also noticed that the man at the back
appeared to be slouching, with his legs stretched out in front of him and his arms hanging out
and hidden from view as if he was tinkering with something. When Andales would get near the
man, the latter would glare at him. Andales admitted, however, that he did not report the
suspicious characters to the police.

As soon as the bus reached the stoplight at the corner of Ayala Avenue and EDSA, the two men
insisted on getting off the bus. According to Andales, the bus driver initially did not want to let
them off the bus, because a Makati ordinance prohibited unloading anywhere except at
designated bus stops. Eventually, the bus driver gave in and allowed the two passengers to alight.

The two immediately got off the bus and ran towards Ayala Avenue. Moments after, Andales felt
an explosion. He then saw fire quickly engulfing the bus. He ran out of the bus towards a nearby
mall. After a while, he went back to where the bus was. He saw their bus passengers either lying
on the ground or looking traumatized. A few hours after, he made a statement before the Makati
Police Station narrating the whole incident.

Issue: Whether there was sufficient proof to prove that there was conspiracy from the acts of the
accused and the confessions made by the witnesses.

Held: The Court affirms the finding of the existence of conspiracy involving accused Baharan,
Trinidad, and Rohmat. Conspiracy was clearly established from the collective acts of the
accused appellants before, during and after the commission of the crime. As correctly declared
by the trial court in its Omnibus Decision

While said conspiracy involving the four malefactors has not been expressly admitted by accused
Baharan, Angelo Trinidad, and Rohmat, more specifically with respect to the latters
participation in the commission of the crimes, nonetheless it has been established by virtue of the
aforementioned evidence, which established the existence of the conspiracy itself and the
indispensable participation of accused Rohmat in seeing to it that the conspirators criminal
design would be realized.

It is well-established that conspiracy may be inferred from the acts of the accused, which clearly
manifests a concurrence of wills, a common intent or design to commit a crime (People v.
Lenantud, 352 SCRA 544). Hence, where acts of the accused collectively and individually
demonstrate the existence of a common design towardsthe accomplishment of the same unlawful
purpose, conspiracy is evident and all the perpetrators will be held liable as principals.

23. People V. Montanir 647 SCRA 170

Facts: Rosalina Reyes and her partner Rafael Mendoza was kidnapped by the group of Josie
Herrera, Robert Uy, Alicia a.k.a. Alice Buenaflor, together with appellants Ronald Norva and
Eduardo Chua. In the course of the kidnapping Rafael Mendoza died of cardiac arrest while
Rosalina escaped with the help of some of her kidnappers Larry, Jack and Boy. She would later
ask the help of her attorney to file a case against her kidnappers.

Upon arraignment, with the assistance of counsel, Jonard and appellants Ronald, Dima and
Eduardo, pleaded not guiltyto the crime charged. Robert Uy, Alice Buenaflor and Jessie Doe
remained at-large during the trial of the case. Jonard was later on discharged as a state witness.
Afterwards, the trial on
the merits ensued.

Issue: whether or not Chua, Norva and Montanir is guilty of conspiracy to kidnapping.

Held: Yes. When conspiracy is established, the responsibility of the conspirators is collective, not
individual. This renders all of them equally liable regardless of the extent of their respective
participations, the act of one being deemed to be the act of the other or the others, in the
commission of the felony.

Each conspirator is responsible for everything done by his confederates which follows
incidentally in the execution of a common design as one of its probable and natural
consequences even though it was not intended as part of the original design. Responsibility of a
conspirator is not confined to the accomplishment of a particular purpose of conspiracy but
extends to collateral acts and offenses incident to and growing out of the purpose intended.

Conspirators are held to have intended the consequences of their acts and by purposely engaging
in conspiracy which necessarily and directly produces a prohibited result, they are, in
contemplation of law, chargeable with intending that result. Conspirators are necessarily liable
for the acts of another conspirator unless such act differs radically and substantively from that
which they intended to commit.

24. People v. Quirol 473 SCRA 509

Facts: On December 4, 1993, in celebration of a fiesta in Apas, Lahug, Cebu City, a benefit
disco dance was held at the local UCMA Village. Appellants, Juanito and Mario Quirol, and the
two victims, Benjamin Silva and Roel Ngujo, attended.

At the dance, Juanito, Mario and Jed were together and drank all through the night with some
friends. The dance ended just prior to 4 a.m. and prosecution principal witness Wilson Cruz
testified that it was about that time when he was asked by Benjamin and Roel to accompany
them in escorting some ladies home.

Wilson told them to go ahead and that he would just follow. Wilson was behind them at a
distance of 7 to 10 fathoms when the group passed by the house of Jed. From his vantage point,
Wilson saw Jed stop the two victims in front of his house and frisk them.

Thereafter, Wilson saw Jed bind Benjamin and Roel together with a pair of handcuffs and lead
them towards the control tower of the old airport of Lahug, Cebu City. There, the three were met
by Juanito and Mario and together they proceeded to the airport runway.

Wilson, hidden behind a bush, said he could hear Benjamin plead for his life. A few seconds
later, Jed took out his .38 caliber service revolver and shot Benjamin at point-blank range on the
head. As Benjamin fell, Roel was dragged down to his knees since he was handcuffed to
Benjamin. Mario then held Roel while Juanito started stabbing him using a Batangas knife. Jed
finished it by shooting Roel.

The lower court and Court of Appeals find that there was conspiracy and convicting them despite
their defense of alibi.

Issue: Can there be a conspiracy based on the foregoing facts?

Decision: Conspiracy need not be proven by direct evidence of prior agreement to commit the
crime.Neither it is necessary to show that all the conspirators actually hit and killed the victim.
What has to be shown is that all the participants performed specific acts with such closeness and
coordination as to unmistakably indicate a common purpose and design. The conspiracy in the
instant case was sufficiently proven by Jed meeting with appellants at the old airport tower and
walking together with them towards the runway where appellants and Jed performed acts in
unison with each other as to unmistakably reveal a common purpose and design.

Anent Marios defense of alibi, despite corroboration from Exequiel Aranas, it is still an
inherently weak defense and cannot prevail over a positive identification from a witness found
credible by the trial court. Absent arbitrariness or oversight of some fact or circumstance of
significance and influence, we will not interfere with the credence given to the testimony of
Wilson over that of Mario and that of Exequiel, as assessments of credibility are generally left to
the trial court whose proximate contact with those who take the witness stand places it in a more
competent position to discriminate between true and false testimony. Moreover, as correctly
discussed by the Court of Appeals, the distance between the scene of the crimes and where Mario
claims he passed out is not so far away as to prevent him from being physically present at the
place of the crimes or its immediate vicinity at the time the crimes were committed.

25. People V. Bulan 459 SCRA 550

Facts: Jose Bulan and his sons, Allan and Estemson, were charged with murder in an Information
filed on October 11, 1994. The accusatory portion of the Information reads:
That on or about the 6th day of June 1994 at Barangay Datag, Municipality of Caramoran,
Province of Catanduanes, Philippines, within the jurisdiction of this Honorable Court, the abovenamed accused, conspiring, confederating and helping one another for a common purpose, that
is, to kill with treachery and evident premeditation, did then and there, wilfully, unlawfully and
feloniously, Jose Bulan and Allan Bulan held both hands of Alberto Mariano to deprive him of
any defense, while Estemson Bulan stabbed him from behind with a deadly weapon, hitting him
twice at the back which resulted to his instantaneous death, to the damage and prejudice of the
victims heirs.
Jose and Allan were duly arraigned on March 6, 1995, and pleaded not guilty. Estemson, on the
other hand, remained at large.
After trial, the trial court rendered judgment convicting the accused of murder as accomplices.
The trial court declared that there was no conspiracy between the appellants and Estemson.
On appeal to the CA, the appellate court rendered judgment on June 13, 2000 affirming the
decision of the RTC, with the modification that the appellants and Estemson were guilty of
murder as principals by indispensable cooperation. The appellate court declared that as gleaned
from the evidence on record, the three of them conspired to kill the deceased.
This case was certified by the Court of Appeals (CA) to this Court for review, in view of its
finding in its Decision that appellants Jose Bulan and his son, Allan Bulan, are guilty of murder
as principals by indispensable cooperation punishable by reclusion perpetua to death, and not
merely as accomplices as found by the Regional Trial Court (RTC) of Virac, Catanduanes,
Branch 42.
Issue: Whether or not there was conspiracy in this case.
Held: Yes, there is conspiracy. Article 8 of the Revised Penal Code provides that there is
conspiracy when two or more persons agree to commit a crime and decide to commit it. Direct

proof is not essential to prove conspiracy; it may be established by acts of the accused before,
during and after the commission of the crime charged, from which it may be logically inferred
the existence of a common purpose to commit the same. The prosecution must prove conspiracy
by the same quantum of evidence as the felony charged itself. Indeed, proof of previous
agreement among the malefactors to commit the crime is not essential to prove conspiracy. It is
not necessary to show that all the conspirators actually hit and killed the victim; what is
primordial is that all the participants performed specific acts with such closeness and
coordination as to indicate a common purpose or design to bring out the victims death. Once
conspiracy is established, it is unnecessary to prove who among the conspirators inflicted the
fatal injury. If conspiracy is proved, all the conspirators are criminally liable for the crime
charged and proved. The act of one is the act of all.
In this case, the appellants were waiting outside the dance hall near the gate when Edwin Solo
brought the victim towards them, onto the street. Jose held the victim by the right shoulder,
while Allan held him by the left. Estemson suddenly appeared from behind the victim and
stabbed the latter at the back with a small bolo. The appellants continued holding the victim as
Estemson stabbed him yet again. Even as Estemson fled, the appellants dragged the victim from
the gate, towards the store, where they dropped the victims body and fled from the scene. Allan
then left Catanduanes and hid in Pasay City where he was arrested by the NBI on August 7,
1994.
Considering the foregoing, the Court affirms the finding of the CA that the appellants are guilty
as principals by direct participation in the killing of Alberto Mariano

26. People V. Comadre 431 SCRA 366

Facts: As culled from the records, at around 7:00 in the evening of August 6, 1995, Robert
Agbanlog, Jimmy Wabe, Gerry Bullanday, Rey Camat and Lorenzo Eugenio were having a
drinking spree on the terrace of the house of Roberts father, Barangay Councilman Jaime
Agbanlog, situated in Barangay San Pedro, Lupao, Nueva Ecija. Jaime Agbanlog was seated on
the banister of the terrace listening to the conversation of the companions of his son.
As the drinking session went on, Robert and the others noticed appellants Antonio Comadre,
George Comadre and Danilo Lozano walking. The three stopped in front of the house. While
his companions looked on, Antonio suddenly lobbed an object which fell on the roof of the
terrace. Appellants immediately fled by scaling the fence of a nearby school.

The object, which turned out to be a hand grenade, exploded ripping a hole in the roof of the
house. Robert Agbanlog, Jimmy Wabe, Gerry Bullanday, Rey Camat and Lorenzo Eugenio were
hit by shrapnel and slumped unconscious on the floor. They were all rushed to the San Jose
General Hospital in Lupao, Nueva Ecija for medical treatment. However, Robert Agbanlog died
before reaching the hospital.

Dr. Tirso de los Santos, the medico-legal officer who conducted the autopsy on the cadaver of
Robert Agbanlog, certified that the wounds sustained by the victim were consistent with the
injuries inflicted by a grenade explosion and that the direct cause of death was hypovolemic
shock due to hand grenade explosion. The surviving victims, Jimmy Wabe, Rey Camat, Jaime
Agbanlog and Gerry Bullanday sustained shrapnel injuries.

Issue: Whether or not the presence of George Comadre and Danilo Lozano at the time the
grenade was thrown by Antonio Comadre, enough to make them as co-conspirators and prove
the existence of conspiracy among them.

Held: No. Similar to the physical act constituting the crime itself, the elements of conspiracy
must be proven beyond reasonable doubt. Settled is the rule that to establish conspiracy, evidence
of actual cooperation rather than mere cognizance or approval of an illegal act is required.
A conspiracy must be established by positive and conclusive evidence. It must be shown to exist
as clearly and convincingly as the commission of the crime itself. Mere presence of a person at
the scene of the crime does not make him a conspirator for conspiracy transcends
companionship.
The evidence shows that George Comadre and Danilo Lozano did not have any participation in
the commission of the crime and must therefore be set free. Their mere presence at the scene of
the crime as well as their close relationship with Antonio are insufficient to establish conspiracy
considering that they performed no positive act in furtherance of the crime.
Neither was it proven that their act of running away with Antonio was an act of giving moral
assistance to his criminal act. The ratiocination of the trial court that their presence provided
encouragement and sense of security to Antonio, is devoid of any factual basis. Such finding is
not supported by the evidence on record and cannot therefore be a valid basis of a finding of
conspiracy.

Time and again we have been guided by the principle that it would be better to set free ten men
who might be probably guilty of the crime charged than to convict one innocent man for a crime
he did not commit. There being no conspiracy, only Antonio Comadre must answer for the crime.
27. People V. Ramos 427 SCRA 299
Facts: Eulalia San Roque de Francisco, was charged with murder along with Narciso Ramos,
Ramon San Roque, Wilfredo Ramos and three (3) other John Does.
The the case began when William Lomida was picked up in his house by Narciso Ramos, Ramon
San Roque and three (3) others, he was then brought to Narcisos house where he pleaded for his
life with the appellant Eulalia and his live in partner but was nevertheless stabbed, shot and later
burned in a pile of rubber.
All this was seen by the witness Bernie Ambal and later Saturnino Rivera which proceeded to
call the NBI and became witnesses.
On April 24, 1998, the trial court rendered a Decision, the dispositive portion of which reads:
WHEREFORE, judgment is hereby rendered, finding the accused Eulalia San Roque de
Francisco y dela Cruz alias Laling GUILTY beyond reasonable doubt of the crime of murder as
charged in the Information and hereby sentences her to suffer the penalty of reclusion perpetua
with accessory penalties as provided by the law and to indemnify the heirs of the victim in the
sum of P50,000.00.
Issue: whether there is enough evidence to prove that Eulalia was part of the conspiracy to kill
William.
Held: Yes. In determining the existence of conspiracy, it is not necessary to show that all the
conspirators actually hit and killed the victim. The presence of conspiracy among the accused
can be proven by their conduct before, during or after the commission of the crime showing that
they acted in unison with each other, evincing a common purpose or design. There must be a
showing that appellant cooperated in the commission of the offense, either morally, through
advice, encouragement or agreement or materially through external acts indicating a manifest
intent of supplying aid in the perpetration of the crime in an efficacious way. In such case, the
act of one becomes the act of all, and each of the accused will thereby be deemed equally guilty
of the crime committed.
The series of events in this case convincingly show that appellant and her co-accused acted in
unison and cooperated with each other in killing William Lomida. Appellant was the one who
opened the door and allowed the other accused to enter the house. She joined them in bringing
the victim to the residence of Narciso Ramos, her brother-in-law. While her co-accused dragged
the helpless victim, tied him to a santol tree, stabbed him twice by a bladed knife, and shot him 5
to 7 times, appellant merely watched intensely. She even turned her back as the lifeless body

of the victim was being burned. And after attaining their purpose, she fled with the other
accused.
The above circumstances clearly show the common purpose and concerted efforts on the part of
appellant and her co-accused. We agree with the trial court in concluding that their acts were
indications of a criminal conspiracy to commit the crime of murder.

28. People V. Pacificador 376 SCRA 180

Facts: On April 16, 1986, senior state prosecutor Tirso C. Velasco filed with the Regional Trial
Court, Antique, San Jose, Branch 11 an amended information for multiple murder and frustrated
murder against Arturo F. Pacificador, S/Sgt. Domingo Dalmacio PC, Sgt. Enrico Cabaero PC,
C2C Reynaldo Aliupala PC, Pfc Vicente Vegafria PA, Cpl. Hector Fullon INP, and Pat. Lorenzo
Mingote INP, which reads:
"That on or about May 13, 1984 in the evening, at the Municipality of Sibalom, Province of
Antique, Philippines, a place within the jurisdiction of this Honorable Court, accused
Assemblyman ARTURO F. PACIFICADOR, his military security men: S/SGT. DOMINGO
DALMACIO PC, SGT. ENRICO CABAERO PC, C2C REYNALDO ALIPALA PC, PFC
VICENTE VEGAFRIA PA, CPL. HECTOR FULLON INP and PAT LORENZO MINGOTE
INP, whose military jurisdiction over their case had been waived by the President of the
Philippines pursuant to P. D. 1952 in favor of the civil court, conspiring, confederating and
mutually helping one another, did, then and there, willfully, unlawfully and feloniously, with
intent to kill, treachery,and evident premeditation, attack, assault and shoot Rhium Sanchez,
Plaridel Sanchez IV, Aldrick Sanchez, Mamerto Zaldivar, Jr., Clemente Samulde, Abner Varon
and Pat. Armelito Tamboong with the use of armalite rifles and explosives thus inflicting gunshot
wounds on vital parts of their bodies which caused their death as a consequence, and the
wounding of Luna Sanchez on vital parts of his body which ordinarily would cause his death
thus performing all the acts of execution which should have produced the crime of murder as a
consequence but nevertheless did not produce it by reason of cause independent of his will, that
is, by the timely and able medical assistance rendered to Luna Sanchez which prevented his
death.
"CONTRARY TO LAW with aggravating circumstances of nighttime, by a band, with evident
premeditation or treachery, superior strength and by means of motor vehicle.

Accused S/Sgt. Domingo Dalmacio, C2C Reynaldo Alipala, PFC Vicente Vegafria, PCpl. Hector
Fullon and Pat. Lorenzo Mingote, appealed from the decision of the Regional Trial Court,

Branch 11, San Jose, Antique finding them guilty beyond reasonable doubt of multiple murder
committed against Clemente Samulde, Rhium Sanchez, Plaridel Sanchez IV, Aldrick Sanchez,
Mamerto Zaldivar, Jr., Armelito Tamboong and Abner Varon and frustrated murder committed
against Luna Sanchez.
On March 6, 2000, accused-appellants filed their brief. Accused-appellants contend that the trial
court erred in ruling that: (1) murder has been proved beyond reasonable doubt by the
prosecution; (2) sufficient evidence existed to establish conspiracy; (3) the aggravating
circumstances of treachery and evident premeditation were present. They reiterated that the
group of Luna Sanchez was the first to fire against them and they merely fired back as an act of
retaliation and in self-defense. Hence, they claim that it was erroneous for the trial court to find
all of them guilty of multiple murder and frustrated murder and sentencing them accordingly.
Accused-appellants anchor their defense on the theory that the group of the victims staged the
ambush, and that they merely retaliated and fired back as an act of self-defense. In support of
such theory, they presented evidence that all the deceased on the side of the prosecution were
found positive for nitrates indicating that they fired guns, some of which are armalite rifles,
others revolvers or pistols.

Issue: Whether or not there is sufficient evidence to prove conspiracy notwithstanding that the
victims were positive for nitrates, an indication that they have fired their weapons and that the
ambush was staged by the victims.
Held: Yes, conspiracy was sufficiently proven in this case. Conspiracy need not be established by
direct evidence, for it may be inferred from the conduct of the accused, before, during, and after
the commission of the crime, which, if all taken together, would reasonably be strong enough to
show a community of criminal design.
Prosecution witness Luna Sanchez, was present at the scene of the crime, he being one of the
victims. He stated that during the election campaign trails on May 13, 1984, the group of
Pacificador, escorted by the accused-appellants overtook their Ford Tamaraw vehicle several
times, and the latter pointed their guns against them, making it very apparent that accusedappellants were tailing the group of Luna Sanchez. And ultimately, accused-appellants
perpetrated their unlawful design against the group of Luna Sanchez when they strategically
positioned themselves at the southern end of the single-lane Pangpang Bridge in Sibalom,
Antique, parked their vehicle near the foot of the bridge making sure that the group of Luna
Sanchez would not be able pass through and took cover in the nearby canal and waited for the
arrival of their prey. The simultaneous acts of leaving, waiting for their victims to come out,
tailing and firing at them continuously at close range, and escaping from the crime scene clearly
establish a conspiracy among the malefactors.

Hence, the trial court did not err when it ruled that conspiracy exists. Where the acts of the
accused collectively and individually demonstrate the existence of a common design towards the
accomplishment of the same unlawful purpose, conspiracy is evident, and all the perpetrators
will be liable as principals.

Supplementary Application of the RPC provisions to Special Laws.

29. Ladonga V. People 451 SCRA 673

Facts: Facts: In 1989, spouses Adronico and Evangeline Ladonga became Alfredo Oculams
regular customers in his pawnshop business. Sometime in May 1990, the Ladonga spouses
obtained a P9,075.55 loan from him, guaranteed by United Coconut Planters Bank (UCPB)
Check No. 284743, post dated to July 7, 1990 issued by Adronico; sometime in the last week of
April 1990 and during the first week of May 1990, the Ladonga spouses obtained an additional
loan of P12,730.00, guaranteed by UCPB Check No. 284744, post dated to July 26, 1990 issued
by Adronico; between May and June 1990, the Ladonga spouses obtained a third loan in the
amount of P8,496.55, guaranteed by UCPB Check No. 106136, post dated to July 22, 1990
issued by Adronico; the three checks bounced upon presentment for the reason CLOSED
ACCOUNT; when the Ladonga spouses failed to redeem the check, despite repeated demands,
he filed a criminal complaint against them. While admitting that the checks issued by Adronico
bounced because there was no sufficient deposit or the account was closed, the Ladonga spouses
claimed that the checks were issued only to guarantee the obligation, with an agreement that
Oculam should not encash the checks when they mature; and, that petitioner is not a signatory of
the checks and had no participation in the issuance thereof. The RTC rendered a joint decision
finding the Ladonga spouses guilty beyond reasonable doubt of violating B.P. Blg. 22. Petitioner
brought the case to the Court of Appeals. The Court of Appeals affirmed the conviction of
petitioner.

Issue: Whether or not the RTC erred in finding her criminally liable for conspiring with her
husband as the principle of conspiracy is inapplicable to B.P. Blg. 22 which is a special law;
moreover, she is not a signatory of the checks and had no participation in the issuance thereof.

Held: Article 10 is composed of two clauses. The first provides that offenses which in the future
are made punishable under special laws are not subject to the provisions of the RPC, while the
second makes the RPC supplementary to such laws. While it seems that the two clauses are
contradictory, a sensible interpretation will show that they can perfectly be reconciled.
The first clause should be understood to mean only that the special penal laws are controlling
with regard to offenses therein specifically punished. Said clause only restates the elemental rule
of statutory construction that special legal provisions prevail over general ones. Lex specialis
derogant generali. In fact, the clause can be considered as a superfluity, and could have been
eliminated altogether. The second clause contains the soul of the article. The main idea and
purpose of the article is embodied in the provision that the "code shall be supplementary" to
special laws, unless the latter should specifically provide the contrary.
The appellate courts reliance on the cases of People vs. Parel, U.S. vs. Ponte, and U.S. vs.
Bruhez rests on a firm basis. These cases involved the suppletory application of principles under
the then Penal Code to special laws. People vs. Parel is concerned with the application of Article
22 of the Code to violations of Act No. 3030, the Election Law, with reference to the retroactive
effect of penal laws if they favor the accused. U.S. vs. Ponte involved the application of Article
17 of the same Penal Code, with reference to the participation of principals in the commission of
the crime of misappropriation of public funds as defined and penalized by Act No. 1740. U.S. vs.
Bruhez covered Article 45 of the same Code, with reference to the confiscation of the
instruments used in violation of Act No. 1461, the Opium Law.
B.P. Blg. 22 does not expressly proscribe the suppletory application of the provisions of the RPC.
Thus, in the absence of contrary provision in B.P. Blg. 22, the general provisions of the RPC
which, by their nature, are necessarily applicable, may be applied suppletorily. Indeed, in the
recent case of Yu vs. People, the Court applied suppletorily the provisions on subsidiary
imprisonment under Article 39 of the RPC to B.P. Blg. 22.
The suppletory application of the principle of conspiracy in this case is analogous to the
application of the provision on principals under Article 17 in U.S. vs. Ponte. For once conspiracy
or action in concert to achieve a criminal design is shown, the act of one is the act of all the
conspirators, and the precise extent or modality of participation of each of them becomes
secondary, since all the conspirators are principals.
Notwithstanding, the accused Evangeline is acquitted for failure of the prosecution to prove
conspiracy and her guilt beyond reasonable doubt.

30. People V. Bustinera 431 SCRA 673

Facts:ESC Transport hired Luisito Bustinera as a taxi driver. It was agreed that appellant would
drive the taxi from 6:00 a.m. to 11:00 p.m., after which he would return it to ESC Transport's
garage and remit the boundary fee in the amount of P780.00 per day. On December 25,1996,
appellant admittedly reported for work and drove the taxi, but he did not return it on the same
day as he was supposed to. The owner of ESC reported the taxi stolen. On January 9, 1997,
Bustinera's wife went to ESC Transport and revealed that the taxi had been abandoned. ESC was
able to recover. The trial court found him guilty beyond reasonable doubt of qualified theft.

Issue: whether or not the accused was rightfully convicted of qualified theft.

Held:Bustinera was convicted of qualified theft under Article 310 of the Revised Penal Code, as
amended for the unlawful taking of a motor vehicle. However, Article 310 has been modified,
with respect to certain vehicles, by Republic Act No. 6539, as amended, otherwise known as
"AN ACT PREVENTING AND PENALIZING CARNAPPING. "When statutes are in pari
materia or when they relate to the same person or thing, or to the same class of persons or things,
or cover the same specific or particular subject matter, or have the same purpose or object, the
rule dictates that they should be construed together. The elements of the crime of theft as
provided for in Article 308 of the Revised Penal Code are: (1) that there be taking of personal
property; (2) that said property belongs to another; (3) that the taking be done with intent to gain;
(4) that the taking be done without the consent of the owner; and (5) that the taking be
accomplished without the use of violence against or intimidation of persons or force upon things.

Theft is qualified when any of the following circumstances is present: (1) the theft is committed
by a domestic servant; (2) the theft is committed with grave abuse of confidence; (3) the property
stolen is either a motor vehicle, mail matter or large cattle; (4) the property stolen consists of
coconuts taken from the premises of a plantation; (5) the property stolen is fish taken from a fish
pond or fishery; and (6) the property was taken on the occasion of fire, earthquake, typhoon,
volcanic eruption, or any other calamity, vehicular accident or civil disturbance. On the other
hand, Section 2 of Republic Act No.6539, as amended defines "car napping" as "the taking, with
intent to gain, of a motor vehicle belonging to another without the latter's consent, or by means
of violence against or intimidation of persons, or by using force upon things." The elements of
car napping are thus: (1) the taking of a motor vehicle which belongs to another; (2) the taking is
without the consent of the owner or by means of violence against or intimidation of persons or
by using force upon things; and (3) the taking is done with intent to gain. Car napping is
essentially the robbery or theft of a motorized vehicle, the concept of unlawful taking in theft,

robbery and car napping being the same. From the foregoing, since appellant is being accused of
the unlawful taking of a Daewoo sedan, it is the anti-car napping law and not the provisions of
qualified theft which would apply.

31. Tan V. Spouses Tan 451 SCRA 673

Facts: On April 18, 1999, Sharica Mari L. Go-Tan (petitioner) and Steven L. Tan (Steven) were
married. Out of this union, two female children were born, Kyra Danielle and Kristen
Denise. On January 12, 2005, barely six years into the marriage, petitioner filed a Petition with
Prayer for the Issuance of a Temporary Protective Order (TPO) against Steven and her parentsin-law, Spouses Perfecto C. Tan and Juanita L. Tan (respondents) before the RTC. She alleged
that Steven, in conspiracy with respondents, were causing verbal, psychological and economic
abuses upon her in violation of Section 5, paragraphs (e)(2)(3)(4), (h)(5), and (i) of Republic Act
(R.A.) No. 9262, otherwise known as the Anti-Violence Against Women and Their Children Act
of 2004.

On March 7, 2005, the RTC issued a Resolution dismissing the case as to respondents on the
ground that, being the parents-in-law of the petitioner, they were not included/covered as
respondents under R.A. No. 9262 under the well-known rule of law
expressio unius est exclusio alterius.

Issue: Whether or not the RPC rules on conspiracy can apply to a special law (R.A. 9262)

Held: Yes. The SC ruled that while RA 9262 provides that the offender be related or connected
to the victim by marriage, former marriage, or a sexual or dating relationship, it does not
preclude the application of the principle of conspiracy under the RPC.
Hence, legal principles developed from the Penal Code may be applied in a supplementary
capacity to crimes punished under special laws, such as R.A. No. 9262, in which the special law
is
silent
on
a
particular
matter.
The principle of conspiracy under Article 8 of the RPC may be applied suppletorily to R.A. No.
9262 because of the express provision of Section 47 that the RPC shall be supplementary to said
law. Thus, general provisions of the RPC, which by their nature, are necessarily applicable, may

be

applied

suppletorily.

"Thus, the principle of conspiracy may be applied to R.A. No. 9262. For once conspiracy
or action in concert to achieve a criminal design is shown, fellester.blogspot.com the act of one is
the act of all the conspirators, and the precise extent or modality of participation of each of them
becomes
secondary,
since
all
the
conspirators
are
principals."
It bears mention that the intent of the statute is the law and that this intent must be
effectuated by the courts. In the present case, the express language of R.A. No. 9262 reflects the
intent of the legislature for liberal construction as will best ensure the attainment of the object of
the law according to its true intent, meaning and spirit - the protection and safety of victims of
violence
against
women
and
children.
Thus, contrary to the RTC's pronouncement, the maxim "expressio unios est exclusio
alterius finds no application here. It must be remembered that this maxim is only an ancillary
rule of statutory construction. It is not of universal application. Neither is it conclusive. It
should be applied only as a means of discovering legislative intent which is not otherwise
manifest and should not be permitted to defeat the plainly indicated purpose of the legislature.

32. Sanches V. People G.R. 179090

Facts:Accused-appellant, Leonilo Sanchez, physically abused VVV, a 16 year-old minor, by


hitting her thrice in the upper part of her legs.
The RTC of Tagbilaran City, Boholconvicted accused-appellant of Other Acts of Child Abuse
punishable under R.A. 7610 in relation to P.D. 603.
The accused-appellant argues that the injuries inflicted by him were minor in nature and that it
was not prejudicial to the child-victims development, and that, therefore, P.D. 603 was not
applicable so he should be charged instead under the Revised Penal Code (RPC) for slight
physical injuries.

Issue: Whether or not the accused-appellant was right in his contention that P.D. 603 was not
applicable to the case.

Held:No. In this case, the Court held that the applicable laws are Art. 59 of P.D. 603 and Sec.
10(a) of R.A. 7610.Sec. 10(a) of R.A. 7610 provides that any person who shall commit any
other acts of child abuse, cruelty or exploitation or be responsible for other conditions prejudicial
to the childs development including those covered by Art. 59 of P.D. 603, as amended, but not
covered by the RVP, as amended, shall suffer the penalty of prision mayor in its minimum
period.

Appellant contended that the act should not be considered as child abuse but merely as slight
physical injuries defined and punishable under Article 266 of the RPC. But according to the
Court, appellant must have conveniently forgotten that, when the incident happened, VVV was a
child entitled to the protection extended by R.A. No. 7610.

Contrary to petitioners assertion, an accused can be prosecuted and be convicted under Section
10 (a), Article VI of Republic Act No. 7610 if he commits any of the four acts namely: (a) child
abuse, (b) child cruelty, (c) child exploitation, and (d) being responsible for conditions
prejudicial to the childs development. The prosecution need not prove that the acts of child
abuse, child cruelty, and child exploitation have resulted in the prejudice of the child because an
act prejudicial to the development of the child is different from the former acts.

33. Urbano V. People G.R. 182750

Facts: Petitioner Urbano testified being, in the afternoon of September 28, 1993, in the nearby
town of Bugallon for a picnic. He was with Tomelden and several others, including Dominador
Navarro, Chairperson of Lingayen Water District. At a restaurant in Bugallon, the group ordered
goats meat and drank beer. When it was time to depart, Navarro asked petitioner to inform
Tomelden, then seated in another table, to prepare to leave.
When so informed, Tomelden insulted petitioner, telling the latter he had no business stopping
him from further drinking as he was paying for his share of the bill. Chastised, petitioner
returned to his table to report to Navarro. At that time, petitioner saw that Tomelden had already
consumed 17 bottles of beer. In all, the group stayed at the picnic place for three and a half hours
before returning to the LIWAD. Upon reaching the LIWAD compound, Tomelden allegedly
slapped and hurled insults at him, calling him "sipsip" just to maintain his employment as
Navarros tricycle driver. Tomelden allegedly then delivered several fist and kick blows at
petitioner, a couple of which hit him despite his evasive actions. Petitioner maintained that he

only boxed the victim in retaliation, landing that lucky punch in the course of parrying the
latters blows.

Thereafter, Tomelden went to the hospital several times complaining of dizziness, headache, and
other pains. The last time he went to the hospital, things turned for the worst. Tomelden died due,
per Dr. Arellano, to "cardio-respiratory arrest secondary to cerebral concussion with resultant
cerebral hemorrhage due to mauling incident."
Orje Salazar, their co-worker, attests to the provocative acts of Tomelden and to his being the
aggressor.

Issue: Whether or not the victims insulting remarks directed at the accused, and uttered
immediately before the fist fight constitute sufficient provocation?

Decision: When the law speaks of provocation either as a mitigating circumstance or as an


essential element of self-defense, the reference is to an unjust or improper conduct of the
offended party capable of exciting, inciting, or irritating anyone; it is not enough that the
provocative act be unreasonable or annoying; the provocation must be sufficient to excite one to
commit the wrongful act and should immediately precede the act. This third requisite of selfdefense is present: (1) when no provocation at all was given to the aggressor; (2) when, even if
provocation was given, it was not sufficient; (3) when even if the provocation was sufficient, it
was not given by the person defending himself; or (4) when even if a provocation was given by
the person defending himself, it was not proximate and immediate to the act of aggression.
In the instant case, Tomeldens insulting remarks directed at petitioner and uttered immediately
before the fist fight constituted sufficient provocation. This is not to mention other irritating
statements made by the deceased while they were having beer in Bugallon. Petitioner was the
one provoked and challenged to a fist fight.

34. Sanchez V. People 510 SCRA 365

Facts: Sanchez's account of the facts shows that he and Jamero were tenants of adjacent lots
located in San Jose, Mahayag, Zamboanga del Sur. At about 7:00 o'clock in the morning of
September 4, 1993, Sanchez saw Jamero destroying the dike which served as the boundary
between the two lots. Sanchez confronted Jamero and told the latter that he was encroaching on
his land. Jamero struck him with a shovel. The shovel got stuck in the mud so Jamero resorted to
throwing mud at Sanchez. Fighting back, Sanchez hacked Jamero with a bolo, resulting in the
latter's death. Sanchez then proceeded to the municipal building to surrender upon the advice of
his son-in-law.

According to the OSG, Jamero's attack on Sanchez was unsuccessful because the latter was able
to evade it and Jamero's shovel got stuck in the mud. Jamero fled toward the ricefield when
Sanchez unsheathed his bolo. Sanchez pursued him and struck his head with a bolo. Jamero fell
down but was able to stand up again. He ran away but after a short distance, fell down again.
Sanchez approached him and stabbed him several times. Not satisfied, Sanchez pushed Jamero's
face down into the knee-deep mud. After Jamero's aggression ceased when he fled and left his
shovel stuck in the mud, there was no longer any justification for Sanchez to go after him and
hack him to death.

Issue: Whether or not unlawful aggression, if not continuous, does not constitute aggression
warranting self-defense?

Decision: There can be no self-defense, complete or incomplete, unless the accused proves the
first essential requisiteunlawful aggression on the part of the victim. Unlawful aggression
presupposes an actual, sudden and unexpected or imminent danger on the life and limb of a
person a mere threatening or intimidating attitude is not sufficient. There must be actual
physical force or a threat to inflict physical injury. In case of a threat, it must be offensive and
positively strong so as to display a real, not imagined, intent to cause injury. Aggression, if not
continuous, does not constitute aggression warranting self-defense.

In this case, the twin circumstances of Jamero's shovel getting stuck in the mud and his running
away from Sanchez convincingly indicate that there was no longer any danger to the latter's life

and limb which could have justified his pursuit of Jamero and subsequent hacking and killing of
the latter.

Sanchez's failure to prove unlawful aggression by Jamero and the prosecution's evidence
conclusively showing that it was Sanchez who was the unlawful aggressor completely discounts
Sanchez's claim of self-defense. Even incomplete self-defense by its very nature and essence
would always require the attendance of unlawful aggression initiated by the victim which must
clearly be shown.

35. People V. Beltran, Jr. 503 SCRA 715

Facts: On 25 October 1999, at about 10:00 in the evening, Ever D. Sales left his workplace and
proceeded home using his bicycle. While traversing the Velasquez Road, he saw appellant
holding a bolo and standing in front of his house situated at the side of Velasquez Road. On the
opposite side of the same road, he saw Norman H. Concepcion (Norman) standing in front of an
automobile repair shop. Exhausted by the travel, Ever decided to stop by and rest momentarily at
a nipa hut near the same road. Minutes later, he saw appellant, from a distance of six meters,
stalking Norman who was then walking near the automobile shop. Appellant approached
Norman, and, without a warning, hacked him with a bolo. Norman tried to avoid the blow by
moving backwards and shielding his face with his left arm. However, Norman's left hand was hit
and wounded by the bolo. When Norman turned around and ran, appellant hacked him at the
back causing him to fall down on a grassy area. Appellant repeatedly hacked Norman with a
bolo.

In his defense Appellant admitted that he hacked Norman with a bolo but insisted that he did the
same in self-defense. He narrated that on 25 October 1999, at about 10:00 in the evening, he and
his mother were resting inside their house when suddenly, he heard Norman shouting and
insulting him outside their house and challenging him to a fight. When he came out of the house,
he noticed that Norman was accompanied by several unidentified persons. Thereafter, he tried to
pacify Norman but the latter slapped the back of his head and pulled out an ice pick from his
pocket. He retreated and looked for something to defend himself. He found a bolo near a
tamarind tree in front of their house and took the same. When Norman was about to enter
appellant's house, the latter hacked him with the bolo. Norman tried to avoid the blow but the
same hit his left arm. Appellant lost grip of the bolo and the same fell on the ground. While
appellant was reaching for the bolo, Norman grabbed his head and tried to stab him with the ice-

pick. Appellant, however, eluded the counter-attack but he sustained a minor wound on the
forehead. Upon gaining control of the scuffle, appellant took the bolo and hacked Norman four
consecutive times, most of them landed on the head. When appellant noticed that Norman was no
longer moving, he fled therein and went to his brother, Sherman Beltran, in Bauan, Batangas,
where he stayed that same night and hid therein the bolo. The next day, he went to his sister's
house in Lipa City. Later that day, he went to the Granja Hospital, also in Lipa City, for treatment
of his wound on the forehead.

Issue: Whether or not Beltran is entitled to the justifying circumstance under Art. 11 of the RPC.
Held: No. Article 11, paragraph (1), of the Revised Penal Code provides for the elements and/or
requisites in order that a plea of self-defense may be validly considered in absolving a person
from criminal liability, viz:
ART. 11. Justifying circumstances. The following do not incur any criminal liability:
Anyone who acts in defense of his person or rights, provided that the following circumstances
concur;
First. Unlawful aggression;
Second. Reasonable necessity of the means employed to prevent or repel it;
Third. Lack of sufficient provocation on the part of the person defending himself.
As an element of self-defense, unlawful aggression refers to an assault or attack, or a threat
thereof in an imminent and immediate manner, which places the defendant's life in actual peril. It
is an act positively strong showing the intent of the aggressor and not merely a threatening or
intimidating attitude. It is also described as a sudden and unprovoked attack of immediate and
imminent kind to the life, safety or rights of the person attacked.
There is an unlawful aggression on the part of the victim when he puts in actual or imminent
peril the life, limb, or right of the person invoking self-defense. There must be actual physical
force or actual use of weapon. In order to constitute unlawful aggression, the person attacked
must be confronted by a real threat on his life and limb; and the peril sought to be avoided is
imminent and actual, not merely imaginary.
In the instant case, there was no unlawful aggression on the part of Norman that justified the act
of appellant in hacking him to death. There was no actual or imminent danger on the life of
appellant when he came face to face with Norman. As narrated by Ever, Norman was just
walking on the road and was not provoking appellant into a fight. It was the appellant who
approached and suddenly hacked Norman repeatedly even when the latter was already fallen on
the ground. In short, appellant was the unlawful aggressor.

Even if this Court were to adopt the version of facts of appellant, the result or conclusion would
be the same.
Appellant alleged that he was resting inside his house when he heard Norman shouting
invectives against him and challenging him to a fight. When he went outside the house to pacify
Norman, the latter slapped the back of his head and brought out an ice-pick. Appellant retreated
and when Norman tried to follow him inside the house, he took a bolo and repeatedly hacked
Norman. The foregoing circumstances do not justify the act of appellant in hacking Norman.
Obviously, mere shouting of invectives and challenging one to a fight does not put one's life in
actual or imminent danger. In the same vein, mere slapping of one's head does not place a
person's life in serious danger such that it compels him to use a bolo and hack the offender.
As regards the brandishing of an ice-pick, appellant had several less harmful means of avoiding
the same as he was not cornered or trapped. He could have run inside his house and locked the
door, or, called the neighbors or authorities for help. Unfortunately, appellant did not avail
himself of any of those options and instead chose to hack Norman. Quite conspicuously, no
convincing evidence was presented to show that Norman was, indeed, armed with an ice-pick at
the time of the incident. In fact, no ice-pick was found in the crime scene nor in the body of
Norman. There was also no proof adduced showing that Norman attempted to stab appellant or
tried to barge into the latter's house.

36. People V. Dagani 499 SCRA 64


Facts: At about 4:45 in the afternoon of September 11, 1989, a group composed of Ernesto Javier
(Javier), Lincoln Miran (Miran), and two other individuals had been drinking at the canteen
located inside the compound of the Philippine National Railways (PNR) along C.M. Recto
Avenue, Tondo, Manila. All of a sudden, appellants, who were security officers of the PNR and
covered by the Civil Service Rules and Regulations, entered the canteen and approached the
group. Appellant Dagani shoved Miran, causing the latter to fall from his chair. Dagani then held
Javier while Santiano shot Javier twice at his left side, killing the latter.
The defense proceeded to prove their version of the facts:
Appellants testified that they were ordered by their desk officer to investigate a commotion at the
canteen. Upon reaching the place, Santiano ordered his co-accused, Dagani, to enter, while the
former waited outside.
Dagani approached Javier who had been striking a bottle of beer on the table. Javier then pulled
out a .22 caliber revolver and attempted to fire at Dagani, but the gun failed to go off. Then
suddenly, while outside the canteen, Santiano heard gunfire and, from his vantage point, he saw

Javier and Dagani grappling for a .22 caliber gun which belonged to Javier. During the course of
the struggle, the gun went off, forcing Santiano to fire a warning shot. He heard Javiers gun fire
again, so he decided to rush into the canteen. Santiano then shot Javier from a distance of less
than four meters.
Appellants invoked the justifying circumstances of self-defense and lawful performance of
official duty as PNR security officers. They also argued that the prosecution failed to establish
treachery and conspiracy.

The lower court ruled in favor of the victim, hence, the accused appealed.

Issue: Whether or not the lower court made a mistake of not appreciating self defense on the part
of the accused.
Held: When self-defense is invoked, the burden of evidence shifts to the accused to show that the
killing was legally justified. Having owned the killing of the victim, the accused should be able
to prove to the satisfaction of the Court the elements of self-defense in order to avail of this
extenuating circumstance. He must discharge this burden by clear and convincing evidence.
When successful, an otherwise felonious deed would be excused, mainly predicated on the lack
of criminal intent of the accused. Self-defense requires that there be (1) an unlawful aggression
by the person injured or killed by the offender, (2) reasonable necessity of the means employed
to prevent or repel that unlawful aggression, and (3) lack of sufficient provocation on the part of
the person defending himself. All these conditions must concur.
Unlawful aggression, a primordial element of self-defense, would presuppose an actual, sudden
and unexpected attack or imminent danger on the life and limb of a person not a mere
threatening or intimidating attitude but most importantly, at the time the defensive action was
taken against the aggressor. To invoke self-defense successfully, there must have been an
unlawful and unprovoked attack that endangered the life of the accused, who was then forced to
inflict severe wounds upon the assailant by employing reasonable means to resist the attack.13
In the instant case, the assertions that it was "quite probable" that Javier, during the course of the
struggle for the firearm, "could have easily killed" the appellants are uncertain and speculative.
There is aggression in contemplation of the law only when the one attacked faces real and
immediate threat to ones life. The peril sought to be avoided must be imminent and actual, not
just speculative.
The second element of self-defense demands that the means employed to neutralize the unlawful
aggression are reasonable and necessary. It is settled that reasonable necessity of the means
employed does not imply material commensurability between the means of attack and defense.

What the law requires is rational equivalence. The circumstances in their entirety which surround
the grappling of the firearm by Dagani and Javier, such as the nature and number of gunshot
wounds sustained by the victim which amounted to two fatal wounds, that Dagani was able to
restrain the hands of Javier and push them away from his body, that Dagani was larger than
Javier and had finished Special Weapons and Tactics (SWAT) hand-to-hand combat training, and
Javier, as admitted by the appellants, was inebriated at the time of the incident, do not justify
appellant Santianos act of fatally shooting the victim twice.
All things considered, the appellants plea of self-defense is not corroborated by competent
evidence. The plea of self-defense cannot be justifiably entertained where it is not only
uncorroborated by any separate competent evidence but is in itself extremely doubtful. Whether
the accused acted in self-defense is a question of fact. Like alibi, the affirmative defense of selfdefense is inherently weak because, as experience has demonstrated, it is easy to fabricate and
difficult to disprove. This Court, therefore, finds no reversible error on the part of the courts a
quo in rejecting the claim of self-defense.

37. Toledo V. People 439 SCRA 94


Facts: On September 16, 1995, appellant went to a black-smith who made the design of his bolo.
When he went home to Tuburan, Odiongan, Romblon late in the afternoon, appellant saw the
group of Lani Famero, Michael Fosana, Rex Cortez and Ricky Guarte drinking gin at the house
of the Spouses Manuel and Eliza Guarte, Rickys parents. Appellants house is about five (5)
meters away from the house of Spouses Guarte. Appellant requested the group of Ricky to
refrain from making any noise. Thereupon, appellant proceeded inside his house and went to
sleep. Around 9:00 p.m., Gerardo Faminia, Eliza Guartes brother arrived at the Guarte house
and asked for any left-over food. Eliza prepared dinner for him and after Gerardo finished eating,
he went home accompanied by Ricky. Gerardos home is about twelve (12) meters away from
the Guarte home. Minutes later, Ricky came back and together with Lani, Rex and Michael, went
to sleep at the Guarte house. They had not laid down for long when they heard stones being
hurled at the roof of the house. The stoning was made three (3) times. Ricky rose from bed and
peeped through a window. He saw appellant stoning their house. Ricky went out of the house and
proceeded to appellants house. Ricky asked appellant, his uncle, why he was stoning their
house. Appellant did not answer but met Ricky at the doorstep of his (appellants) house and,
without any warning, stabbed Ricky on the abdomen with a bolo. Eliza had followed his son
Ricky and upon seeing that Ricky was stabbed, shouted for help. Lani heard Elizas cry for help
and immediately rushed outside the house. Lani saw Ricky leaning on the ground and supporting
his body with his hands. Lani helped Ricky stand up and brought him to the main road. Lani
asked Ricky who stabbed him and Ricky replied that it was appellant who stabbed him. Then
Docloy Cortez arrived at the scene on board his tricycle. Accordingly, Ricky was put on the
tricycle and taken to the Romblon Provincial Hospital where he died.

The petitioner adduced evidence that at around 5:00 p.m. on September 16, 1995, he was on his
way home at Tuburan, Odiongan, Romblon. He saw his nephew, Ricky Guarte, and the latters
friends, Michael Fosana, Rex Cortez, and Lani Famero, about five meters away from his house,
having a drinking spree. He ordered them not to make loud noises, and they obliged. He then
went to his house, locked the door with a nail, and went to sleep. However, he was awakened at
around 9:30 p.m. by loud noises coming from Ricky and his three companions. He peeped
through the window grills of his house and admonished them not to make any loud noises. Ricky,
who was then already inebriated, was incensed; he pulled out a balisong, pushed the door, and
threatened to stab the petitioner. The petitioner pushed their sala set against the door to block the
entry of Ricky, but the latter continued to push the door open with his hands and body. The
petitioner ran to the upper portion of their house and got his bolo. He returned to the door and
pushed it with all his might using his left hand. He then pointed his bolo, which was in his right
hand, towards Ricky. The bolo accidentally hit Ricky on the stomach, and the latter lost his
balance and fell to the floor. The petitioner, thereafter, surrendered to the barangay captain at
11:00 a.m. on September 17, 1995.

After trial, the court rendered judgment finding the petitioner guilty as charged. The trial court
did not give credence and probative weight to the testimony of the petitioner that his bolo
accidentally hit the victim on the stomach.

The petitioner testified that his bolo hit the victim accidentally. He asserted in the RTC and in the
CA that he is exempt from criminal liability for the death of the victim under Article 12,
paragraph 4 of the Revised Penal Code.

However, the petitioner changed gear, so to speak, and now alleges that he acted in self-defense
when he stabbed the victim. As such, he contends, he is not criminally liable under Article 11,
paragraph 1 of the Revised Penal Code

Issue: Whether or not the accused is entitled to a complete self defense when he accidentally
stabbed the victim in the stomach.

Held: It is an aberration for the petitioner to invoke the two defenses at the same time because
the said defenses are intrinsically antithetical. There is no such defense as accidental self-defense
in the realm of criminal law.
To prove self-defense, the petitioner was burdened to prove the essential elements thereof,
namely: (1) unlawful aggression on the part of the victim; (2) lack of sufficient provocation on
the part of the petitioner; (3) employment by him of reasonable means to prevent or repel the
aggression. Unlawful aggression is a condition sine qua non for the justifying circumstances of
self-defense, whether complete or incomplete. Unlawful aggression presupposes an actual,
sudden, and unexpected attack, or imminent danger thereof, and not merely a threatening or
intimidating attitude. We agree with the ruling of the CA that the petitioner failed to prove selfdefense, whether complete or incomplete.
Appellant was not justified in stabbing Ricky. There was no imminent threat to appellants life
necessitating his assault on Ricky. Unlawful aggression is a condition sine qua non for the
justifying circumstance of self-defense. For unlawful aggression to be appreciated, there must be
an actual, sudden, unexpected attack or imminent danger thereof, not merely a threatening or
intimidating attitude. In the absence of such element, appellants claim of self-defense must fail.

38. People V. Cajurao 420 SCRA 207


Facts: The appellant Joseph Cajurao and his friend Allan Daosos went to the dance hall. Felix
Teruel and Nena Carmelo were then manning the gate and the ticket booth. Since Cajurao and
Daosos had no tickets, they were not allowed to enter. The appellant and Daosos tarried within
the vicinity and repeatedly tried to enter the hall without tickets, to no avail.
At about 10:30 p.m., Pordios was surprised when Santiago Betita suddenly arrived at her stall.
When he took the "lamp" that illuminated her wares, she got angry and confronted him. Pordios
asked him why he took the lamp, but Betita ignored her and held on to the lamp in a defensive
stance. Betita appeared perturbed. Shortly thereafter, someone threw a stone, prompting people
to scamper away. Still holding on to the lamp, Betita moved over to the stall beside Pordios.
Suddenly, the appellant sped towards Betita and stabbed him on the right nipple. Betita fell to the
ground. The appellant then threw his knife away and fled.
Domingo Tecson, a civilian volunteer assigned to take charge of the peace and order situation in
the area, was then on patrol. He saw the appellant pass by, running. He looked towards the
direction where the appellant had come from and saw Betita slumped on the ground, mortally
wounded. Tecson rushed to where Betita was and shouted for help. He instructed his fellow
volunteers to run after the appellant and collar him. After a brief chase, the appellant was caught
by a volunteer in the carnival ground and was thereafter turned over to the police authorities.

Tecson went back to the crime scene to look for the weapon the appellant used to kill Betita.
With the aid of the light from a nearby fire truck, he found the knife and its scabbard. Tecson
turned the weapon over to the police. Pordios and Tecson gave their respective statements to PO3
Lino D. Antonino.
After the incident the accused was caught and trial ensued.
The trial court rejected the appellants defense and concluded that he failed to present clear and
convincing evidence to prove that he killed Betita in self-defense. It gave credence and full
probative weight to the testimony of the witnesses for the prosecution, that the appellant stabbed
the defenseless victim. It also appreciated the presence of treachery, qualifying the crime to
murder.
Issue: Whether Cajurao is entitled to self-defense and therefore should be acquitted of the
offense charged.
Held: No. There can be no self-defense, complete or incomplete, unless there is clear and
convincing proof of unlawful aggression on the part of the victim. The unlawful aggression, a
constitutive element of self-defense, must be real or at least imminent and not merely imaginary.
A belief that a person is about to be attacked is not sufficient. Even an intimidating or threatening
attitude is by no means enough. Unlawful aggression presupposes an actual or imminent danger
on the life or limb of a person. Mere shouting, an intimidating or threatening attitude of the
victim does not constitute unlawful aggression. Unlawful aggression refers to an attack that has
actually broken out or materialized or at the very least is clearly imminent; it cannot consist in
oral threats or merely a threatening stance or posture. The settled rule in jurisprudence is that
when unlawful aggression ceases, the defender no longer has the right to kill or even wound the
former aggressor. Retaliation is not a justifying circumstance. Upon the cessation of the unlawful
aggression and the danger or risk to life and limb, the necessity for the person invoking selfdefense to attack his adversary ceases. If he persists in attacking his adversary, he can no longer
invoke the justifying circumstance of self-defense. Self-defense does not justify the unnecessary
killing of an aggressor who is retreating from the fray.
In this case, Pordios testified that the appellant stabbed Betita even as the latter moved over to
the next stall, still holding the lamp with the lighted wick which he took from her stall to defend
himself from the appellant. Betita had anticipated that the appellant would assault him. Betitas
fears proved to be well-founded, as the appellant rushed to where he was and stabbed him on the
right nipple. Pordios did not testify that before the stabbing, Betita was about to throw the bottle
at the appellant.
39. People V. Antonio 383 SCRA 751
Facts: The facts as presented by the prosecution show that at 1:00 in the early morning of
October 11, 1998, the victim, Jomar Ephan, was engaged in a drinking session with Reynaldo

Ephan and Roselito Dacillo in front of a store in Barangay Pakna-an, Mandaue City. Accusedappellant arrived and bought cigarettes. Then, he ordered Jomar, Reynaldo and Roselito to count
the cigarettes he bought, but the three told accused-appellant to let the storekeeper do the
counting. Rebuked, accused-appellant left the store. He returned minutes later and suddenly
stabbed the victim at the back, after which he immediately fled. The victim was rushed by his
companions to the hospital but died the following day.
Meanwhile, Eduardo Juban, a Barangay Tanod, was awakened by one of his neighbors and was
told that there was trouble at a nearby store. When Eduardo went out, he saw accused-appellant
being chased by a crowd who were shouting, "thief." The group mauled accused-appellant when
they caught up with him. Eduardo, however, pacified the mob and brought accused-appellant to
the barangay hall. Eduardo later learned from the group that accused-appellant had stabbed
somebody.
On the other hand, the defense tried to prove that: at around 1:00 a.m. of October 11, 1998,
accused-appellant was in the house of his friend, Fernando Gelig, at Pakna-an, Mandaue City.
While they were drinking liquor, accused-appellant went out and bought cigarettes from a store
across the street. As a token of friendship, accused-appellant offered the cigarettes to the people
in front of the store, but nobody accepted his offer. Accused-appellant went back to the house of
his friend. After a short while, he went back to the same store to buy "pulutan." For no reason at
all, somebody struck him with a stool hitting him on the left eyebrow. Accused-appellant fell on
the ground but the group of the deceased, who were then in front of the store, ganged up on him.
The deceased attempted to hit accused-appellant but because the former was very drunk, he
missed and fell on his belly. It was at this point when accused-appellant got hold of a knife he
saw under the table and stabbed the deceased at the back. Thereafter, accused-appellant
immediately fled but the crowd chased and mauled him. Fortunately, a Barangay Tanod came
and stopped the mob.

The RTC ruled in favor of the victim and found the accused guilty of murder. Hence, he appealed
the said decision.
Issue: Whether or not the lower court failed to appreciate the justifying circumstance of selfdefense in favor of the accused.
Held: In the case at bar, even if we sustain the version of accused-appellant that the initial act of
aggression came from the group of the deceased, still we cannot uphold his plea of self-defense.
As testified by accused-appellant himself, the deceased who was at that time very drunk tried to
hit him but missed and fell on the ground. At that point, unlawful aggression ceased and it was
no longer necessary for him to stab the deceased. It was accused-appellant, therefore, who
became the aggressor when he, despite the condition of the deceased, proceeded to stab the latter
at the back. His act can no longer be interpreted as an act of self-preservation but a perverse

desire to kill. Hence, he cannot successfully claim the benefit of self-defense. Furthermore, if it
were true that the companions of the deceased ganged up on him, his attack should have been
directed against them and not against the deceased who was already defenseless and lying on the
ground.
40. People V. Ubaldo 367 SCRA 432
Facts: Norberto Ventura who was drunk and violently disrupting the wedding festivities of his
brother Reynaldos, son was shot by Teodoro Ubaldo a barangay captain called by Reynaldo to
pacify Norberto. Norberto was shot 3 times which instantly caused his death.
After the incident Teodoro fled the scene of the crime and later claim self defense as the reason
for shooting Norberto.
Issue: Whether or not the lower court failed to appreciate self defense in the part of Teodoro.
Held: Basilia's testimony on the lack of unlawful aggression on the part of the victim is
corroborated by the findings of the medico-legal expert who testified that the victim was first
shot at the lateral side of the neck about two feet from his assailant. We find the location of the
first gunshot wound significant since it establishes the relative positions of appellant and victim.
The autopsy findings show that appellant was not in front of the victim when the first shot was
fired, but was behind him and towards the latter's side. These belie appellant's claim that he was
face to face with the victim and grappling for possession of the gun when the victim was hit. The
physical evidence in this case is a mute but eloquent manifestation of truth, which ranks high in
the hierarchy of trustworthy evidence.
On the question of whether the means employed by appellant were reasonable, the number of
gunshot wounds inflicted on the deceased shows that the means employed were hardly
reasonable at all. Assuming arguendo that the victim fired first at appellant, a single shot could
have already disabled the former who was inebriated. The nature and number of wounds inflicted
upon the victim are important indicia which disprove a plea of self-defense. The gruesome,
multiple gunshot wounds inflicted upon the deceased show that appellant's act was not one of
self-defense, but was a determined and purposeful attack upon the victim.

41. People V. Narvaez 121 SCRA 389

Facts: Mamerto Narvaez has been convicted of murder (qualified by treachery) of David
Fleischer and Flaviano Rubia. On August 22, 1968, Narvaez shot Fleischer and Rubia during the
time the two were constructing a fence that would prevent Narvaez from getting into his house
and rice mill. The defendant was taking a nap when he heard sounds of construction and found

fence being made. He addressed the group and asked them to stop destroying his house and
asking if they could talk things over. Fleischer responded with "No, gadamit, proceed, go ahead."
Defendant lost his "equilibrium," and shot Fleisher with his shotgun. He also shot Rubia who
was running towards the jeep where the deceased's gun was placed. Prior to the
shooting, Fleischer and Co. (the company of Fleischer's family) was involved in a legal battle
with the defendant and other land settlers of Cotabato over certain pieces of property. At the time
of the shooting, the civil case was still pending for annulment (settlers wanted granting of
property to Fleisher and Co. to be annulled). At time of the shooting, defendant had leased his
property from Fleisher (though case pending and ownership uncertain) to avoid trouble. On June
25, defendant received letter terminating contract because he allegedly didn't pay rent.
He was given 6 months to remove his house from the land. Shooting was barely 2 months after
letter. Defendant claims he killed in defense of his person and property. CFI ruled that
Narvaez was guilty. Aggravating circumstances of evident premeditation offset by the mitigating
circumstance of voluntary surrender. For both murders, CFI sentenced him to reclusion perpetua,
to
indemnify
the
heirs,
and
to
pay
for
moral
damages.

he

Issues: 1. Whether or not CFI erred in convicting defendant-appellant despite the fact that
acted
in
defense
of
his
person.

Held: No. The courts concurred that the fencing and chiselling of the walls of the house
of the defendant was indeed a form of aggression on the part of the victim. However, this
aggression was not done on the person of the victim but rather on his rights to property. On the
first issue, the courts did not err. However, in consideration of the violation of property rights,
the courts referred to Art. 30 of the civil code recognizing the right of owners to close and fence
their
land.
Although is not in dispute, the victim was not in the position to subscribe to the article
because his ownership of the land being awarded by the government was still pending, therefore
putting ownership into question. It is accepted that the victim was the original aggressor.
2. WON the court erred in convicting defendant-appellant although he acted in defence of his
rights.
Held: Yes. However, the argument of the justifying circumstance of self-defense is
applicable only if the 3 requirements are fulfilled. Art. 11(1) RPC enumerates these requisites:

Unlawful aggression. In the case at bar, there was unlawful aggression towards appellant's
property rights. Fleisher had given Narvaez 6 months and he should have left him in peace
before time was up, instead of chiseling Narvaez's house and putting up fence. Art. 536 of the

Civil Code also provides that possession may not be acquired through force or intimidation;
while Art. 539 provides that every possessor has the right to be respected in his possession

Reasonable necessity of means employed to prevent or repel attack. In the case, killing was
disproportionate to the attack.

Lack of sufficient provocation on part of person defending himself. Here, there was no
provocation at all since he was asleep.

Since not all requisites present, defendant is credited with the special mitigating circumstanceof
incomplete defense, pursuant to Art. 13(6) RPC. These mitigating circumstances are: voluntary
surrender and passion and obfuscation (read p. 405 explanation) Crime is homicide (2 counts)
not murder because treachery is not applicable on account of provocation by the deceased. Also,
assault was not deliberately chosen with view to kill since slayer acted instantaneously. There
was also no direct evidence of planning or preparation to kill. Art. 249 RPC: Penalty for
homicide is reclusion temporal. However, due to mitigating circumstances and incomplete
defense,
it
can
be
lowered
three
degrees
(Art.
64) to
arrestomayor.

42. People V. Genosa 419 SCRA 537

Facts: That Marivic Genosa, the Appellant on the 15 November1995, attacked and wounded his
husband, which ultimately led to his death. According to the appellant she did not provoke her
husband when she got home that night it was her husband who began the provocation. The
Appellant said she was frightened that her husband would hurt her and she wanted to make sure
she would deliver her baby safely. In fact, The Appelant had to be admitted later at the Rizal
Medical Centre as she was suffering from eclampsia and hypertension, and the baby was born
prematurely
on
December
1,
1995.
The Appellant testified that
least five (5) times, but that Ben
Apellant said that the reason why
because
'he
was
crazy

during her marriage she had tried to leave her husband at


would always follow her and they would reconcile. The
Ben was violent and abusive towards her that night was
about
his
recent
girlfriend,
Lulu
Rubillos.

The Appellant after being interviewed by specialists, has been shown to be suffering from

Battered

Woman

Syndrome.

The appellant with a plea of self defense admitted the killing of her husband, she was then found
guilty of Parricide, with the aggravating circumstance of treachery, for the husband was attacked
while
asleep.

can

Issues: Can Marivic Genosa be granted the Justifying circumstance of Self-defense, and
she be held liable for the aggravating circumstance of treachery?

No, Since self-defense since the existence of Battered woman syndrome, which the
appellant has been shown to be suffering in the relationship does not in itself establish the legal
right of the woman to kill her abusive partner. Evidence must still be considered in the context of
self-defense.
In the present case, however, according to the testimony of the appellant there was a sufficient
time interval between the unlawful aggression of the husband and her fatal attack upon him. She
had already been able to withdraw from his violent behavior and escape to their children's
bedroom. During that time, he apparently ceased his attack and went to bed. The reality or even
the imminence of the danger he posed had ended altogether. He was no longer in a position that
presented
an
actual
threat
on
her
life
or
safety.

does

Without continuous aggression there can be no self-defense. And absence of aggression


not
warrant
complete
or
incomplete
self-defense.

No, There is treachery when one commits any of the crimes against persons by
employing means, methods or forms in the execution thereof without risk to oneself arising from
the
defense
that
the
offended
party
might
make.
The circumstances must be shown as indubitably as the killing itself; they cannot be
deduced from mere inferences, or conjectures, which have no place in the appreciation of
evidence. Besides, equally axiomatic is the rule that when a killing is preceded by an argument
or a quarrel, treachery cannot be appreciated as a qualifying circumstance, because the deceased
may be said to have been forewarned and to have anticipated aggression from the assailant.
In the present case, however it was not conclusively shown, that the appellant
intentionally chose a specific means of successfully attacking her husband without any risk to
herself from any retaliatory act that he might make. To the contrary, it appears that the thought of
using the gun occurred to her only at about the same moment when she decided to kill her
spouse. In the absence of any convincing proof that she consciously and deliberately employed

the method by which she committed the crime in order to ensure its execution, the doubt should
be
resolved
in
her
favor.
The conviction of Appellant Marivic Genosa for parricide is hereby AFFIRMED. However, there
being two (2) mitigating circumstances and no aggravating circumstance attending her
commission of the offense, her penalty is REDUCED to six (6) years and one (1) day of prision
mayor as minimum; to 14 years, 8 months and 1 day of reclusion temporal as maximum.

43. Balunueco V. People 401 SCRA 76


Facts: As principal witness for the prosecution, Amelia Iguico narrated that on 2 May 1982 at
around 6:00 oclock in the evening she was coddling her youngest child in front of her house at
Bagong Tanyag, Taguig, when she saw accused Reynaldo, his father Juanito and brothers
Ricardo and Ramon, all surnamed Balunueco, and one Armando Flores chasing her brother-inlaw Servando Iguico. With the five (5) individuals in hot pursuit, Servando scampered into the
safety of Amelias house.
Meanwhile, according to private complainant Amelia, her husband Senando, who was then
cooking supper, went out of the house fully unaware of the commotion going on outside. Upon
seeing Senando, Reynaldo turned his attention on him and gave chase. Senando instinctively fled
towards the fields but he was met by Armando who hit him with a stone, causing Senando to feel
dizzy. Reynaldo, Ricardo, and Armando cornered their quarry near a canal and ganged up on
him. Armando placed a can on top of Senandos head and Ricardo repeatedly struck Senando
with an ax on the head, shoulder, and hand. At one point, Ricardo lost his hold on the ax, but
somebody tossed him a bolo and then he continued hacking the victim who fell on his knees. To
shield him from further violence, Amelia put her arms around her husband but it was not enough
to detract Ricardo from his murderous frenzy. Amelia was also hit on the leg.
The trial court disbelieved the version of accused Ricardo, thus he was found guilty of homicide
in Crim. Case No. 49576 and frustrated homicide in Crim. Case No. 49577. It reasoned that the
testimony of Amelia Iguico was clear, positive, straightforward, truthful and convincing. On the
other hand, according to the trial court, the denial of Ricardo was self-serving and calculated to
extricate himself from the predicament he was in. Further, the trial court added that the wounds
allegedly received by Ricardo in the hands of the victim, Senando Iguico, if at all there were any,
did not prove that Senando was the aggressor for the wounds were inflicted while Senando was
in the act of defending himself from the aggression of Ricardo and his co-conspirators.
Issue: Whether or not the lower court failed to appreciate the defense of relatives in favor of the
appellant.

Held: No. In effect, petitioner invokes the justifying circumstance of defense of relatives under
Art. 11, par. (2), of The Revised Penal Code. The essential elements of this justifying
circumstance are the following: (a) unlawful aggression; (b) reasonable necessity of the means
employed to prevent or repel it; and, (c) in case the provocation was given by the person
attacked, the one making the defense had no part therein.
Of the three (3) requisites of defense of relatives, unlawful aggression is a condition sine qua
non, for without it any defense is not possible or justified. In order to consider that an unlawful
aggression was actually committed, it is necessary that an attack or material aggression, an
offensive act positively determining the intent of the aggressor to cause an injury shall have been
made; a mere threatening or intimidating attitude is not sufficient to justify the commission of an
act which is punishable per se, and allow a claim of exemption from liability on the ground that
it was committed in self-defense or defense of a relative. It has always been so recognized in the
decisions of the courts, in accordance with the provisions of the Penal Code.
Decision of the CA is affirmed with regards to the homicide of Senando. The charge of attempted
homicide against Amelia Iguico is downgraded to slight physical injury for lack of evidence.

Intestate Estate of Gonzales Vda. De Carungcong V. People G.R. 181409


Facts: (a) Sato presented a document to Manolita (who was already blind at that time) and
induced her to sign and thumbmark the same;
(b) he made Manolita believe that the said document was in connection with her taxes when it
was in fact a special power of attorney (SPA) authorizing his minor daughter Wendy to sell,
assign, transfer or otherwise dispose of Manolitas properties in Tagaytay City;
(c) relying on Satos inducement and representation, Manolita signed and thumbmarked the SPA
in favor of Wendy Mitsuko Sato, daughter of Sato;
(d) using the document, he sold the properties to third parties but he neither delivered the
proceeds to Manolita nor accounted for the same and
(d) despite repeated demands, he failed and refused to deliver the proceeds, to the damage and
prejudice of the estate of Manolita.
Issues: (1) the effect of death on the relationship by affinity created between a surviving spouse
and the blood relatives of the deceased spouse and (2) the extent of the coverage of Article 332.
Held: The resolution of this case rests on the interpretation of Article 332 of the Revised Penal
Code. In particular, it calls for the determination of the following: (1) the effect of death on the
relationship by affinity created between a surviving spouse and the blood relatives of the
deceased spouse and (2) the extent of the coverage of Article 332.

The second view (the continuing affinity view) maintains that relationship by affinity between
the surviving spouse and the kindred of the deceased spouse continues even after the death of the
deceased spouse, regardless of whether the marriage produced children or not.Under this view,
the relationship by affinity endures even after the dissolution of the marriage that produced it as a
result of the death of one of the parties to the said marriage. This view considers that, where
statutes have indicated an intent to benefit step-relatives or in-laws, the "tie of affinity" between
these people and their relatives-by-marriage is not to be regarded as terminated upon the death of
one of the married parties.
Thus, for purposes of Article 332(1) of the Revised Penal Code, we hold that the relationship by
affinity created between the surviving spouse and the blood relatives of the deceased spouse
survives the death of either party to the marriage which created the affinity. (The same principle
applies to the justifying circumstance of defense of ones relatives under Article 11[2] of the
Revised Penal Code, the mitigating circumstance of immediate vindication of grave offense
committed against ones relatives under Article 13[5] of the same Code and the absolutory cause
of relationship in favor of accessories under Article 20 also of the same Code.)
However, the coverage of Article 332 of the RPC is strictly limited to the simple crimes of theft,
swindling (estafa) and malicious mischief. It does not apply when any of these crimes is
complexed with another with another crime as estafa through falsification. In this case, the
Information charges William not with simple estafa but with the complex crime of estafa through
falsification of public documents. The action provided under said article concerns the private
relations of the parties as family members and is limited to the civil aspect between the offender
and the offended party. When estafa is committed through falsification of public document,
however, the matter acquires a very serious public dimension and goes beyond the respective
rights and liabilities of family members among themselves. Hence William is removed from the
protective mantle of Article 332 exempting him from criminal liability
44. INTESTATE ESTATE OF GONZALES VDA CARUNGCONG) PAGE 3
45. People V. Eduarte 187 SCRA 291

Facts: The evidence of prosecution establishes that Fredeswindo Eduarte, suspecting that his wife
was having an illicit affair with another man, got a scissor. Upon seeing his intention, Roberto
(Fredeswindos brother -in-law) tried to pacify him. However, instead of being appeased,
Fredeswindo thrusted the scissor to Roberto. Thereafter, Roberto drove his jeep to seek the help
of policemen. On the way back, he saw Fredeswindo lying flat on the road so he stopped and
alighted to help. Just when Roberto was in the act of extending assistance,

Florentino Eduarte, Fredeswindos brother, shot him, which caused his death.Not long after, the
police authorities arrived and the place and looked for Florentino, but failed to locate him.

On the other hand, the defense, in its counter-statement of facts, relates that when Fredeswindo
confronted his wife, Roberto butted in and berated him which resulted to an exchange of words
leading to the altercation. Fredeswindo defended himself by grasping a scissor from his back and
thrusting it against his assailant. When Robertos wife saw this, she sought the help of Robertos
three brothers and they ganged up on Fredeswindo. Julie Eduarte, Fredewindos brother, saw this
and thereafter sought the help of their other brother, Florentino Eduarte. When Florentino arrived
at the place of incident, he saw Roberto clubbing his brother who was lying, face downward and
his shirt soaked with blood.

The accused-appellant maintains that he is innocent of the crime as charged invoking the
justifying circumstance of defense of relatives. Instead of making an assignment of errors, the
accused-appellant states that the trial court was confronted with two conflicting versions, one
asserting that Roberto Trinidad was shot while assisting Fredeswindo Eduarte and the other
stating that Roberto was shot while clubbing Fredeswindo. In both cases, the appellant states that
defense of relative should be appreciated.

Issue: Whether the defense of relatives under Article II, subparagraph (2) of the Revised Penal
Code can be invoked by the accused-appellant in his favour.

Held: Article II, subparagraph 2 of the Revised Penal Code provides that:Anyone who acts in
defense of the person or rights of his spouse, ascendants, descendants, or legitimate, natural or
adopted brothers or sisters, or of his relatives by affinity in the same degrees and those by
consanguinity within the fourth civil degree, provided that the first and second requisites
prescribed in the next preceding circumstances a represent, and the further requisite, in case the
provocation was given by the person attacked, that the one making defense had no part therein.

The first and second requisites are unlawful aggression and reasonable necessity of the means
employed to prevent or repel the unlawful aggression. Hence, it was incumbent upon the
accused-appellant to prove the existence of the three essential requisites of the justifying
circumstance of defense of relatives namely: (1) unlawful aggression; (2) reasonable necessity of

the means employed to prevent or repel it; and (3) in case the provocation was given by the
person attacked, that the one making the defense had no part therein.

The accused-appellant claims that there was unlawful aggression on the part of RobertoTrinidad
considering that the scenario he saw after he was summoned from his place of work by his other
brother Julie was that of his allegedly assaulted brother Fredeswindo sprawled on the ground and
bathed in his own blood with Roberto Trinidad clubbing Fredeswindo. It was when Roberto was
about to deliver the final blow to Fredeswindo that the accused-appellant claimed he shot
Roberto in defense of his brother. Thus, according to the accused-appellant, the second requisite
is also present considering that the use of a gun at that pressing moment was reasonable and
necessary to preventor repel the aforestated unlawful aggression. As regards the third requisite,
the accused-appellant contends that there is no debate as to its presence since the accusedappellant was clearly not a part of the melee in question.

The SC found no merit in the claim that the shooting of Roberto was done in defense of a
relative. For this justifying circumstance to prosper, the evidence adduced must be persuasive.
Although it is true that the accused-appellant took no part in the provocation that led to the
killing incident, his testimony that there was unlawful aggression on the part of Roberto was selfserving and uncorroborated. Hence, for lack of a clear unlawfulaggression on the part of the
victim Roberto and of the reasonable necessity of themeans employed by the accused-appellant,
the justifying circumstance of defense of relative cannot be availed of. Accused-appellant
Florentino Eduarte is found guiltybeyond reasonable doubt of the crime of HOMICIDE without
any aggravating or mitigating circumstance.
46. People

Dijan

383

SCRA

15

Facts: Based on the story of the defense. On the evening of 11 April 1998, about ten o'clock,
Roderick Silvestre and Alvaro Hilario were at a store located around the corner of Paraiso and
Sumulong Streets in Parang, Marikina City, to buy some cigarettes when they saw the group of
Crispulo Dijan, Romualdo Paglinawan and Oliver Lizardo, passing by the store. The two groups
came to an encounter when Romualdo Paglinawan suddenly confronted Alvaro Hilario for
purportedly giving him a "bad stare." Silvestre apologized to the group and, offering them some
cigarettes, explained that it was the natural way Hilario gazed at people. Dijan, Paglinawan and
Lizardo then left the place while Silvestre and Hilario who lived in the same house proceeded
home. While Silvestre and Hilario were walking, the three accused, who apparently were waiting
for the duo, suddenly ganged up on, and took turns in stabbing, Hilario. At that point, Hilario,
who was walking slightly ahead of Silvestre, cried out and told the latter to flee. Silvestre ran
away until he was able to cling to a passing passenger jeepney.

A version of the defense is as follows:


The defense claimed that on the night of the incident, Crispulo Dijan and his two companions,
Romualdo Paglinawan and Oliver Lizardo, were walking on their way home when they dropped
by a store to buy some cigarettes. There, they met two persons, later identified to be Alvaro
Hilario and Roderick Silvestre. who were partaking of drinks. Paglinawan accosted one of the
duo for allegedly sharply staring at him but the other apologized to their group and explained that
his companion was already drunk. Paglinawan himself then also made an apology, and
everybody shook hands. Dijan and his friends started to walk along Paraiso Street. When Dijan
happened to look behind, he was surprised to see Paglinawan being stabbed with a knife by
Alvaro Hilario. He saw that when Paglinawan was hit on the left arm, the two grappled for the
knife's possession. Seeing Roderick Silvestre to have pulled out an icepick himself, Dijan
promptly held his hand. After disarming Silvestre, Dijan saw Paglinawan still grappling with
Hilario for the knife's possession. Realizing that Paglinawan was no match for Hilario, the latter
being much taller than Paglinawan, Dijan helped his friend and stabbed Hilario with the icepick
he wrestled away from Silvestre. He assisted Paglinawan in getting home which was only about
20 meters away from the scene of the crime.
Dijan's two co-accused, Oliver Morales Lizardo and Romualdo Paglinawan, gave a similar
account. Lizardo claimed that he ran away when Silvestre, holding an icepick, rushed towards
them. Romualdo Paglinawan said that, when their group was already at the corner of Paraiso and
Sumulong streets, he heard rushing steps of slippers and, turning his head around, Hilario
suddenly stabbed him with a knife. He was able to evade the thrust directed on his chest,
wounding him instead on his left forearm. The two grappled for the knife's possession for about
five minutes until he was weakened by the bleeding of his wound. Dijan was able to timely pull
away Hilario. Dijan then stabbed Hilario. Paglinawan stood up and walked home followed by
Dijan. He requested Dijan to bring him to the hospital for treatment but it was the policemen,
who meanwhile arrived, who brought him to the hospital. After his wounds were treated, he was
taken to the police headquarters.

The defense also presented Lani Sarmiento and Dr. Alfredo Garcia to the stand. Sarmiento
claimed that when she and a companion passed by Linda's Bakery on the night of the incident,
they noticed two male persons, a tall fellow and the other of average height, overtake them
causing her to exclaim "Fe, tingnan mo yan, parang nagmamadali, parang galit sa
mundo."2 Nearing Sumulong Street, they saw the two men approach three other male persons
who were walking towards Paraiso Street. Suddenly, the tall guy pulled out a knife and gave a
stabbing thrust to one of the three men. When they reached home, they learned that it was their
"Kuya" Jojo or Romualdo Paglinawan who had been stabbed. Dr. Garcia testified having treated
Romualdo Paglinawan on 11 April 1998 at the Amang Rodriguez Medical Centre for a stab
wound at the right forearm.

The lower court ruled in favor of the prosecution and found Dijan guilty of murder while
acquitting Paglinawan and Lizardo for failure of the prosecution to prove their guilt beyond
reasonable doubt.

Issue: Whether or not the defense of stranger should be appreciated in favor of Dijan.
Held: No. A party who invokes the justifying circumstance of "defense of a stranger" has the
burden of proving by clear and convincing evidence the exculpatory cause that can save him
from conviction.5 In order to successfully put up this defense an accused must show (1) the
existence of unlawful aggression on the part of the victim; (2) the reasonable necessity of the
means employed to prevent or repel it; and (3) that the accused has not been induced by revenge,
resentment, or other evil motive.6 The unlawful aggression must be a continuing circumstance or
must have been existing at the time the defense is made. Once unlawful aggression is found to
have ceased, the one making the defense of a stranger would likewise cease to have any
justification for killing, or even just wounding, the former aggressor.7

From the defense account, it would appear that Hilario was already disarmed and the unlawful
aggression by Hilario (if indeed he was the aggressor) to have by then been abated, when
accused-appellant still delivered the fatal thrusts on the victim.

The number of wounds sustained by the victim would itself likewise negate accused-appellant's
claim of defense of a stranger. The autopsy conducted on the corpse would show that the
deceased sustained fourteen injuries consisting of nine stab wounds, three punctured wounds, an
incised wound and an abrasion.9 Certainly, the nature and number of wounds inflicted by an
accused on the victim should be significant indicia in determining the plausibility of the defense
plea.

The judgement was affirmed with modification and was downgraded from murder to Homicide.
47. People V. Toring 191 SCRA 38

Facts: A benefit dance was held in one sitio in Lapu lapu City for the last canvassing of votes for
the candidates for princesses, attended by the entire family of one of the candidates. Also present

were members of the kwaknit gang, headed by Toring, noted for their bird-like way of dancing
and their propensity for drunkenness and provoking trouble.

Samuel, the father of the declared winner, while answering the call of nature, was approached by
Toring and two others and was stabbed from behind by Toring using a knife handed to him by a
companion. Samuel died of stab wound he sustained on the right side of his abdomen.

An information for murder was filed against Toring. The lower court rendered a decision
discrediting Toring's claim that the killing of Samuel was justified because it was done in defense
of a stranger. While Toring testified that Samuel was aiming his shotgun at the chest of Ely
Amyon (Amion), prosecution witness Joel Escobia claimed that he was at the receiving end of
Samuel's thrusts with the butt of his shotgun. To the court, such discrepancy is fatal to the
defense because in appreciating the justifying circumstance of defense of a stranger, the court
must know "with definiteness the identity of the stranger defended by the accused.

Upon appeal, Toring seeks his exoneration by contending that his assault on Samuel was justified
because he acted in defense of his first cousin, Joel Escobia is the first cousin of Toring their
fathers being brothers, although no explanation appears on record why they have
different surnames. At any rate, this allegation on relationshipwas not rebutted by the
prosecution. Escobia attested that as he was about to dance with a girl, Samuel stopped him,
pointed his shotgun at him, took a bullet from his jacket pocket, showed it to Escobia. Samuel
pointed the shotgun at his chin and told him to eat the bullet.

Issue: Whether the act of Toring in stabbing Samuel was justified for being done in defense of
his relative, Joel Escobia.

Held: NO. SC ruled that there was no reason to doubt Joel Escobia's assertion of Samuel's
unlawful aggression and that prosecution failed to prove that Joel testified to favor Toring.
However, the presence of unlawful aggression on the part of the victim and the lack of proof of
provocation on the part of Toring notwithstanding, full credence cannot be given, to Toring's
claim of defense of a relative.

Toring himself admitted in court that in 1979, he was shot with a .22 caliber revolver by Edgar
Augusto, Samuel's brother. It cannot be said, therefore, that in attacking Samuel, Toring was
impelled by pure compassion or beneficence or the lawful desire to avenge the immediate wrong
inflicted on his cousin. Rather, he was motivated by revenge, resentment or evil motive because
of a "running feud" between the Augusto and the Toring brothers. Indeed, vendetta appears to
have driven both camps to commit unlawful acts against each other.

48. Ty V. People 439 SCRA 220

Facts: This case stemmed from the filing of 7 Informations for violation of B.P. 22 against Ty
before the RTC of Manila. The said accused drew and issue to Manila Doctors Hospital to apply
on account or for value to Editha L.Vecino several post-dated checks. The said accused well
knowing that at the time of issue she did not have sufficient funds in or credit with the drawee
bank for payment of such checks in full upon its presentment, which check when presented for
payment within ninety (90) days from the date hereof, was subsequently dishonored by the
drawee bank for Account Closed and despite receipt of notice of such dishonor, said accused
failed to pay said Manila Doctors Hospital the amount of the checks or to make arrangement for
full payment of the same within five (5)banking days after receiving said notice. Ty suggests in
the prefatory statement of her Petition and Memorandum that the justifying circumstance of state
of necessity under par. 4, Art. 11 of the Revised Penal Code may find application in this case.

Issue: Whether or not justifying circumstance of state of necessity under par. 4, Art. 11 of the
Revised Penal Code may find application in this case.
Held: No. We do not agree. The law prescribes the presence of three requisites to exempt the
actor from liability under this paragraph: (1) that the evil sought to be avoided actually exists;
(2) that the injury feared be greater than the one done to avoid it; (3) that there be no other
practical and less harmful means of preventing it.
In the instant case, the evil sought to be avoided is merely expected or anticipated. If the evil
sought to be avoided is merely expected or anticipated or may happen in the future, this defense

is not applicable. Ty could have taken advantage of an available option to avoid committing a
crime. By her own admission, she had the choice to give jewelry or other forms of security
instead of postdated checks to secure her obligation.
Moreover, for the defense of state of necessity to be availing, the greater injury feared should not
have been brought about by the negligence or imprudence, more so, the willful inaction of the
actor. In this case, the issuance of the bounced checks was brought about by Tys own failure to
pay her mothers hospital bills.

49. People V. Recohermoso 56 SCRA 431

Facts: Geminiano de Leon, together with his common-law wife, son Marianito de Leon and one
Rizal Rosales, chanced upon Pio Ricohermoso. Owning a parcel of land, which Ricohermoso
cultivated as kaingin, Geminiano asked about his share of palay harvest and added that she
should be allowed to taste the palay harvested from his land. Ricohermoso said Geminiano could
collect the palay anytime.

Upon returning from his trip to Barrio Bagobasin, Geminiano dropped by Ricohermosos house
and asked him about the palay, to which the latter answered defiantly that he will not give him
the palay, whatever happens. Geminiano remonstrated and that point (as if by prearrangement),
Ricohermoso unsheathed his bolo, while his father-in-law Severo Padernal got an axe, and
attacked Geminiano. At the same time and place, Ricohermosos brother-in-law Juan Padernal
suddenly embraced Marianito. They grappled and rolled down the hill, at which point Marianito
passed out. When he regained consciousness, he discovered that the rifle he carried beforehand
was gone and that his father was mortally wounded.

The defendants shifted the responsibility of killing in their version of the case.

Issue: Whether or not appellant Juan Padernal can invoke the justifying circumstance of
avoidance of a greater evil or injury

Held: No. Juan Padernals reliance on the justifying circumstance is erroneous because his act in
preventing Marianito from shooting Ricohermoso and Severo Padernal, the aggressors in this
case, was designed to insure the killing of Geminiano de Leon without any risk to the assailants
and not an act to prevent infliction of greater evil or injury. His intention was to forestall any
interference in the assault.

Treachery was also appreciated in the case. The trial court convicted the appellants with lesiones
leves, from an attempted murder charge with respect to Marianito de Leon.
Judgment as to Juan Padernal affirmed.

50. Ambil V. Sandiganbayan 653 SCRA 576

Facts: Petitioner Ambil, Jr. testified that he was the Governor of Eastern Samar from 1998 to
2001. According to him, it was upon the advice of Adalims lawyers that he directed the transfer
of Adalims detention to his home. He cites poor security in the provincial jail as the primary
reason for taking personal custody of Adalim considering that the latter would be in the company
of inmates who were put away by his sister and guards identified with his political opponents.

For her part, Atty. White stated that she is the District Public Attorney of Eastern Samar and the
sister of Mayor Adalim. She recounted how Mayor Adalim was arrested while they were
attending a wedding in Sulat, Eastern Samar, on September 6, 1998. According to Atty. White,
she sought the alternative custody of Gov. Ambil, Jr. after Provincial Warden and herein
petitioner Apelado, Sr. failed to guarantee the mayors safety.

Meanwhile, Francisco Adalim introduced himself as the Mayor of Taft, Eastern Samar. He
confirmed his arrest on September 6, 1998 in connection with a murder case filed against him in
the Regional Trial Court (RTC) of Borongan, Eastern Samar. Adalim confirmed Atty. Whites
account that he spotted inmates who served as bodyguards for, or who are associated with, his
political rivals at the provincial jail. He also noticed a prisoner, Roman Akyatan, gesture to him
with a raised clenched fist. Sensing danger, he called on his sister for help. Adalim admitted

staying at Ambil, Jr.s residence for almost three months before he posted bail after the charge
against him was downgraded to homicide.

Petitioner Apelado, Sr. testified that he was the Provincial Jail Warden of Eastern Samar. He
recalls that on September 6, 1998, SPO3 Felipe Balano fetched him at home to assist in the arrest
of Mayor Adalim. Allegedly, Atty. White was contesting the legality of Mayor Adalims arrest
and arguing with the jail guards against booking him for detention. At the provincial jail,
petitioner was confronted by Atty. White who informed him that he was under the governor, in
the latters capacity as a provincial jailer. Petitioner claims that it is for this reason that he
submitted to the governors order to relinquish custody of Adalim.

On September 16, 2005, the Sandiganbayan, First Division, promulgated the assailed
Decision finding petitioners guilty of violating Section 3(e) of R.A. No. 3019. The court ruled
that in moving Adalim to a private residence, petitioners have conspired to accord him
unwarranted benefits in the form of more comfortable quarters with access to television and
other privileges that other detainees do not enjoy. It stressed that under the Rules, no person
under detention by legal process shall be released or transferred except upon order of the court or
when he is admitted to bail

Issue 1: Whether or not appellant Ambil is entitled to the justifying circumstance of fulfillment of
duty under Article 11(5) of the RPC.

Held: No. In the present case, when petitioners transferred Mayor Adalim from the provincial jail
and detained him at petitioner Ambil, Jr.s residence, they accorded such privilege to Adalim, not
in his official capacity as a mayor, but as a detainee charged with murder. Thus, for purposes of
applying the provisions of Section 3(e), R.A. No. 3019, Adalim was a private party.

Moreover, in order to be found guilty under the second mode, it suffices that the accused has
given unjustified favor or benefit to another in the exercise of his official, administrative or
judicial functions. The word "unwarranted" means lacking adequate or official support;
unjustified; unauthorized or without justification or adequate reason. "Advantage" means a more
favorable or improved position or condition; benefit, profit or gain of any kind; benefit from
some course of action. "Preference" signifies priority or higher evaluation or desirability; choice
or estimation above another.

Without a court order, petitioners transferred Adalim and detained him in a place other than the
provincial jail. The latter was housed in much more comfortable quarters, provided better
nourishment, was free to move about the house and watch television. Petitioners readily
extended these benefits to Adalim on the mere representation of his lawyers that the mayors life
would be put in danger inside the provincial jail.

As the Sandiganbayan ruled, however, petitioners were unable to establish the existence of any
risk on Adalims safety. To be sure, the latter would not be alone in having unfriendly company
in lockup. Yet, even if we treat Akyatans gesture of raising a closed fist at Adalim as a threat of
aggression, the same would still not constitute a special and compelling reason to warrant
Adalims detention outside the provincial jail. For one, there were nipa huts within the perimeter
fence of the jail which could have been used to separate Adalim from the rest of the prisoners
while the isolation cell was undergoing repair. Anyhow, such repair could not have exceeded the
85 days that Adalim stayed in petitioner Ambil, Jr.s house. More importantly, even if Adalim
could have proven the presence of an imminent peril on his person to petitioners, a court order
was still indispensable for his transfer

51. Mamangon V. People G.R. 149152

Facts: Policeman (PO2) Rufino Mamangun was responding to a robbery-holdup call, with his
fellow police officers, at Brgy. Calvario, Meycauayan, Bulacan. A certain Liberty Contreras was
heard shouting, which prompted residents to respond and chase the suspect, who entered the yard
and proceeded to the rooftop of Antonio Abacan. Mamangun, with PO2 Diaz and Cruz, each
armed with a drawn handgun, searched the rooftop and saw a man who they thought was the
robbery suspect. Mamangun, who was ahead of the group, fired his gun once and hit the man,
who turned out to be Gener Contreras (not the suspect) Contreras died of the gunshot wound.

According to the lone witness Crisanto Ayson, he accompanied the policemen to the lighted
rooftop. He was beside Mamangun when he (Ayson) recognized the deceased. According to
Ayson, Mamangun pointed his gun at the man, who instantly exclaimed Hindi ako, hindi ako!
to which Mamangun replied, Anong hindi ako? and shot him.

The defense rejects this testimony, alleging that they were the only ones at the dark rooftop when
Mamangun noticed a crouching man who suddenly continued to run. Mamangun shouted Pulis,
tigil! whereupon the person stopped and raised a steel pipe towards Mamanguns head. This
prompted Mamangun to shoot the person. The three police claim that Contreras only said Hindi
ako, hindi ako only when they approached him. Mamangun then asked Why did you go to the
rooftop? You know there are policemen here. Mamangun reported the incident to the desk
officer who directed investigator Hernando Banez to investigate the incident. Banez later on
found a steel pipe on the roof.

Issue: Whether or not the death of the victim was the necessary consequence of the petitioners
fulfillment of his duty.

Held: No. The Court denies the instant petition and affirms Sandiganbayans decision after
finding the petitioners testimony to be nothing but a concocted story designed to evade criminal
liability. Per Sandiganbayans observations, the defense was self-serving for the accused and
biased with respect to his co-policemen-witnesses because:

After supposed introductions and forewarnings uttered allegedly by Mamangun, it is contrary to


human experience for a man (who is not the suspect) to attack one of three policemen with
drawn guns;

Mamanguns admission that he did not ask the victim Why did you try to hit me, if you are not
the one? clearly belies their claim;

The location of the entry of bullet belies their claim because it appears that the victim
instinctively shielded himself instead.

Additionally, petitioners pretense that Contreras struck him was not initially reported to the desk
and was only conveniently remembered when the investigator found a pipe in the crime scene.

Acts in the fulfillment of duty and self-defense does not completely justify the petitioners firing
the fatal gunshot. The element of unlawful aggression on the part of the victim was absent, which
leads to the failure of the petitioners plea. Also, there can only be incomplete justification (a
privileged mitigating circumstance) in the absence of a necessary justifying circumstance the
injury was caused by necessary consequence of due performance of duty.

52. Baxinela V. People 485 SCRA 331

Facts: Petitioner SPO2 Eduardo L. Baxinela was in a pub drinking with two other policemen in
as early as 11:00 p.m. of October 18, 1996. At around 12:00 a.m. to 12:30 a.m. there was a minor
altercation between the deceased Sgt. Lajo and another customer at the pub but eventually the
two were able to patch things up. While on his way out, Lajo was followed by Braxinela with a
gun already drawn out. From behind, Baxinela held Lajos left arm and asked why he was
carrying a gun. Thereafter an explosion coming from Baxinelas gun was heard. Lajo, still
standing,
took
two
steps
and
then
fell
down.
Issue: Whether or not fulfilment of duty may validly be invoked by the petitioner?

Decision: No. In order to avail of this justifying circumstance it must be shown that: 1) the
accused acted in the performance of a duty or in the lawful exercise of a right or office; and 2)
the injury caused or the offense committed is the necessary consequence of the due performance
of duty or the lawful exercise of a right or office. While the first condition is present, the second
is clearly lacking. Baxinelas duty was to investigate the reason why Lajo had a gun tucked
behind his waist in a public place. This was what Baxinela was doing when he confronted Lajo at
the entrance, but perhaps through anxiety, edginess or the desire to take no chances, Baxinela
exceeded his duty by firing upon Lajo who was not at all resisting. The shooting of Lajo cannot
be considered due performance of a duty if at that time Lajo posed no serious threat or harm to
Baxinela or to the civilians in the pub. The Court will, however, attribute to Baxinela the
incomplete defense of fulfillment of a duty as a privileged mitigating circumstance. In Lacanilao
v. Court of Appeals, it was held that if the first condition is fulfilled but the second is wanting,
Article 69 of the Revised Penal Code is applicable so that the penalty lower than one or two
degrees than that prescribed by law shall be imposed.

53. Pomoy V. People 439 SCRA 439

Facts: Roweno Pomoy a member of the PNP, Provincial Mobile Force Company, went to the
stockade to fetch Tomas Balboa for tactical interrogation. While holding the door knob of the
interrogation room Balboa suddenly grabbed the gun holstered by Pomoy and a fight ensued
which led to the shooting of Balboa twice while they were grappling.

The RTC and CA ruled that because of the 2 shots, it could not have been self defense, hence,
they convicted Pomoy of homicide.

Issue: Whether or not the accused should be acquitted based on self defense.

Held: Yes. The foregoing account demonstrates that petitioner did not have control of the gun
during the scuffle. The deceased persistently attempted to wrest the weapon from him, while he
resolutely tried to thwart those attempts. That the hands of both petitioner and the victim were all
over the weapon was categorically asserted by the eyewitness. In the course of grappling for the
gun, both hands of petitioner were fully engaged -- his right hand was trying to maintain
possession of the weapon, while his left was warding off the victim. It would be difficult to
imagine how, under such circumstances, petitioner would coolly and effectively be able to
release the safety lock of the gun and deliberately aim and fire it at the victim.

The elements of accident are as follows: 1) the accused was at the time performing a lawful act
with due care; 2) the resulting injury was caused by mere accident; and 3) on the part of the
accused, there was no fault or no intent to cause the injury.27 From the facts, it is clear that all
these elements were present. At the time of the incident, petitioner was a member -- specifically,
one of the investigators -- of the Philippine National Police (PNP) stationed at the Iloilo
Provincial Mobile Force Company. Thus, it was in the lawful performance of his duties as
investigating officer that, under the instructions of his superior, he fetched the victim from the
latters cell for a routine interrogation.

Again, it was in the lawful performance of his duty as a law enforcer that petitioner tried to
defend his possession of the weapon when the victim suddenly tried to remove it from his
holster. As an enforcer of the law, petitioner was duty-bound to prevent the snatching of his
service weapon by anyone, especially by a detained person in his custody. Such weapon was
likely to be used to facilitate escape and to kill or maim persons in the vicinity, including
petitioner himself.

Petitioner cannot be faulted for negligence. He exercised all the necessary precautions to prevent
his service weapon from causing accidental harm to others. As he so assiduously maintained, he
had kept his service gun locked when he left his house; he kept it inside its holster at all times,
especially within the premises of his working area.
At no instance during his testimony did the accused admit to any intent to cause injury to the
deceased, much less kill him. Furthermore, Nicostrato Estepar, the guard in charge of the
detention of Balboa, did not testify to any behavior on the part of petitioner that would indicate
the intent to harm the victim while being fetched from the detention cell.

The participation of petitioner, if any, in the victims death was limited only to acts committed in
the course of the lawful performance of his duties as an enforcer of the law. The removal of the
gun from its holster, the release of the safety lock, and the firing of the two successive shots -- all
of which led to the death of the victim -- were sufficiently demonstrated to have been
consequences of circumstances beyond the control of petitioner. At the very least, these factual
circumstances create serious doubt on the latters culpability.

54. Angcaco v. People G.R. No. 146664

Facts: At around 4 o'clock in the morning of September 25, 1980, Noe Bergante and his brother
Noel Bergante and his cousin Freddie Ganancial were awakened by the sound of gunfire while
they were asleep in their house. Petitioner John Angcaco and his co-accused were serving a
warrant of arrest issued on Restituto Bergante, who was wanted in connection with a robbery
case. Noel informed the policemen that his father was not in the house, having gone to Puerto
Princesa. One of them ordered the men in the house to come out. Noel accordingly went to the
gate and later called Noe to also come out of the house. Noe and his cousin, Freddie Ganancial,
did as bidden.

Once they were outside the house, Noe and Freddie were flanked by petitioner Angcaco on the
right side and accused Ramon Decosto on the left side. Decosto pointed an armalite at the two
and warned them not to run. Noe and Freddie joined Noel Bergante. Protacio Edep approached
Freddie saying, "You are tough," and pushed him. Then, shots rang out from the armalite and
short firearm of Decosto and Edep, as a result of which Freddie Ganancial turned around and
dropped to the ground face down.

Issue: Whether or not the justifying circumstance of fulfilment of duty is applicable in this case?

Decision: No. For this justifying circumstance to be appreciated, the following must be
established: (1) that the offender acted in the lawful exercise of a right or a duty; and (b) that the
injury or offense committed be the necessary consequence of the due performance of such right
or office.

In this case, the mission of petitioner and his colleagues was to effect the arrest of Restituto
Bergante. The standard procedure in making an arrest was, first, to identify themselves as police
officers and to show the warrant to the arrestee and to inform him of the charge against him, and,
second, to take the arrestee under custody. But, it was not shown here that the killing of
Ganancial was in furtherance of such duty. No evidence was presented by the defense to prove
that Ganancial attempted to prevent petitioner and his fellow officers from arresting Restituto
Bergante. There was in fact no clear evidence as to how Freddie Ganancial was shot. Indeed, as
already stated, any attempt by the victim to arrest the wanted person was pointless as Restituto
Bergante was not in his house. As regards the second requisite, there can be no question that the
killing of Freddie Ganancial was not a necessary consequence of the arrest to be made on
Restituto Bergante.

55. Sycip V. CA G.R. 125059


Facts: On August 24, 1989, Francisco T. Sycip agreed to buy, on installment, from Francel Realty
Corporation (FRC), a townhouse unit in the latter's project at Bacoor, Cavite.
Upon execution of the contract to sell, Sycip, as required, issued to FRC, forty-eight (48)
postdated checks, each in the amount of P9,304.00, covering 48 monthly installments.
After moving in his unit, Sycip complained to FRC regarding defects in the unit and incomplete
features of the townhouse project. FRC ignored the complaint. Dissatisfied, Sycip served on FRC
two (2) notarial notices to the effect that he was suspending his installment payments on the unit

pending compliance with the project plans and specifications, as approved by the Housing and
Land Use Regulatory Board (HLURB). Sycip and 12 out of 14 unit buyers then filed a complaint
with the HLURB. The complaint was dismissed as to the defects, but FRC was ordered by the
HLURB to finish all incomplete features of its townhouse project. Sycip appealed the dismissal
of the complaint as to the alleged defects.
Notwithstanding the notarial notices, FRC continued to present for encashment Sycip's postdated
checks in its possession. Sycip sent "stop payment orders" to the bank. When FRC continued to
present the other postdated checks to the bank as the due date fell, the bank advised Sycip to
close his checking account to avoid paying bank charges every time he made a "stop payment"
order on the forthcoming checks. Due to the closure of petitioner's checking account, the drawee
bank dishonored six postdated checks. FRC filed a complaint against petitioner for violations of
B.P. Blg. 22 involving said dishonored checks.

Issue: Whether or not the CA erred in affirming the conviction of petitioner for suspending the
installment payments on the units pending the compliance with the project plans and
specifications.
Held: No. To rely on the presumption created by B.P. No. 22 as the prosecution did in this case,
would be to misconstrue the import of requirements for conviction under the law. It must be
stressed that every element of the offense must be proved beyond reasonable doubt, never
presumed. Furthermore, penal statutes are strictly construed against the State and liberally in
favor of the accused. Under the Bouncing Checks Law, the punishable act must come clearly
within both the spirit and letter of the statute.
While B.P. Blg. 22 was enacted to safeguard the interest of the banking system, it is difficult to
see how conviction of the accused in this case will protect the sanctity of the financial system.
Moreover, protection must also be afforded the interest of townhouse buyers under P.D. No.
957. A statute must be construed in relation to other laws so as to carry out the legitimate ends
and purposes intended by the legislature. Courts will not strictly follow the letter of one statute
when it leads away from the true intent of legislature and when ends are inconsistent with the
general purpose of the act. More so, when it will mean the contravention of another valid statute.
Both laws have to be reconciled and given due effect.
Note that we have upheld a buyer's reliance on Section 23 of P.D. 957 to suspend payments until
such time as the owner or developer had fulfilled its obligations to the buyer. This exercise of a
statutory right to suspend installment payments, is to our mind, a valid defense against the
purported violations of B.P. Blg. 22 that petitioner is charged with.
Given the findings of the HLURB as to incomplete features in the construction of petitioner's and
other units of the subject condominium bought on installment from FRC, we are of the view that

petitioner had a valid cause to order his bank to stop payment. To say the least, the third element
of "subsequent dishonor of the check... without valid cause" appears to us not established by the
prosecution. As already stated, the prosecution tried to establish the crime on a prima
facie presumption in B.P. Blg. 22. Here that presumption is unavailing, in the presence of a valid
cause to stop payment, thereby negating the third element of the crime.
Offenses punished by a special law, like the Bouncing Checks Law, are not subject to the
Revised Penal Code, but the Code is supplementary to such a law. We find nothing in the text of
B.P. Blg. 22, which would prevent the Revised Penal Code from supplementing it. Following
Article 11 (5) of the Revised Penal Code, petitioner's exercise of a right of the buyer under
Article 23 of P.D. No. 957 is a valid defense to the charges against him.

56. Ambil V. Sandiganbayan 653 SCRA 576

Facts: Petitioner Ambil, Jr. testified that he was the Governor of Eastern Samar from 1998 to
2001. According to him, it was upon the advice of Adalims lawyers that he directed the transfer
of Adalims detention to his home. He cites poor security in the provincial jail as the primary
reason for taking personal custody of Adalim considering that the latter would be in the company
of inmates who were put away by his sister and guards identified with his political opponents.

For her part, Atty. White stated that she is the District Public Attorney of Eastern Samar and the
sister of Mayor Adalim. She recounted how Mayor Adalim was arrested while they were
attending a wedding in Sulat, Eastern Samar, on September 6, 1998. According to Atty. White,
she sought the alternative custody of Gov. Ambil, Jr. after Provincial Warden and herein
petitioner Apelado, Sr. failed to guarantee the mayors safety.

Meanwhile, Francisco Adalim introduced himself as the Mayor of Taft, Eastern Samar. He
confirmed his arrest on September 6, 1998 in connection with a murder case filed against him in
the Regional Trial Court (RTC) of Borongan, Eastern Samar. Adalim confirmed Atty. Whites
account that he spotted inmates who served as bodyguards for, or who are associated with, his
political rivals at the provincial jail. He also noticed a prisoner, Roman Akyatan, gesture to him
with a raised clenched fist. Sensing danger, he called on his sister for help. Adalim admitted
staying at Ambil, Jr.s residence for almost three months before he posted bail after the charge
against him was downgraded to homicide.

Petitioner Apelado, Sr. testified that he was the Provincial Jail Warden of Eastern Samar. He
recalls that on September 6, 1998, SPO3 Felipe Balano fetched him at home to assist in the arrest
of Mayor Adalim. Allegedly, Atty. White was contesting the legality of Mayor Adalims arrest
and arguing with the jail guards against booking him for detention. At the provincial jail,
petitioner was confronted by Atty. White who informed him that he was under the governor, in
the latters capacity as a provincial jailer. Petitioner claims that it is for this reason that he
submitted to the governors order to relinquish custody of Adalim.

On September 16, 2005, the Sandiganbayan, First Division, promulgated the assailed
Decision finding petitioners guilty of violating Section 3(e) of R.A. No. 3019. The court ruled
that in moving Adalim to a private residence, petitioners have conspired to accord him
unwarranted benefits in the form of more comfortable quarters with access to television and
other privileges that other detainees do not enjoy. It stressed that under the Rules, no person
under detention by legal process shall be released or transferred except upon order of the court or
when he is admitted to bail

Issue 2: Whether or not Apelado Sr., is entitled to the justifying circumstance of obedience to an
order issued by a superior for some lawful purpose under Article 11(6) of the RPC.

Held: No. Petitioner Apelado, Sr. invokes the justifying circumstance of obedience to an order
issued for some lawful purpose. Under paragraph 6, Article 11 of the RPC, any person who acts
in obedience to an order issued by a superior for some lawful purpose does not incur any
criminal liability. For this justifying circumstance to apply, the following requisites must be
present: (1) an order has been issued by a superior; (2) such order must be for some lawful
purpose; and (3) the means used by the subordinate to carry out said order is lawful. Only the
first requisite is present in this case.

While the order for Adalims transfer emanated from petitioner Ambil, Jr., who was then
Governor, neither said order nor the means employed by petitioner Apelado, Sr. to carry it out
was lawful. In his capacity as the Provincial Jail Warden of Eastern Samar, petitioner Apelado,
Sr. fetched Mayor Adalim at the provincial jail and, unarmed with a court order, transported him
to the house of petitioner Ambil, Jr. This makes him liable as a principal by direct participation
under Article 17(1) of the RPC.

57. Tabuena V. Sandiganbayan 268 SCRA 297

Facts: Then President Marcos instructed Tabuena over the phone to pay directly to the presidents
office and in cash what the MIAA owes the Philippine National Construction Corporation
(PNCC), to which Tabuena replied, Yes, sir, I will do it. About a week later, Tabuena received
from Mrs. Fe Roa-Gimenez, then private secretary of Marcos, a Presidential Memorandum dated
January 8, 1986 (hereinafter referred to as MARCOS Memorandum) reiterating in black and
white such verbal instruction.

In obedience to President Marcos verbal instruction and memorandum, Tabuena, with the help
of Dabao and Peralta, caused the release of P55 Million of MIAA funds by means of three (3)
withdrawals.
The first withdrawal was made on January 10, 1986 for P25 Million, following a letter of even
date signed by Tabuena and Dabao requesting the PNB extension office at the MIAA - the
depository branch of MIAA funds, to issue a managers check for said amount payable to
Tabuena. The check was encashed, however, at the PNB Villamor Branch. Dabao and the
cashier of the PNB Villamor branch counted the money after which, Tabuena took delivery
thereof. The P25 Million in cash were then placed in peerless boxes and duffle bags, loaded on a
PNB armored car and delivered on the same day to the office of Mrs. Gimenez located at Aguado
Street fronting Malacaang. Mrs. Gimenez did not issue any receipt for the money received.
Similar circumstances surrounded the second withdrawal/encashment and delivery of
another P25 Million, made on January 16, 1986.
The third and last withdrawal was made on January 31, 1986 for P5 Million. Peralta was
Tabuenas co-signatory to the letter- request for a managers check for this amount. Peralta
accompanied Tabuena to the PNB Villamor branch as Tabuena requested him to do the counting
of the P5 Million. After the counting, the money was placed in two (2) peerless boxes which
were loaded in the trunk of Tabuenas car. Peralta did not go with Tabuena to deliver the money
to Mrs. Gimenez office at Aguado Street. It was only upon delivery of the P5 Million that Mrs.
Gimenez issued a receipt for all the amounts she received from Tabuena. The receipt, dated
January 30, 1986.
The position of the prosecution was that there were no outstanding obligations in favor of PNCC
at the time of the disbursement of the P55 Million. On the other hand, the defense of Tabuena
and Peralta, in short, was that they acted in good faith. Tabuena claimed that he was merely
complying with the MARCOS Memorandum which ordered him to forward immediately to the
Office of the President P55 Million in cash as partial payment of MIAAs obligations to PNCC,

and that he (Tabuena) was of the belief that MIAA indeed had liabilities to PNCC. Peralta for
his part shared the same belief and so he heeded the request of Tabuena, his superior, for him
(Peralta) to help in the release of P5 Million.
With the rejection by the Sandiganbayan of their claim of good faith which ultimately led to their
conviction.

Issue: Whether or not the accused should be acquitted based on the fact that they complied with
the order of then President Marcos, to pay PNCC 55million pesos.

Held: Yes they should be acquitted. Tabuena had no other choice but to make the withdrawals,
for that was what the MARCOS Memorandum required him to do. He could not be faulted if he
had to obey and strictly comply with the presidential directive, and to argue otherwise is
something easier said than done. Marcos was undeniably Tabuenas superior the former being
then the President of the Republic who unquestionably exercised control over government
agencies such as the MIAA and PNCC. In other words, Marcos had a say in matters involving
inter-government agency affairs and transactions, such as for instance, directing payment of
liability of one entity to another and the manner in which it should be carried out. And as a
recipient of such kind of a directive coming from the highest official of the land no less, good
faith should be read on Tabuenas compliance, without hesitation nor any question, with the
MARCOS Memorandum. Tabuena therefore is entitled to the justifying circumstance of Any
person who acts in obedience to an order issued by a superior for some lawful purpose. The
subordinate-superior relationship between Tabuena and Marcos is clear. And so too, is the
lawfulness of the order contained in the MARCOS Memorandum, as it has for its purpose
partial payment of the liability of one government agency (MIAA) to another (PNCC).

What is more significant to consider is that the MARCOS Memorandum is patently legal (for on
its face it directs payment of an outstanding liability) and that Tabuena acted under the honest
belief that the P55 million was a due and demandable debt and that it was just a portion of a
bigger liability to PNCC

Thus, even if the order is illegal if it is patently legal and the subordinate is not aware of its
illegality, the subordinate is not liable, for then there would only be a mistake of fact committed
in good faith.

We do not agree. It must be stressed that the MARCOS Memorandum directed Tabuena to pay
immediately the Philippine National Construction Corporation, thru this office, the sum of
FIFTY FIVE MILLION...., and that was what Tabuena precisely did when he delivered the
money to Mrs. Gimenez. Such delivery, no doubt, is in effect delivery to the Office of the
President inasmuch as Mrs. Gimenez was Marcos secretary then. Furthermore, Tabuena had
reasonable ground to believe that the President was entitled to receive the P55 Million since he
was certainly aware that Marcos, as Chief Executive, exercised supervision and control over
government agencies. And the good faith of Tabuena in having delivered the money to the
Presidents office (thru Mrs. Gimenez), in strict compliance with the MARCOS Memorandum,
was not at all affected even if it later turned out that PNCC never received the money.

58. People V. Beronilla 96 PHIL 566

Facts: Manuel Beronilla, Policarpio Paculdo, Filipino Velasco and Jacinto Adriatico file an
appeal from the judgement of the Abra CFI, which convicted them of murder for the execution of
Arsenio Borjal, the elected mayor of La, Paz, Abra (at the outbreak of war), which was found to
be aiding the enemy.

Borjal moved to Bangued because of death threats was succeeded by Military Mayor Manuel
Beronilla, who was appointed by Lt. Col. Arbold, regimental commander of the 15th Infantry of
the Phil. Army, operating as guerilla unit in Abra. Simultaneously upon his appointment,
Beronilla received a memorandum which authorized him to appoint a jury of 12 bolo men to try
persons accused of treason, espionage and aiding or abetting the enemy.

Upon the return of Borjal and his family to Abra, to escape bombing in Bangued, he was placed
under custody and tried and sentenced to death by the jury based on various complaints made by
the residents. Beronilla reported this to Col. Arnold who replied, saying I can only
compliment you for your impartial but independent way of handling the whole case.

Two years thereafter, Beronilla, along with the executioner, digger and jury, were indicted for the
murder of Borjal. Soon after, President Manuel Roxas issued Executive Proclamation 8, which
granted amnesty to persons who committed acts in furtherance of the resistance to the enemy
against persons aiding in the war efforts of the enemy.

The rest of defendants applied and were granted amnesty, but Beronilla and others were
convicted on the grounds that the crime was made on purely personal motives and that the crime
was committed after the expiration of time limit for amnesty proclamation.

Issue: W/N the defendant-appellants actions are covered by justifying circumstances for
obedience to lawful order of superior

Held:Yes. The accused acted upon orders of their superior officers, which as military
subordinates, they could not question and obeyed in good faith without the being aware of its
illegality.

The evidence is sufficient to sustain the claim of the defense that arrest, prosecution and trial of
Borjal was done in pursuant to express orders of superiors. Additionally, it could not be
established that Beronilla received the radiogram from Colonel Volckmann, overall area
commander, which called attention to the illegality of Borjals conviction and sentence. Had
Beronilla known the violation, he would not have dared to report it to Arnold. The conduct of the
accused also does not show malice on their part because of the conduct of the trial, defense
through counsel given to Borjal, suspension of trial based on doubts of illegality and death
sentence review sent to the superior officers.

Criminal intent then could not be established. The maxim here is actus non facit reum, nisi mens
rea (Crime is not committed if the mind of the person performing the act complained of to be
innocent).

Additionally, the lower court should not have denied their claim to the benefits of the Guerilla
Amnesty Proclamation No. 8 inspite of contradictory dates of liberation of La Paz, Abra. Even if
the dates were contradictory, the court should have found for the Beronila, et al because if there
are any reasonable doubt as to whether a given case falls within the (amnesty) proclamation
should be resolved in favor of the accused.

59. People V. Bulagao 658 SCRA 746

Facts: On June 17, 2000, at around 8:00 p.m., AAA and FFF were sleeping in a room which had
no door. AAA was suddenly awakened when she felt somebody enter the room. She recognized
the accused-appellant as the intruder, and saw that he was holding a knife. Accused-appellant
poked the knife at AAAs neck, causing her to freeze in fear. Accused-appellant removed AAAs
clothes, and then his own. Both AAA and accused-appellant were wearing t-shirt and shorts
before the undressing. Accused-appellant kissed her neck and inserted his penis into her vagina.
FFF woke up at this moment, but accused-appellant did not stop and continued raping AAA for
one hour.9

On June 29, 2000, AAA was residing in the house of her sister, also located in Lolomboy,
Bocaue, Bulacan. At around 11:00 p.m. on that day, AAA was sleeping in the second floor of the
house, where there are no rooms. AAA was roused from her sleep when accused-appellant was
already undressing her. Accused-appellant removed his shorts and inserted his penis into her
vagina. AAA tried to resist, but accused-appellant held her hands. Accused-appellant then
touched her breasts and kissed her. Accused-appellant remained on top of her for half an hour.
AAA told her mother, BBB, and her brother, EEE, about the rape incidents. Upon learning of the
same, BBB did not believe AAA and whipped her.

When it was time for the defense to present their evidence more than a year later, it also
presented as its witness AAA, who recanted her testimony for the prosecution. This time, she
testified that the sexual encounters between her and the accused-appellant were consensual. She
fabricated the charge of rape against the accused-appellant because she was supposedly angry
with him. She also claimed that she was instructed by the police officer who investigated the
incident to say that the accused-appellant used a knife. She also testified that she was raped by
her father CCC when she was seven years old. She was recanting her previous testimony because
she purportedly was no longer angry with accused-appellant.

On cross-examination, AAA clarified that she fabricated the charge of rape because she was
angry with the accused-appellant for making her do laundry work for him. However, when asked
if she "consented and voluntarily submitted" herself to the accused-appellant when she had
sexual intercourse with him, she answered in the negative. She had been released from the
custody of the DSWD and was alone by herself for some time, but she now lives with the family
of accused-appellant.

Another witness for the defense was Yolanda Palma, a clinical psychologist. She conducted a
mental examination on accused-appellant on September 12, 2002, and found that accusedappellant was suffering from mental retardation as he had an IQ of below 50.
Accused-appellant, who was 40 years old when he testified on June 15, 2005, claimed that AAA
seduced him by removing her clothes. He asserted that they ended up merely kissing each other
and did not have sexual intercourse. He denied pointing a knife at AAA. AAA accused him of
rape because she was asking for P300 from him after they kissed. Accused-appellant also
testified that there was no legal proceeding for the adoption of AAA ("ampun-ampunan lang").

The RTC observed that AAA was in the custody of the DSWD when she testified for the
prosecution, and was returned to the family of the accused-appellant after her original testimony.
It was during the time when she was back in the custody of the accused-appellants family that
she recanted her testimony for the prosecution. According to the RTC, it is clear that she had no
other place to go to as she was completely orphaned and was dependent on the family of the
accused, and it was understandable that she may have recanted in order to remain in the good
graces of the accused-appellants family.21

As regards the defense of accused-appellant that he was suffering from mental retardation, the
RTC noted that the psychological examination of accused-appellant was conducted more than a
couple of years after the dates of the complained of incidents. There was no showing from the
findings of the psychologist that accused-appellant had the same mental or psychological
condition at the time of the said incidents. Even assuming that accused-appellant was of such
mental state at the time of the incidents, the psychologist testified that accused-appellant had the
capacity to discern right from wrong.

On April 14, 2008, the Court of Appeals rendered its Decision affirming that of the RTC, except
with a modification on the penalty in view of the enactment of Republic Act No. 9346
prohibiting the imposition of death penalty.

Issue: Whether or not imbecility should be taken in favor of the accused.

Held: Accused appellant, in his appeal, did not insist on the allegation in the trial court that he
was suffering from mental retardation. Nevertheless, we agree with the finding of the trial court
that there was no proof that the mental condition accused-appellant allegedly exhibited when he

was examined by Yolanda Palma was already present at the time of the rape incidents. Anyone
who pleads the exempting circumstance of insanity bears the burden of proving it with clear and
convincing evidence. Besides, this Court observes that neither the acts of the accused appellant
proven before the court, nor his answers in his testimony, show a complete deprivation of
intelligence or free will. Insanity presupposes that the accused was completely deprived of
reason or discernment and freedom of will at the time of the commission of the crime. Only
when there is a complete deprivation of intelligence at the time of the commission of the crime
should the exempting circumstance of insanity be considered.

60. People V. Valledor 383 SCRA 653

Facts: This case is about Enrico Valledors appeal of his conviction for the crime of
consummated, frustrated and attempted murder. Last 06 March 1991, the appellant attacked
Roger Cabiguen (stabbed on the forearm), Elza Rodriguez (stabbed on the chest), Ricardo
Maglalang (was inflicted w/ physical injuries on different parts of the body) There were two
other people inside the room (they were not harmed by the appellant). Roger and Ricardo were
both wounded, while Elza died from the stab wound. After his arrest, accused-appellant was
intermittently confined at the National Center for Mental Health. Thus, he was arraigned only on
February 19, 1993 wherein he pleaded not guilty. Thereafter, the cases were archived until
November 15, 1994, when accused-appellant was declared mentally fit to withstand trial. This
time, accused-appellant admitted commission of the crimes charged but invoked the exempting
circumstance
of
insanity.
The appellants plea of insanity was anchored on the following instances:
1.) On January 1990 the Mother of the appellant noticed that he is behaving abnormally. The
mother then brought the appellant to Dr. de Guzman, a medical practitioner. She then disclosed
to
the
doctor
that
insanity
runs
in
the
family
2.) Dr. de Guzman then diagnosed the appellant as suffering from psychosis w/ schizophrenia
and prescribed the appellant w/ an anti-depressant known as thoracin which kept the appellant
sane
for
a
period
of
2
months
3.) On 04 March 1991, the appellants mother noticed that he is acting strangely, so she left to
buy Thoracin, but when she returned the appellant is nowhere to be found.

4.) On 06 March 1991 (date of commission of crime) he was seen swimming across the river
Barangay Captain and Councilman took the appellant out of the water inside the boat, accusedappellant kept on crying and uttering words to the effect that his family will be killed. Suspecting
that appellant was mentally ill, the Barangay Captain, asked the Councilman to accompany
accused-appellant to Puerto Princesa City. Sibunga acceded and thereafter took a jeepney with
appellant, while on the jeepney the appellant then jumped off the jeepney and boarded a tricycle.
5.) On 11 March 1991 he was interviewed by the City Health Officer I and was recommended to
be
committed
to
the
NATIONAL
MENTAL
HOSPITAL
6.) While under the care of the hospital the medical findings for the appellant was that he was
suffering
from:
Psychosis
or
Insanity
classified
under
Schizophrenia
Issue: Can Insanity as an exempting circumstance be granted to the accused?
Held: No, since in considering insanity as a defense, it presumed that all persons to be of
sound mind. Otherwise stated, the law presumes all acts to be voluntary, and it is improper to
presume
that
acts
were
done
unconsciously
The acts made by the appellant shows that he does not have complete absence of the power to
discern as shown by his stabbing of the two victims (roger and elza) while leaving the other two
people in the room unharmed, also his action of fleeing from the scene after the incident
indicated
that
he
was
aware
of
the
wrong
he
committed.
The actions performed by the appellant does not sufficiently prove his insanity at the time of
commission of the crime: "A man may act crazy but it does not necessarily and conclusively
prove
that
he
is
legally
so."
Although it can be argued that the appellant is suffering from mental illness what is decisive is
his mental condition at the time of the perpetration of the offense. Failing to discharge the burden
of proving that he was legally insane when he stabbed the victims, he should be held liable for
his
felonious
acts.
61. People V. Jacinto645 SCRA 590

Facts: Accused was found guilty of rape of a minor by the RTC and was sentenced to death.
However, the defense moved to reopen trial for reception of newly discovered evidence stating

that appellant was apparently born on 1 March 1985 and that he was only seventeen (17) years
old when the crime was committed on 28 January 2003. The trial court appreciated the evidence
and reduced the penalty from death to reclusion perpetua.

Issue: Whether or not R.A. 9344 applies in the case of the accused even though the incident
occurred 3 years prior to its enactment and is a heinous crime, thus, allowing him to a suspended
sentence.

Held: Yes. Republic Act No. 9344 warrants the suspension of sentence of a child in conflict with
the law notwithstanding that he/she has reached the age of majority at the time the judgment of
conviction is pronounced. Thus:

SEC. 38. Automatic Suspension of Sentence. - Once the child who is under eighteen (18) years
of age at the time of the commission of the offense is found guilty of the offense charged, the
court shall determine and ascertain any civil liability which may have resulted from the offense
committed. However, instead of pronouncing the judgment of conviction, the court shall place
the child in conflict with the law under suspended sentence, without need of
application: Provided, however, That suspension of sentence shall still be applied even if the
juvenile is already eighteen (18) years of age or more at the time of the pronouncement of his/her
guilt.(Emphasis supplied.)
xxxx

Applying Declarador v. Gubaton, which was promulgated on 18 August 2006, the Court of
Appeals held that, consistent with Article 192 of Presidential Decree No. 603, as amended, the
aforestated provision does not apply to one who has been convicted of an offense punishable by
death, reclusion perpetua or life imprisonment.

Meanwhile, on 10 September 2009, this Court promulgated the decision in Sarcia, overturning
the ruling inGubaton. Thus:

The xxx provision makes no distinction as to the nature of the offense committed by the child in
conflict with the law, unlike P.D. No. 603 and A.M. No. 02-1-18-SC. The said P.D. and Supreme

Court (SC) Rule provide that the benefit of suspended sentence would not apply to a child in
conflict with the law if, among others, he/she has been convicted of an offense punishable by
death, reclusion perpetua or life imprisonment. In construing Sec. 38 of R.A. No. 9344, the Court
is guided by the basic principle of statutory construction that when the law does not distinguish,
we should not distinguish. Since R.A. No. 9344 does not distinguish between a minor who has
been convicted of a capital offense and another who has been convicted of a lesser offense, the
Court should also not distinguish and should apply the automatic suspension of sentence to a
child in conflict with the law who has been found guilty of a heinous crime.
Be that as it may, to give meaning to the legislative intent of the Act, the promotion of the
welfare of a child in conflict with the law should extend even to one who has exceeded the age
limit of twenty-one (21) years, so long as he/she committed the crime when he/she was still a
child. The offender shall be entitled to the right to restoration, rehabilitation and reintegration in
accordance with the Act in order that he/she is given the chance to live a normal life and become
a productive member of the community. The age of the child in conflict with the law at the time
of the promulgation of the judgment of conviction is not material. What matters is that the
offender committed the offense when he/she was still of tender age.

Thus, appellant may be confined in an agricultural camp or any other training facility in
accordance with Sec. 51 of Republic Act No. 9344.

62. People V. Arpon 662 SCRA 506

Facts: AAA was raped by his Uncle, Henry Arpon,once in 1995 when she was 8 yearsold and 7
times in 1999 when she was 12 years old. In defense, Arpon testified that the alleged first rape
incident happened when he was still 13 years old. He stated that in 1995 and 1999 he was living
and working in Tacloban City while the incident happened in a different Municipality.
Furthermore, he did not go to AAAs house because the latters parents were his enemies because
he did not work with them in ther ice field. However, in the cross-examination, he admitted that
he visits his parents who are living 2 kilometers away from AAAs house, once every month.

RTC convicted Arpon of one count of statutory rape and seven counts of rape which was
qualified by AAAs minority and relationship. It found more credible the testimony of AAA as
she was in tears, inconsistencies were understandable, and medical findings confirmed that she
was indeed raped. Alibi of Arpon was disregarded. Appealed to CA asserting that TC failed to

consider his minority (13 and 17 years old) as privileged mitigating circumstance.CA still
affirmed RTCs decision.

Issue: Whether or not the accused is entitled to R.A. 9344 and what is the extent of his liability.

Held: The RTC and the Court of Appeals failed to consider in favor of the accused-appellant the
privileged mitigating circumstance of minority.

Pertinently, the first paragraph of Section 7 of Republic Act No. 9344, otherwise known as the
Juvenile Justice and Welfare Act of 2006, provides for the rule on how to determine the age of
a child in conflict with the law,viz:

In the instant case, the accused-appellant testified that he was born on February 23, 1982 and that
he was only 13 years old when the first incident of rape allegedly happened in 1995. Other than
his testimony, no other evidence was presented to prove the date of his birth. However, the
records of this case show neither any objection to the said testimony on the part of the
prosecution, nor any contrary evidence to dispute the same. Thus, the RTC and the Court of
Appeals should have appreciated the accused-appellants minority in ascertaining the appropriate
penalty.

Although the acts of rape in this case were committed before Republic Act No. 9344 took effect
on May 20, 2006, the said law is still applicable given that Section 68 thereof expressly states:

SEC. 68. Children Who Have Been Convicted and are Serving Sentences. Persons who have
been convicted and are serving sentence at the time of the effectivity of this Act, and who were
below the age of eighteen (18) years at the time of the commission of the offense for which they
were convicted and are serving sentence, shall likewise benefit from the retroactive application
of this Act. They shall be entitled to appropriate dispositions provided under this Act and their
sentences shall be adjusted accordingly. They shall be immediately released if they are so
qualified under this Act or other applicable law.

Accordingly, for the first count of rape, which in the information in Criminal Case No. 2000-0146 was allegedly committed in 1995, the testimony of the accused-appellant sufficiently
established that he was only 13 years old at that time. In view of the failure of the prosecution to
prove the exact date and year of the first incident of rape,i.e., whether the same occurred in 1995
or in 1998 as previously discussed, any doubt therein should be resolved in favor of the
accused, it being more beneficial to the latter. The Court, thus, exempts the accused-appellant
from criminal liability for the first count of rape pursuant to the first paragraph of Section 6 of
Republic Act No. 9344. The accused-appellant, nevertheless, remains civilly liable therefor.

For the second and third counts of rape that were committed in the year 1999, the accusedappellant was already 17 years old. We likewise find that in the said instances, the accusedappellant acted with discernment. In Madali v. People, the Court had the occasion to reiterate
that [d]iscernment is that mental capacity of a minor to fully appreciate the consequences of his
unlawful act. Such capacity may be known and should be determined by taking into
consideration all the facts and circumstances afforded by the records in each case. In this case,
the fact that the accused-appellant acted with discernment was satisfactorily established by the
testimony of AAA, which we had already found to be credible. Verily, AAA testified that she at
first did not tell anybody about the sexual assault she suffered at the hands of the accusedappellant because the latter told her that he would kill her mother if she did so. That the accusedappellant had to threaten AAA in an effort to conceal his dastardly acts only proved that he knew
full well that what he did was wrong and that he was aware of the consequences thereof.

63. Llave V. People 488 SCRA 376

Facts: On Sept. 24, 2002, on an errand from her mother, the victim, who was only 7 years old at
that time, proceeded to their house, where the accused waited for her, and accosted her; he
proceeded to sexually abuse her, while the victim cried for help.

A barbecue vendor nearby heard her cries and came to the scene; the accused fled, and the
vendor told the victim to tell her parents what happened.

Together with her parents, the victim went to the police and reported the incident; the vendor also
testified to what he saw during that time.

The medical examiner found no injury on the hymen and perineum, but found scanty yellowish
discharge between the labia minora; there was also fresh abrasion of the perennial skin at 1
oclock position near the anal opening.

The trial court found the victim guilty, declaring that he acted with discernment, but crediting
him with the special mitigating circumstance of minority.

Issue: Whether or not accused had carnal knowledge of the victim, and if yes, whether he acted
with discernment, being a minor of age more than 9 years old but less than 15?

Held:Yes. Penetration, no matter how slight, or the mere introduction of the male organ into the
labia of the pudendum, constitutes carnal knowledge. Hence, even if the penetration is only
slight, the fact that the private complainant felt pains, points to the conclusion that the rape was
consummated.

While it is true that medical examiner did not find any abrasion or laceration in the private
complainants genitalia, such fact does not negate the latters testimony the petitioner had carnal
knowledge of her. The absence of abrasions and lacerations does not disprove sexual abuses,
especially when the victim is a young girl as in this case.

The court have held that when the offended party is young and immature, from the age of
thirteen to sixteen, courts are inclined to give credence to their account of what transpired,
considering not only their relative vulnerability but also the shame and embarrassment to which
they would be exposed if the matter to which they testified is not true.

Discernment is the mental capacity to understand the difference between right and wrong.

The accused, with methodical fashion, dragged the resisting victim behind the pile of hollow
blocks near the vacant house to insure that passers-by would not be able to discover his dastardly
acts.

64. Jose V. People 448 SCRA 316

Facts: On November 14, 1995, P/Supt Joseph Castro received an information from an unnamed
informant that a big time group of drug pushers from Greenhills will deliver 100 grams of shabu
at Chowking. Acting on such report, SPO1 Bonifacio Gueverra was assigned to act as a poseur
buyer. They positioned their cars at the parking area where they had a commanding view of
people going in and out. In the afternoon a Toyota Corolla arrived, Sonny Zarraga was the driver
with Alvin Jose. The unnamed informant approached and talked to Sonny Zarraga. Then, the
informant called SPO1Bonifacio Guevarra and informed the latter that Sonny Zarraga had with
him100 grams of shabu . SPO1 Guevarra offered to buy the shabu. Sonny Zarraga asked SPO1
Bonifacio Guevarra if he had the money. Guevarra said yes. He showed the aforecited bundle of
"money bills." Sonny Zarraga then asked Alvin Jose to bring out the shabu and handover to
Guevarra. SPO1 Guevarra, in turn, handed the bundle of "money bills. Then the other police
approached and introduced themselves as Narcom Operatives. They arrested Sonny Zarraga and
Alvin Jose.

The RTC finds both the accused Sonny Zarraga and Alvin Jose guilty beyond reasonable doubt,
for violation of R.A. 6425.

On appeal to the CA, the CA rendered judgment affirming the decisionappealed from with
modification. The appellate court reduced the penalty imposed on appellant Alvin Jose, on its
finding that he was only thirteen (13) years old when he committed the crime.

Appellant Jose, now the petitioner, filed his petition for review on certiorari, alleging that under
paragraph 3, Article 12 of the Revised Penal Code, a minor over nine (9) and under fifteen (15)
years of age at the time of the commission of the crime is exempt from criminal liability.

Issue: Whether or not Alvin Jose can be exempt from criminal liability underthe mitigating
circumstances of minority.

Held: Yes. Under Article 12(3) of the Revised Penal Code, a minor over nine years of age and
under fifteen is exempt from criminal liability if charged with afelony. The law applies even if
such minor is charged with a crime defined andpenalized by a special penal law. In such case, it

is the burden of the minor toprove his age in order for him to be exempt from criminal liability.
The reason for the exemption is that a minor of such age is presumed lacking the mental element
of a crime.

65. Talampas V. People 661 SCRA 197

Facts: Prosecution witness Jose Sevillo (Jose) who allegedly witnessed the incident in question,
testified that on July 5, 1995 at about 7:00 oclock in the evening, he together with Eduardo
Matic (Eduardo) and Ernesto Matic (Ernesto) were infront of his house, along the road in Zona
Siete (7), Wawa, Malaban, Bian, Laguna, repairing his tricycle when he noticed the appellant
who was riding on a bicycle passed by and stopped. The latter alighted at about three (3) meters
away from him, walked a few steps and brought out a short gun, a revolver, and poked the same
to Eduardo and fired it hitting Eduardo who took refuge behind Ernesto. The appellant again
fired his gun three (3) times, one shot hitting Ernesto at the right portion of his back causing him
(Ernesto) to fall on the ground with his face down. Another shot hit Eduardo on his nape and fell
down on his back (patihaya). Thereafter, the appellant ran away, while he (Jose) and his
neighbors brought the victims to the hospital. On June 6, 1995, Jose executed a Sworn
Statement at the Bian Police Station.

On his part, Talampas interposed self-defense and accident. He insisted that his enemy had been
Eduardo Matic (Eduardo), not victim Ernesto Matic (Ernesto); that Eduardo, who was then with
Ernesto at the time of the incident, had had hit him with a monkey wrench, but he had parried the
blow; that he and Eduardo had then grappled for the monkey wrench; that while they had
grappled, he had notice that Eduardo had held a revolver; that he had thus struggled with
Eduardo for control of the revolver, which had accidentally fired and hit Ernesto during their
struggling with each other; that the revolver had again fired, hitting Eduardo in the thigh; that he
had then seized the revolver and shot Eduardo in the head; and that he had then fled the scene
when people had started swarming around.

On June 22, 2004, the RTC, giving credence to the testimony of eyewitness Jose Sevilla, found
Talampas guilty beyond reasonable doubt of homicide.

Still, the CA affirmed the conviction based on the RTCs factual and legal conclusions, and ruled
that Talampas, having invoked self-defense, had in effect admitted killing Ernesto and had

thereby assumed the burden of proving the elements of self-defense by credible, clear and
convincing evidence, but had miserably failed to discharge his burden.

Issue: Whether or not his guilt was not proven beyond reasonable doubt, and that the lower
courts both erred in rejecting his claim of self-defense and accidental death.

Held: The records eliminate the intervention of accident. Talampas brandished and poked his
revolver at Eduardo and fired it, hitting Eduardo, who quickly rushed to seek refuge behind
Ernesto. At that point, Talampas fired his revolver thrice. One shot hit Ernesto at the right
portion of his back and caused Ernesto to fall face down to the ground. Another shot hit Eduardo
on the nape, causing Eduardo to fall on his back. Certainly, Talampas acts were by no means
lawful, being a criminal assault with his revolver against bothEduardo and Ernesto.

And, thirdly, the fact that the target of Talampas assault was Eduardo, not Ernesto, did not
excuse his hitting and killing of Ernesto. The fatal hitting of Ernesto was the natural and direct
consequence of Talampas felonious deadly assault against Eduardo. Talampas poor aim
amounted to aberratio ictus, or mistake in the blow, a circumstance that neither exempted him
from criminal responsibility nor mitigated his criminal liability. Lo que es causa de la causa, es
causa del mal causado (what is the cause of the cause is the cause of the evil caused). Under
Article 4 of the Revised Penal Code, criminal liability is incurred by any person committing a
felony although the wrongful act done be different from that which he intended.

66. Toledo V. People 439 SCRA 94


Facts: On September 16, 1995, appellant went to a black-smith who made the design of his bolo.
When he went home to Tuburan, Odiongan, Romblon late in the afternoon, appellant saw the
group of Lani Famero, Michael Fosana, Rex Cortez and Ricky Guarte drinking gin at the house
of the Spouses Manuel and Eliza Guarte, Rickys parents. Appellants house is about five (5)
meters away from the house of Spouses Guarte. Appellant requested the group of Ricky to
refrain from making any noise. Thereupon, appellant proceeded inside his house and went to
sleep. Around 9:00 p.m., Gerardo Faminia, Eliza Guartes brother arrived at the Guarte house
and asked for any left-over food. Eliza prepared dinner for him and after Gerardo finished eating,
he went home accompanied by Ricky. Gerardos home is about twelve (12) meters away from
the Guarte home. Minutes later, Ricky came back and together with Lani, Rex and Michael, went
to sleep at the Guarte house. They had not laid down for long when they heard stones being
hurled at the roof of the house. The stoning was made three (3) times. Ricky rose from bed and

peeped through a window. He saw appellant stoning their house. Ricky went out of the house and
proceeded to appellants house. Ricky asked appellant, his uncle, why he was stoning their
house. Appellant did not answer but met Ricky at the doorstep of his (appellants) house and,
without any warning, stabbed Ricky on the abdomen with a bolo. Eliza had followed his son
Ricky and upon seeing that Ricky was stabbed, shouted for help. Lani heard Elizas cry for help
and immediately rushed outside the house. Lani saw Ricky leaning on the ground and supporting
his body with his hands. Lani helped Ricky stand up and brought him to the main road. Lani
asked Ricky who stabbed him and Ricky replied that it was appellant who stabbed him. Then
Docloy Cortez arrived at the scene on board his tricycle. Accordingly, Ricky was put on the
tricycle and taken to the Romblon Provincial Hospital where he died.

The petitioner adduced evidence that at around 5:00 p.m. on September 16, 1995, he was on his
way home at Tuburan, Odiongan, Romblon. He saw his nephew, Ricky Guarte, and the latters
friends, Michael Fosana, Rex Cortez, and Lani Famero, about five meters away from his house,
having a drinking spree. He ordered them not to make loud noises, and they obliged. He then
went to his house, locked the door with a nail, and went to sleep. However, he was awakened at
around 9:30 p.m. by loud noises coming from Ricky and his three companions. He peeped
through the window grills of his house and admonished them not to make any loud noises. Ricky,
who was then already inebriated, was incensed; he pulled out a balisong, pushed the door, and
threatened to stab the petitioner. The petitioner pushed their sala set against the door to block the
entry of Ricky, but the latter continued to push the door open with his hands and body. The
petitioner ran to the upper portion of their house and got his bolo. He returned to the door and
pushed it with all his might using his left hand. He then pointed his bolo, which was in his right
hand, towards Ricky. The bolo accidentally hit Ricky on the stomach, and the latter lost his
balance and fell to the floor. The petitioner, thereafter, surrendered to the barangay captain at
11:00 a.m. on September 17, 1995.

After trial, the court rendered judgment finding the petitioner guilty as charged. The trial court
did not give credence and probative weight to the testimony of the petitioner that his bolo
accidentally hit the victim on the stomach.

The petitioner testified that his bolo hit the victim accidentally. He asserted in the RTC and in the
CA that he is exempt from criminal liability for the death of the victim under Article 12,
paragraph 4 of the Revised Penal Code.

However, the petitioner changed gear, so to speak, and now alleges that he acted in self-defense
when he stabbed the victim. As such, he contends, he is not criminally liable under Article 11,
paragraph 1 of the Revised Penal Code

Issue: Whether or not the accused is entitled to a complete self defense when he accidentally
stabbed the victim in the stomach.

Held: It is an aberration for the petitioner to invoke the two defenses at the same time because
the said defenses are intrinsically antithetical. There is no such defense as accidental self-defense
in the realm of criminal law.
To prove self-defense, the petitioner was burdened to prove the essential elements thereof,
namely: (1) unlawful aggression on the part of the victim; (2) lack of sufficient provocation on
the part of the petitioner; (3) employment by him of reasonable means to prevent or repel the
aggression. Unlawful aggression is a condition sine qua non for the justifying circumstances of
self-defense, whether complete or incomplete. Unlawful aggression presupposes an actual,
sudden, and unexpected attack, or imminent danger thereof, and not merely a threatening or
intimidating attitude. We agree with the ruling of the CA that the petitioner failed to prove selfdefense, whether complete or incomplete.
Appellant was not justified in stabbing Ricky. There was no imminent threat to appellants life
necessitating his assault on Ricky. Unlawful aggression is a condition sine qua non for the
justifying circumstance of self-defense. For unlawful aggression to be appreciated, there must be
an actual, sudden, unexpected attack or imminent danger thereof, not merely a threatening or
intimidating attitude. In the absence of such element, appellants claim of self-defense must fail.

67. People V. Concepcion 386 SCRA 74


Facts: Both Maximo Sison, jr. and Arturo Yarte testified that between 10:00 and 11:00 in the
evening of November 24, 1997, Lorenzo Galang, a resident of their barangay, got involved in a
quarrel at the town plaza. He was brought to the barangay hall for questioning by Barangay
Captain Remigio Capitli.
Shortly after, appellant Rodolfo Concepcion arrived and fired his rifle twice or thrice past the
ears of Lorenzo, who was then sitting, but without injuring him. After that, however, appellant
thrust the barrel of the gun against the abdomen of Lorenzo. Then there was an
explosion. Lorenzo was shot in the thigh. At least three more shots were fired, hitting Lorenzo

in the chest. According to Sison and Yarte, appellant shot Lorenzo deliberately. Lorenzo died
instantly.
In his defense, appellant Rodolfo Concepcion claimed that the shooting was only
accidental. According to him, he was investigating Lorenzo for the latters disorderly behavior
at the town plaza when it happened. He said Lorenzo appeared drunk and unruly, and even
verbally challenged him to fight. At this juncture, according to appellant, he fired two shots in
the air, but Lorenzo grabbed the barrel of his gun. The gun accidentally fired and Lorenzo was
hit.
Defense witnesses Estelita Baluyot and Milagros Villegas corroborated appellants story. They
said they witnessed the incident because they were among the bystanders who saw the event
happen from the time Lorenzo was brought to the barangay hall for investigation until he was
shot.
Estelita and Milagros testified that Lorenzo was seated while being questioned and pacified by
appellant. Appellant was then standing. All of a sudden, according to the lady-witnesses,
appellant fired two warning shots in the air. Lorenzo stood up and grabbed the barrel of the gun
which was then pointed upwards. When it fired, Lorenzo was hit.
On November 10, 1998, the trial court rendered its decision finding appellant guilty of the crime
of murder.
Issue: Whether or not the lower court erred in not finding the injuries sustained by the deceased
self-inflicted while the accused was in the performance of his official duties.

Held: No. Well settled is the rule in criminal cases, that the prosecution has the burden of proof
to establish the guilt of the accused. However, once the defendant admits the commission of the
offense charged, but raises an exempting circumstance as a defense, the burden of proof is
shifted to him. By invoking mere accident as a defense, appellant now has the burden of proving
that he is entitled to that exempting circumstance under Article 12 (4) of the Code.

The existence of accident must be proved by the appellant to the satisfaction of the court. For
this to be properly appreciated in appellants favor, the following requisites must concur: (1) that
the accused was performing a lawful act with due care; (2) that the injury is caused by mere
accident; and (3) that there was no fault or intent on his part to cause the injury. Appellant must
convincingly prove the presence of these elements in order to benefit from the exempting
circumstance of accident. However, his defense utterly failed to discharge this burden. Thus, we
find no reversible error in the judgment of the trial court.

By appellants own testimony, the victim was unarmed. In contrast, appellant had an armalite
and a handgun. It is highly inconceivable that an unarmed man could pose bodily harm to
another who is heavily armed.

We note, that appellants gun discharged several shots that hit vital parts of the victims
body. Was the discharge purely accidental? We dont think so. As observed by the trial court,
recklessly appellant had put his finger on the trigger of his cocked and loaded rifle. In that state,
with the slightest movement of his finger, the rifle would fire readily. And it did not just once
but several fires.

Appellant claims that the victim Lorenzo, who was drunk at the time, was brought to the
barangay hall for investigation. Lorenzo became unruly while being questioned, so appellant
was constrained to fire two warning shots in the air to frighten him. However, the latter stood up
and immediately grabbed the nozzle of the gun and pulled it towards him. The gun accidentally
went off and hit Lorenzo in the body. To buttress his claim, appellant rationalizes that he could
have killed Lorenzo immediately while creating trouble at the plaza, if that was indeed his
intention. Since he did not, appellant posits that there was no intent on his part to kill Lorenzo.

But we note patent inconsistencies in his claims. He testified on query by the trial court that
when he was pacifying the victim, his rifle was hanging on his shoulder on a swivel, with its
barrel pointed to the floor. At that instance, the victim grabbed the barrel of the gun which
accidentally fired. However, on direct examination by his defense counsel, he testified that the
victim grabbed his rifle only after he had fired the two shots in the air.

His claims do not square with and could not overcome the testimony of prosecution witnesses on
this score. Note that Maximo Sison, Jr., an eyewitness, categorically declared that he saw
appellant shoot the victim with an M-16 armalite.

68. Ty V. People 439 SCRA 220

Facts: This case stemmed from the filing of 7 Informations for violation of B.P. 22 against Ty
before the RTC of Manila. The said accused drew and issue to Manila Doctors Hospital to apply
on account or for value to Editha L.Vecino several post-dated checks. The said accused well
knowing that at the time of issue she did not have sufficient funds in or credit with the drawee
bank for payment of such checks in full upon its presentment, which check when presented for
payment within ninety (90) days from the date hereof, was subsequently dishonored by the
drawee bank for Account Closed and despite receipt of notice of such dishonor, said accused
failed to pay said Manila Doctors Hospital the amount of the checks or to make arrangement for
full payment of the same within five (5)banking days after receiving said notice. Ty claimed that
she issued the checks because of an uncontrollable fear of a greater injury. She claims that she
was forced to issue the checks to obtain release of her mother whom the hospital inhumanely and
harshly treated, and would not discharge unless the hospital bills are paid. The trial court
rendered judgment against Ty. Ty interposed an appeal with the CA and reiterated her defense
that she issued the checks under the impulse of an uncontrollable fear of a greater injury or in
avoidance of a greater evil or injury. The appellate court affirmed the judgment of the trial court
with modification. It set aside the penalty of imprisonment and instead sentenced Ty to pay a fine
of sixty thousand pesos P 60,000.00equivalent to double the amount of the check, in each case.

Issue: Whether or not the defense of uncontrollable fear is tenable to warrant her exemption from
criminal liability?

Held: No. Uncontrollable fear for this exempting circumstance to be invoked successfully, the
following requisites must concur: (1) existence of an uncontrollable fear; (2) the fear must be
real and imminent; and (3) the fear of an injury is greater than or at least equal to that committed.
In the instant case, the evil sought to be avoided is merely expected or anticipated.

If the evil sought to be avoided is merely expected or anticipated or may happen in the future,
this defense is not applicable It must appear that the threat that caused the uncontrollable fear is
of such gravity and imminence that the ordinary man would have succumbed to it. It should be
based on a real, imminent or reasonable fear for ones life or limb. A mere threat of a future
injury is not enough. It should not be speculative, fanciful, or remote. A person invoking
uncontrollable fear must show therefore that the compulsion was such that it reduced him to a
mere instrument acting not only without will but against his will as well. It must be of such
character as to leave no opportunity to the accused for escape.

The fear harbored by Ty was not real and imminent. Ty claims that she was compelled to issue
the checks, a condition the hospital allegedly demanded of her before her mother could be
discharged, for fear that her mothers health might deteriorate further due to the inhumane
treatment of the hospital or worse, her mother might commit suicide. This is speculative fear; it
is not the uncontrollable fear contemplated by law.

69. People V. Dequina 640 SCRA 111

FACTS:Accused-appellants Dequina et al were charged before the RTC-Manila with violations


of the offense of illegal transport of marijuana under the Dangerous Drugs Act of 1972.

Police Officer III Wilfredo Masanggue testified that at about 6:00 a.m., of September 29,1999, he
and SPO1 Anthony Blanco were instructed by their superior to proceed at the corner of Juan
Luna and Raxabago Sts., Tondo, Manila, where, according to the report given by the informant,
three persons a male and two female[s] would be coming from Baguio City to deliver unknown
quantity of marijuana.

At around 9:00 a.m., they noticed a taxi cab coming from Yuseco St. heading towards the
direction of the pier. From it emerged three passengers a man and two women each one of them
carrying a black travelling bag.

As the trio started walking towards the western portion of Raxabago St., they drove and trailed
them. As the patrol car got closer behind them, [Dequina] noticed its presence. She started
walking in a more hurried pace.

While trying to get away, [Dequina] dropped the bag she was carrying. As a result, the zipper of
the bag gave way. Bundles of dried leaves wrapped in transparent plastic bags case into view.
Thus, they arrested the three accused.

During the investigation, it was discovered that each of the three black travelling bags
confiscated from the three accused contained eleven bricks of marijuana.

Accused-appellants assail their conviction, asserting that their arrests were illegal. They were not
doing anything illegal that would have justified their warrantless arrest, much less a warrantless
search of their persons and belongings. A search made without a warrant cannot be justified as an
incident of arrest unless the arrest itself was lawful.

In order to exonerate herself from criminal liability, Dequina contends that she transported the
marijuana under the compulsion of an irresistible fear. Jundoc and Jingabo, on the other hand,
claim that they went along to accommodate Dequina, a trusted childhood friend.

Issue: Whether or not the accused should be acquitted based on a compulsion of an irresistible
fear.

Held: A person who acts under the compulsion of an irresistible force, like one who acts under
the impulse of an uncontrollable fear of equal or greater injury, is exempt from criminal liability
because he does not act with freedom. Actus me invito factus non est meus actus. An act done
by me against my will is not my act. The force contemplated must be so formidable as to reduce
the actor to a mere instrument who acts not only without will but against his will. The duress,
force, fear or intimidation must be present, imminent and impending, and of such nature as to
induce a well-grounded apprehension of death or serious bodily harm if the act be done. A threat
of future injury is not enough. The compulsion must be of such a character as to leave no
opportunity for the accused for escape or self-defense in equal combat. Here, Dequinas version
of events that culminated with her and Jundoc and Jingabos arrests on September 29, 1999 is
implausible. Equally far-fetched is Jundoc and Jingabos assertion of blind trust in Dequina and
total ignorance of the transportation of marijuana.

69.People v Dansico 644 SCRA 151


Facts: Sometime on September 7, 1998 at about 4:30 o'clock in the afternoon, at Brgy. MayAnao, Tigaon, Camarines Sur, Romeo Dansico a.k.a. Lamyak and Augusto Cuadra conspired
and confederated with one another to attain a common purpose which is to feloniously without
authority of law to sell and deliver one piece of marijuana brick wrapped in newspaper
weighing approximately (900) grams for and in consideration of five thousand pesos.
Issue: Were the accused instigated to sell marijuana?

Held:No. Instigation means luring the accused into a crime that he, otherwise, had no intention to
commit, in order to prosecute him. On the other hand, entrapment is the employment of ways
and means in order to trap or capture a lawbreaker.
Instigation presupposes that the criminal intent to commit an offense originated from the inducer
and not the accused who had no intention to commit the crime and would not have committed it
were it not for the initiatives by the inducer. In entrapment, the criminal intent or design to
commit the offense charged originates in the mind of the accused; the law enforcement officials
merely facilitate the apprehension of the criminal by employing ruses and schemes.
In instigation, the law enforcers act as active co-principals. Instigation leads to the acquittal of
the accused, while entrapment does not bar prosecution and conviction.

70. People v. Sta. Maria 516 SCRA 621

Facts: On November 27, 2002, at around 10:00 oclock in the morning, P/Chief Insp. Noli
Pacheco, Chief of the Provincial Drug Enforcement Group of the Bulacan Provincial Office
based at Camp Alejo Santos, Malolos, Bulacanreceived an intelligence report about the illegal
drug activities in Sitio Gulod, Barangay Pantubig, San Rafael, Bulacan of a certain "Fael," who
later turned outto be appellant Rafael Sta. Maria. P/Chief Insp. Pacheco formed a surveillance
team to look for a police asset to negotiate a drug deal with appellant. In the morning of
November 29, 2002, the surveillance team reported to P/Chief Insp. Pacheco that a confidential
asset found by the team had already negotiated a drug deal for the purchase of P200 worth of
shabu from appellant at the latters house at No. 123 Sitio Gulod, Barangay Pantubig, San
Rafael, Bulacan between7:00 and 7:30 in the evening of November 29, 2002. The surveillance
team then prepared for a buy-bust operation, with PO3 Enrique Rullan as team leader, andPO1
Rhoel Ventura, who was provided with two (2) marked P100-bills, as poseur-buyer. At the
appointed time and place, PO1 Ventura and the confidential informant proceeded to appellants
house and knocked at the door. Appellant opened the door and the confidential informant
introduced to him PO1 Ventura as a prospective buyer. PO1 Ventura later handed the two (2)
marked P100-billsto appellant who, in turn, gave him a plastic sachet of shabu. Thereupon,
PO1Ventura sparked his cigarette lighter, which was the pre-arranged signal to the other
members of the buy-bust team that the sale was consummated. Appellant was arrested and the
two marked P100-bills recovered from him. Also arrested on that occasion was one Zedric dela
Cruz who was allegedly sniffing shabu inside appellants house and from whom drug
paraphernalia were recovered. Upon laboratory examination of the item bought from appellant,
the same yielded positive for methylampetamine hydrochloride or shabu weighing 0.041 gram.
The accused was charged of violation of Section 5, Article II of R.A. No.9165, otherwise known
as the Comprehensive Dangerous Drugs Act of 2002.

The trial court found appellant guilty beyond reasonable doubt of the offense charged. The Court
of Appeals promulgated the assailed decision denying the appeal.

Issue: Whether or not instigation was the act which preceded Sta. Marias arrest?

Held: In entrapment, the entrapper resorts to ways and means to trap and capture a lawbreaker
while executing his criminal plan. In instigation, the instigator practically induces the would-bedefendant into committing the offense, and himself becomes a co-principal. In entrapment, the
means originates from the mind of the criminal. The idea and the resolve to commit the crime
come from him. In instigation, the law enforcer conceives the commission of the crime and
suggests to the accused who adopts the idea and carries it into execution. The legal effects of
entrapment do not exempt the criminal from liability. Instigation does. It is no defense to the
perpetrator of a crime that

facilities for its commission were purposely placed in his way, or that the criminal act was done
at the "decoy solicitation" of persons seeking to expose the criminal, or that detectives feigning
complicity in the act were present and apparently assisting its commission. Especially is this true
in that class of cases where the offense is one habitually committed, and the solicitation merely
furnishes evidence of a course of conduct. The solicitation of drugs from appellant by the
informant utilized by the police merely furnishes evidence of the course of conduct. The police
received an intelligence report that appellant has been habitually dealing in illegal drugs. They
duly acted on it by utilizing an informant to effect a drug transaction with the appellant. There
was no showing that the informant induced appellant to sell illegal drugs to him

71. Chang V. People 496 SCRA 321

Facts: Chang was the Municipal Treasurer of Makati who was tasked to examine or investigate
tax returns of private corporations and companies operating within Makati and determine the
sufficiency and insufficiency of the income tax assessed on them and collect payments, San
Mateo was the Chief Operations, Business revenue Examination, Audit division, Makati
Treasurers office.

The examiners found that Group Developers, Inc. (GDI) incurred a tax deficiency inclusive of
penalty in the amount of P494,601.11. The assessment notice was received by Mario Magat,
Chief Operating Officer of GDI. Magat was later able to talk to San Mateo via phone. On May
15, 1991, Magat and San Mateo met for lunch at the Makati Sports Club. Chang later joined the
two, the three agreed that if GDI could pay P125,000 by the end of May 1991, the assessment
would be resolved.

On June 6, 1991, Magat met again for lunch with San Mateo and Chang at the Makati Sports
Club. Magat tried to convince the two that GDI wanted to pay the correct amount of tax to the
municipality. He was advised by San Mateo and Chang, however, that GDI had only two options:
Pay the P494,601.11 to the municipality or P125,000 to them.

On June 12, 1991, Magat met with the NBI Deputy Director Epimaco Velasco who advised him
to file a complaint with the NBI. Magat thus gave a sworn statement. After several days, Magat
contacted San Mateo and asked him if their position was still the same to which the latter said
yes. Magat thereafter told San Mateo that he would deliver the P125,000 on June 19,1991 at the
Makati Sports Club.

On June 19, 1991, Magat informed the NBI that payment was to be made that day around
lunchtime. The NBI formed a team to conduct an entrapment. The genuine money as well as the
boodle money and the envelope where the money was placed were then laced with fluorescent
powder.

San Mateo arrived and joined Magat at his table. Chang arrived and joined the two. Magat told
Chang and San Mateo that GDI was ready to pay and asked them if they could give him the
Certificate of Examination showing that GDI has no more tax liability. Chang handed the
Certificate while Magat gave the brown envelope. At that instant, the NBI agents announced that
they were being arrested.

The Sandiganbayan convicted San Mateo and Chang of violation of sec 3(b) of R.A. No. 3019,
otherwise known as Anti- Graft and Corrupt Practices Act. Hence, this appeal.

Issue: Whether or not there was a valid entrapment operation?

Decision: Petitioners were undisputedly public officers at the time of the commission of the
offense. The prosecution, not only established creditably how the offense charged was
committed. It is established just as creditably how petitioners conspired to commit the crime.

There is entrapment when law officers employ ruses and schemes to ensure the apprehension of
the criminal while in the actual commission of the crime. There is instigation when the accused is
induced to commit the crime. The difference in the nature of the two lies in the origin of the
criminal intent. In entrapment, the mens reoriginates from the mind of the criminal. The idea and
the resolve to commit the crime comes from him. In instigation, the law officer conceives the
commission of the crime and suggests to the accused who adopts the idea and carries it into
execution.

From the evidence of the prosecution, it was clearly established that the criminal intent
originated from the mind of the petitioner. Even before the June 19, 1991 meeting took place,
petitioners already made known to Magat that GDI only had two options to prevent the closure
of the company, either to pay the assessed amount of P494,601.11 to the Municipality or to pay
the amount of P125,000 to them.
72. People v. CA and Tangan 352 SCRA 599

Facts: On December 1, 1984, Navy Captain Eladio C. Tangan was driving alone on Roxas
Boulevard heading south and Generoso Miranda was driving his car in the same direction with
his uncle, Manuel Miranda. Generoso was moving ahead of Tangan. Suddenly, firecrackers were
thrown in Generoso's way, causing him to swerve to the right and cut Tangan's path. Tangan blew
his horn several times. Generoso, slowed down to let Tangan pass. Tangan accelerated and
overtook Generoso, but when he got in front, Tangan reduced speed. Generoso tried four or five
times to overtake on the right lane but Tangan kept blocking his lane. When Tangan slowed down
to make a U-turn, Generoso passed him, pulled over and got out of the car with his uncle. Tangan
also stopped his car and got out. Generoso and Tangan then exchanged expletives. Then Tangan
went to his car and got his .38 caliber handgun on the front seat.

According to the prosecution witnesses, Mary Ann Borromeo, Rosalia Cruz and Manuel
Miranda, the accused pointed his gun at Generoso Miranda and when Manuel Miranda tried to
intervene, the accused pointed his gun at Manuel Miranda, and after that the accused pointed

again the gun to Generoso Miranda, the accused shot Generoso Miranda at a distance of about a
meter. The shot hit the stomach of Generoso Miranda causing the latter to fall. Manuel Miranda
grappled for the possession of the gun and during their grappling, Rosalia Cruz intervened and
took hold of the gun and after Rosalia Cruz has taken hold of the gun, a man wearing a red Tshirt took the gun from her. The man in T-shirt was chased by Manuel Miranda who was able to
get the gun where the man in red T-shirt placed it.

On the other hand, the defense, particularly the accused and his witness by the name of Nelson
Pante claimed that after the gun was taken by the accused from inside his car, the Mirandas
started to grapple for possession of the gun and during the grappling, and while the two Mirandas
were trying to wrest away the gun from the accused, they fell down at the back of the car of the
accused. The accused lost the possession of the gun after falling at the back of his car and as
soon as they hit the ground, the gun fell, and it exploded hitting Generoso Miranda.

Tangan ran away while Generoso lay on the ground bloodied. Manuel looked for the gun and ran
after Tangan. Tangan found a policeman who allowed him to enter his patrol car. Manuel arrived
and told the policeman that Tangan had just shot his nephew. Manuel went back to where
Generoso lay and there found two ladies, Mary Ann Borromeo and Rosalina Cruz, helping his
nephew board a taxi. Manuel suggested that Generoso be brought to the hospital in his car. He
was rushed to the Philippine General Hospital but he expired on the way.

Tangan was charged with the crime of murder with the use of an unlicensed firearm. However,
the information was amended to homicide with the use of a licensed firearm, and he was
separately charged with illegal possession of unlicensed firearm. Tangan entered a plea of not
guilty in the homicide case, but moved to quash the information for illegal possession of
unlicensed firearm on various grounds. The motion to quash was denied, whereupon he filed a
petition for certiorari with this Court. On November 5, 1987, said petition was dismissed and the
joint trial of the two cases was ordered.

After trial, the lower court acquitted Tangan of illegal possession of firearm, but convicted him
of homicide. The privileged mitigating circumstance of incomplete self-defense and the ordinary
mitigating circumstances of sufficient provocation on the part of the offended party and of
passion and obfuscation were appreciated in his favor; Tangan was released from detention after
the promulgation of judgment and was allowed bail in the homicide case.

Tangan appealed to the Court of Appeals, which affirmed the judgment of the trial court but
increased the award of civil indemnity to P50,000.00. His subsequent motion for reconsideration
and a motion to cite the Solicitor General in contempt were denied by the Court of Appeals.

The Solicitor General, on behalf of the prosecution, alleging grave abuse of discretion, filed a
petition for certiorari under Rule 65, naming as respondents the Court of Appeals and Tangan,
where it prayed that the appellate court's judgment be modified by convicting accused-appellant
of homicide without appreciating in his favor any mitigating circumstance.

Issue: Whether or not Tangan acted in incomplete self-defense?

Decision: Incomplete self-defense is not considered as a justifying act, but merely a mitigating
circumstance; hence, the burden of proving the crime charged in the information is not shifted to
the accused. In order that it may be successfully appreciated, however, it is necessary that a
majority of the requirements of self-defense be present, particularly the requisite of unlawful
aggression on the part of the victim. Unlawful aggression by itself or in combination with either
of the other two requisite suffices to establish incomplete self-defense. Absent the unlawful
aggression, there can never be self-defense, complete or incomplete, because if there is nothing
to prevent or repel, the other two requisites of defense will have no basis.

The element of unlawful aggression in self-defense must not come from the person defending
himself but from the victim.

A mere threatening or intimidating attitude is not sufficient. The exchange of insulting words and
invectives between Tangan and Generoso Miranda, no matter how objectionable, could not be
considered as unlawful aggression, except when coupled with physical assault. There being no
lawful aggression on the part of either antagonists, the claim of incomplete self-defense falls.
73. People v. Callet 382 SCRA 43

Facts: Elbert S. Callet was charged and found guilty of the crime of Murder in the death of
Alfredo Senador. Callet used a 9-inch hunting knife in stabbing the latter on the left shoulder
near the base of the neck causing Senadors death shortly thereafter. Callet appealed his

conviction claiming that the Regional Trial Court of Negros Oriental, Dumaguete City (Branch
30) gravely erred in failing to consider the mitigating circumstance of the fact that he had no
intention to commit so grave a wrong therefore his liability should be mitigated.

Issue: Whether or not the criminal liability of Callet be mitigated in that he had no intention to
commit so grave a wrong?

Decision: The Supreme Court ruled in the negative. The lack of intent to commit a wrong so
grave is an internal state. It is weighed based on the weapon used, the part of the body injured,
the injury inflicted and the manner it is inflicted. The fact that the accused used a 9-inch hunting
knife in attacking the victim from behind, without giving him an opportunity to defend himself,
clearly shows that he intended to do what he actually did, and he must be held responsible
therefore, without the benefit of this mitigating circumstance.

74. People vs Sales 658 SCRA 367

Facts: On September 19, 2002, brothers Noemar and Junior, then nine and eight years old,
respectively, left their home to attend the fluvial procession of Our Lady of Peafrancia without
the permission of their parents. They did not return home that night. When their mother, Maria
Litan Sales (Maria), looked for them the next day, she found them in the nearby Barangay of
Magsaysay. Afraid of their fathers rage, Noemar and Junior initially refused to return home but
their mother prevailed upon them. When the two kids reached home a furious appellant
confronted them. Appellant then whipped them with a stick which was later broken so that he
brought his kids outside their house. With Noemars and Juniors hands and feet tied to a
coconut tree, appellant continued beating them with a thick piece of wood.

When the beating finally stopped, the three walked back to the house, Noemar collapsed and lost
consciousness. Maria then told appellant to call a quack doctor. He left and returned with one,
who told them that they have to bring Noemar to a hospital. Appellant thus proceeded to take
the unconscious Noemar to the junction and waited for a vehicle to take them to a hospital. As
there was no vehicle and because another quack doctor they met at the junction told them that
Noemar is already dead, appellant brought his son back to their house.

Appellant denied that his son died from his beating since no parent could kill his or her child. He
claimed that Noemar died as a result of difficulty in breathing. In fact, he never complained of
the whipping done to him. Besides, appellant recalled that Noemar was brought to a hospital
more than a year before September 2002 and diagnosed with having a weak heart.

On the other hand, Maria testified that Noemar suffered from epilepsy. Whenever he suffers from
epileptic seizures, Noemar froths and passes out. But he would regain consciousness after 15
minutes. His seizures normally occur whenever he gets hungry or when scolded.

The trial court charged the accused guilty of parricide and slight physical injuries.

Issue: Whether or not the accused is guilty of the crimes charged.

Held:Yes. All the elements of the crime of parricide is present in this case.

Parricide is committed when: (1) a person is killed; (2) the deceased is killed by the accused; (3)
the deceased is the father, mother, or child, whether legitimate or illegitimate, or a legitimate
other ascendant or other descendant, or the legitimate spouse of accused.

In the case at bar, there is overwhelming evidence to prove the first element, that is, a person was
killed. There is likewise no doubt as to the existence of the second element that the appellant
killed the deceased. It is sufficiently established by the positive testimonies of Maria and
Junior. As to the third element, appellant himself admitted that the deceased is his child.

With regard to charge of Physical injuries, the victim himself, Junior testified that he, together
with his brother Noemar, were beaten by their father, herein appellant, while they were tied to a
coconut tree. He recalled to have been hit on his right eye and right leg and to have been
examined by a physician thereafter. Maria corroborated her sons testimony.
75. Romera vs. People 434 SCRA 467

Facts: Petitioner stabbed the victim in the stomach as a result of an altercation. After the
stabbing, petitioner surrendered to the CAFGU. He was then charged with frustrated murder with
the regional trial court, where he alleged that it was the victim who hacked the walls of his house
with a bolo and thrust the same at him, after which, they grappled with the bolo where the
petitioner then stabbed the victim. He was then convicted, but he was credited with the
mitigating circumstance of voluntary surrender in the imposition of his sentence. The Court of
Appeals then affirmed the trial courts judgment.
Issue: Whether or not the mitigating circumstances of provocation and passion and obfuscation
are present in this case.
Held: Yes. The Court held that thrusting his bolo at petitioner, threatening to kill him, and
hacking the bamboo walls of his house are sufficient provocation to enrage any man, or stir his
rage and obfuscate his thinking, more so when the lives of his wife and children are in danger.
Petitioner stabbed the victim as a result of those provocations, and while petitioner was still in a
fit of rage.
But, it is stressed that provocation and passion or obfuscation are not two separate mitigating
circumstances. Well-settled is the rule that if these two circumstances are based on the same
facts, they should be treated together as one mitigating circumstance. From the facts established
in this case, it is clear that both circumstances arose from the same set of facts aforementioned.
Hence, they should not be treated as two separate mitigating circumstances.
Nonetheless, since the mitigating circumstance of voluntary surrender is also present, Article 64
(5) of the Revised Penal Code should be applied. The penalty for frustrated homicide, pursuant to
Article 50[8] of the Revised Penal Code, is the penalty next lower in degree than that prescribed
by law for consummated homicide. The penalty for consummated homicide is reclusion
temporal, hence the penalty next lower in degree is prision mayor. There being two mitigating
circumstances and no aggravating circumstance, pursuant to Article 64 (5) of the Revised Penal
Code, the next lower penalty, prision correccional, is the next statutory penalty. But following the
Indeterminate Sentence Law herein applicable, the minimum term of the penalty that should be
imposed on petitioner for frustrated homicide should be within the range of arresto mayor in any
of its periods or from one (1) month and one (1) day to six (6) months, while the maximum term
should be within the range of prision correccional in its medium period or two (2) years, four (4)
months and one (1) day to four (4) years and two (2) months.

76. People v. Torpio 431 SCRA 9

Facts: In the evening of October 11, 1997, Anthony went to the house of Dennis and invited the
latter for a drinking spree. Afterwards both left the house of Dennis and went to a nearby store
and started drinking with a companion named Porboy Perez. The three proceeded to Shoreline.
In a cottage, Anthony tried to let Dennis drink gin and as the latter refused, Anthony bathed
Dennis with gin and mauled him several times. Dennis crawled beneath the table and Anthony
tried to stab him with a 22 fan knife but did not hit him. Dennis got up and ran towards their
home.

Upon reaching home, he got a knife. Alarmed by the action of Dennis, his mother shouted.
Manuel, his father, tried to scold his son and confiscate from him the knife but failed to do so,
resulting to Manuels incurring a wound in his hand. He went back to the cottage. Upon seeing
Dennis, Anthony ran towards the creek but Dennis blocked him and stabbed him. When he was
hit, Anthony ran but got entangled with fishing net and fell on his back. Dennis then mounted on
him and continued stabbing him resulting to the latters death. After stabbing, Dennis left and
went to Camp Downes and slept there. The next morning, Dennis voluntarily surrendered
himself to Boy Estrera, a well-known police officer.

The trial court rendered a judgment convicting Dennis for the crime of Murder qualified by
treachery or evident premeditation and appreciating three mitigating circumstances. His father
Manuel was acquitted. Not satisfied with the judgment, Dennis appealed his case.

Issue: Whether or not the mitigating circumstance of having acted in the immediate vindication
of a grave offense is appreciated?

Decision: The Supreme held that the mitigating circumstance of having acted in the immediate
vindication of a grave offense was properly appreciated. Dennis was humiliated, mauled and
almost stabbed by the Anthony. Although the unlawful aggression had ceased when Dennis
stabbed Anthony, it was nonetheless a grave offense for which the Dennis may be given the
benefit of a mitigating circumstance.

However, the mitigating circumstance of sufficient provocation cannot be considered apart from
the circumstance of vindication of a grave offense. These two circumstances arose from one and
the same incident, i.e., the attack on the
appellant by Anthony, so that they should be considered as only one mitigating circumstance.

77. People v. Torpio 431 SCRA 9

Facts: In the evening of October 11, 1997, Anthony went to the house of Dennis and invited the
latter for a drinking spree. Afterwards both left the house of Dennis and went to a nearby store
and started drinking with a companion named Porboy Perez. The three proceeded to Shoreline.
In a cottage, Anthony tried to let Dennis drink gin and as the latter refused, Anthony bathed
Dennis with gin and mauled him several times. Dennis crawled beneath the table and Anthony
tried to stab him with a 22 fan knife but did not hit him. Dennis got up and ran towards their
home.

Upon reaching home, he got a knife. Alarmed by the action of Dennis, his mother shouted.
Manuel, his father, tried to scold his son and confiscate from him the knife but failed to do so,
resulting to Manuels incurring a wound in his hand. He went back to the cottage. Upon seeing
Dennis, Anthony ran towards the creek but Dennis blocked him and stabbed him. When he was
hit, Anthony ran but got entangled with fishing net and fell on his back. Dennis then mounted on
him and continued stabbing him resulting to the latters death. After stabbing, Dennis left and
went to Camp Downes and slept there. The next morning, Dennis voluntarily surrendered
himself to Boy Estrera, a well-known police officer.

The trial court rendered a judgment convicting Dennis for the crime of Murder qualified by
treachery or evident premeditation and appreciating three mitigating circumstances. His father
Manuel was acquitted. Not satisfied with the judgment, Dennis appealed his case.

Issue: Whether or not the mitigating circumstance of having acted in the immediate vindication
of a grave offense is appreciated?

Decision: The Supreme held that the mitigating circumstance of having acted in the immediate
vindication of a grave offense was properly appreciated. Dennis was humiliated, mauled and
almost stabbed by the Anthony. Although the unlawful aggression had ceased when Dennis
stabbed Anthony, it was nonetheless a grave offense for which the Dennis may be given the
benefit of a mitigating circumstance.

However, the mitigating circumstance of sufficient provocation cannot be considered apart from
the circumstance of vindication of a grave offense. These two circumstances arose from one and
the same incident, i.e., the attack on the
appellant by Anthony, so that they should be considered as only one mitigating circumstance.
78. People V. Malejana 479 SCRA 610
Facts: As to the events that transpired on July 28, 1990, Madrid narrated that around 7:15 p.m.,
while he was seated in front of his jeep parked at the side of the road at Marisfoque, Pilar,
Sorsogon in the company of Roces, Sy, Andrade, Bernarda Sy, Jose Belmonte and Ernesto
Francisco, he saw appellant at about 30 meters away heading towards their direction. Upon
reaching their group, appellant asked them where Roces was. When he noticed Roces who was
sitting at a distance of 1 meter beside Madrid, appellant brandished an armalite rifle and fired a
shot into the air. Then he pointed the barrel of the gun at Roces and fired five (5) times, hitting
Roces thrice. After the victim fell to the ground, appellant left the scene of the incident and went
to his house about 150 meters away. Madrid and his other companions tried to assist Roces but
discovered that the latter was already dead, presumably from the gunshot wounds that were
inflicted upon him by appellant.3
The statement of Madrid was corroborated by the testimonies of Antonio Sy and Samuel
Andrade who both confirmed that, on the day in question, appellant approached their group
looking for Roces and after locating him, fired his armalite rifle once in the air and then at least
five times at the victim.
As to the events that transpired on July 28, 1990, Madrid narrated that around 7:15 p.m., while
he was seated in front of his jeep parked at the side of the road at Marisfoque, Pilar, Sorsogon in
the company of Roces, Sy, Andrade, Bernarda Sy, Jose Belmonte and Ernesto Francisco, he saw
appellant at about 30 meters away heading towards their direction. Upon reaching their group,
appellant asked them where Roces was. When he noticed Roces who was sitting at a distance of
1 meter beside Madrid, appellant brandished an armalite rifle and fired a shot into the air. Then
he pointed the barrel of the gun at Roces and fired five (5) times, hitting Roces thrice. After the
victim fell to the ground, appellant left the scene of the incident and went to his house about 150
meters away. Madrid and his other companions tried to assist Roces but discovered that the latter
was already dead, presumably from the gunshot wounds that were inflicted upon him by
appellant.
The statement of Madrid was corroborated by the testimonies of Antonio Sy and Samuel
Andrade who both confirmed that, on the day in question, appellant approached their group
looking for Roces and after locating him, fired his armalite rifle once in the air and then at least
five times at the victim.

Court finds accused Floro Malejana guilty beyond doubt of Murder defined and penalized under
Article 248 Revised Penal Code

Issue: Whether or not passion or obfuscation may be appreciated in favor of the accused.
Held: No. In this case, appellant admits that on the night in question, he approached the victim
and the latter allegedly attempted to strike him. In response thereto, appellant allegedly fired a
warning shot in the air then left after reprimanding the group. However, this defense cannot
prevail over the positive identification of appellant as the author of the crime by no less than
three credible witnesses.
Passion and obfuscation similarly cannot be appreciated in favor of appellant. To be entitled to
this mitigating circumstance, the following elements must be present: (1) There should be an act
both unlawful and sufficient to produce such condition of mind; (2) the act that produced the
obfuscation was not far removed from the commission of the crime by a considerable length of
time, during which the perpetrator might recover his normal equanimity. The bare assertion that
the victim and appellant had an argument does not provide justifiable basis for applying to him
this mitigating circumstance. The cause that produced the passion and obfuscation has not been
established nor proven by clear and convincing evidence. The defense advances mere
speculations and conjectures to gloss over the fact that there is lack of proof of the cause. Courts
are not permitted to render judgments upon guesses or surmises. Suspicion, it has been said,
cannot give probative force to testimony which in itself is insufficient to establish or justify an
inference of a particular fact.
79. PEOPLE VS. GENOSA
FACTS: This case stemmed from the killing of Ben Genosa, by his wife Marivic
Genosa, appellant herein. During their first year of marriage, Marivic and Ben
lived happily but apparently thereafter, Ben changed and the couple would always
quarrel and sometimes their quarrels became violent. Appellant testified that
every time her husband came home drunk, he would provoke her and sometimes
beat her. Whenever beaten by her husband, she consulted medical doctors who
testified during the trial. On the night of the killing, appellant and the victim were
quarreled and the victim beat the appellant. However, appellant was able to run to
another room. Appellant admitted having killed the victim with the use of a gun.
The information for parricide against appellant, however, alleged that the cause of
death of the victim was by beating through the use of a lead pipe. Appellant
invoked self defense and defense of her unborn child. After trial, the Regional
Trial Court found appellant guilty beyond reasonable doubt of the crime of
parricide with an aggravating circumstance of treachery and imposed the penalty
of
death.

On automatic review before the Supreme Court, appellant filed an URGENT


OMNIBUS MOTION praying that the Honorable Court allow (1) the exhumation
of Ben Genosa and the re-examination of the cause of his death; (2) the
examination of Marivic Genosa by qualified psychologists and psychiatrists to
determine her state of mind at the time she killed her husband; and finally, (3) the
inclusion of the said experts reports in the records of the case for purposes of the
automatic review or, in the alternative, a partial re-opening of the case a quo to
take the testimony of said psychologists and psychiatrists. The Supreme Court
partly granted the URGENT OMNIBUS MOTION of the appellant. It remanded
the case to the trial court for reception of expert psychological and/or psychiatric
opinion on the battered woman syndrome plea. Testimonies of two expert
witnesses on the battered woman syndrome, Dra. Dayan and Dr. Pajarillo, were
presented and admitted by the trial court and subsequently submitted to the
Supreme
Court
as
part
of
the
records.
ISSUE:
1. Whether or not appellant herein can validly invoke the battered woman
syndrome
as
constituting
self
defense.
2. Whether or not treachery attended the killing of Ben Genosa.
Ruling: 1. The Court ruled in the negative as appellant failed to prove that she is
afflicted
with
the
battered
woman
syndrome.
A battered woman has been defined as a woman who is repeatedly subjected to
any forceful physical or psychological behavior by a man in order to coerce her to
do something he wants her to do without concern for her rights. Battered women
include wives or women in any form of intimate relationship with men.
Furthermore, in order to be classified as a battered woman, the couple must go
through the battering cycle at least twice. Any woman may find herself in an
abusive relationship with a man once. If it occurs a second time, and she remains
in
the
situation,
she
is
defined
as
a
battered
woman.
More graphically, the battered woman syndrome is characterized by the so-called
cycle of violence, which has three phases: (1) the tension-building phase; (2)
the acute battering incident; and (3) the tranquil, loving (or, at least, nonviolent)
phase.
The Court, however, is not discounting the possibility of self-defense arising from
the battered woman syndrome. First, each of the phases of the cycle of violence
must be proven to have characterized at least two battering episodes between the

appellant and her intimate partner. Second, the final acute battering episode
preceding the killing of the batterer must have produced in the battered persons
mind an actual fear of an imminent harm from her batterer and an honest belief
that she needed to use force in order to save her life. Third, at the time of the
killing, the batterer must have posed probable -- not necessarily immediate and
actual -- grave harm to the accused, based on the history of violence perpetrated
by the former against the latter. Taken altogether, these circumstances could
satisfy the requisites of self-defense. Under the existing facts of the present case,
however,
not
all
of
these
elements
were
duly
established.
The defense fell short of proving all three phases of the cycle of violence
supposedly characterizing the relationship of Ben and Marivic Genosa. No doubt
there were acute battering incidents but appellant failed to prove that in at least
another battering episode in the past, she had gone through a similar pattern.
Neither did appellant proffer sufficient evidence in regard to the third phase of the
cycle.
In any event, the existence of the syndrome in a relationship does not in itself
establish the legal right of the woman to kill her abusive partner. Evidence must
still be considered in the context of self-defense. Settled in our jurisprudence, is
the rule that the one who resorts to self-defense must face a real threat on ones
life; and the peril sought to be avoided must be imminent and actual, not merely
imaginary. Thus, the Revised Penal Code provides that the following requisites of
self-defense must concur: (1) Unlawful aggression; (2) Reasonable necessity of
the means employed to prevent or repel it; and (3) Lack of sufficient provocation
on
the
part
of
the
person
defending
himself.
Unlawful aggression is the most essential element of self-defense. It presupposes
actual, sudden and unexpected attack -- or an imminent danger thereof -- on the
life or safety of a person. In the present case, however, according to the testimony
of Marivic herself, there was a sufficient time interval between the unlawful
aggression of Ben and her fatal attack upon him. She had already been able to
withdraw from his violent behavior and escape to their childrens bedroom.
During that time, he apparently ceased his attack and went to bed. The reality or
even the imminence of the danger he posed had ended altogether. He was no
longer in a position that presented an actual threat on her life or safety.
The mitigating factors of psychological paralysis and passion and obfuscation
were, however, taken in favor of appellant. It should be clarified that these two
circumstances -- psychological paralysis as well as passion and obfuscation -- did
not
arise
from
the
same
set
of
facts.

The first circumstance arose from the cyclical nature and the severity of the
battery inflicted by the batterer-spouse upon appellant. That is, the repeated
beatings over a period of time resulted in her psychological paralysis, which was
analogous to an illness diminishing the exercise of her will power without
depriving
her
of
consciousness
of
her
acts.
As to the extenuating circumstance of having acted upon an impulse so powerful
as to have naturally produced passion and obfuscation, it has been held that this
state of mind is present when a crime is committed as a result of an uncontrollable
burst of passion provoked by prior unjust or improper acts or by a legitimate
stimulus so powerful as to overcome reason. To appreciate this circumstance, the
following requisites should concur: (1) there is an act, both unlawful and
sufficient to produce such a condition of mind; and (2) this act is not far removed
from the commission of the crime by a considerable length of time, during which
the
accused
might
recover
her
normal
equanimity.
2. NO. Because of the gravity of the resulting offense, treachery must be proved
as conclusively as the killing itself. Besides, equally axiomatic is the rule that
when a killing is preceded by an argument or a quarrel, treachery cannot be
appreciated as a qualifying circumstance, because the deceased may be said to
have been forewarned and to have anticipated aggression from the assailant.
Moreover, in order to appreciate alevosia, the method of assault adopted by the
aggressor must have been consciously and deliberately chosen for the specific
purpose of accomplishing the unlawful act without risk from any defense that
might
be
put
up
by
the
party
attacked.
The appellant acted upon an impulse so powerful as to have naturally produced
passion or obfuscation. The acute battering she suffered that fatal night in the
hands of her batterer-spouse, in spite of the fact that she was eight (8) months
pregnant with their child, overwhelmed her and put her in the aforesaid emotional
and mental state, which overcame her reason and impelled her to vindicate her life
and
that
of
her
unborn
child.
The Supreme Court affirmed the conviction of appellant for parricide. However,
considering the presence of two (2) mitigating circumstances and without any
aggravating circumstance, the penalty is reduced to six (6) years and one (1) day
of prision mayor as minimum; to 14 years 8 months and 1 day of reclusion
temporal as maximum. Inasmuch as appellant has been detained for more than the
minimum penalty hereby imposed upon her, the director of the Bureau of
Corrections may immediately RELEASE her from custody upon due

determination that she is eligible for parole, unless she is being held for some
other
lawful
cause.

NOTE: After this case was decided by the Supreme Court, R.A. 9262, otherwise
known as Anti-Violence Against Women and their Children Act of 2004 was
enacted. Sec. 26 of said law provides that "xxx. Victim-survivors who are found
by the courts to be suffering from battered women syndrome do not incur any
criminal and civil liability nothwithstanding the absence of any of the elements
for justifying circumstances of self-defense under the Revised Penal Code

80. PEOPLE VS BATES

Facts:
Around 2:00 in the afternoon of November 28, 1995, Edgar Fuentes, Simon Fuentes and Jose
Boholst left Barangay Esperanza, Ormoc City to deliver copra to a certain Fely Rodado at
Barangay Green Valley, Ormoc City. After delivering copra around 5:00 in the afternoon, the
three men headed back to Barangay Esperanza. While they were along a trail leading to the
house of Carlito Bates, the latter suddenly emerged from the thick banana plantation surrounding
the trail, aiming his firearm at Jose Boholst who was then walking ahead of his companions.
Jose grabbed Carlitos right hand and elbow and tried to wrest possession of the firearm. While
the two were grappling for possession, the gun fired, hitting Carlito who immediately fell to the
ground. At that instant, Marcelo Bates and his son Marcelo Bates, Jr., brother and nephew of
Carlito, respectively, emerged from the banana plantation each brandishing a bolo.

They

immediately attacked Jose hacking him several times. Jose fell to the ground and rolled but
Marcelo and his son kept on hacking him. Marcelo, then, turned to Simon and Edgar and
shouted huwes de kutsilyo. Upon hearing the same, Simon and Edgar ran.
Upholding the prosecution evidence, the trial court rendered its Judgment, finding Marcelo Bates
guilty beyond reasonable doubt of the crime of Murder.

Issue:
Whether or not Marcelo could validly invoke the mitigating circumstance of passion and
obfuscation?
Decision:
Passion and obfuscation may not be properly appreciated in favor of appellant. To be considered
as a mitigating circumstance, passion or obfuscation must arise from lawful sentiments and not
from a spirit of lawlessness or revenge or from anger and resentment. In the present case,
clearly, Marcelo was infuriated upon seeing is brother, Carlito, shot by Jose. However, a
distinction must be made between the first time that Marcelo hacked Jose and the second time
that the former hacked the latter. When Marcelo hacked Jose right after seeing the latter shoot at
Carlito, and if appellant refrained from doing anything else after that, he could have validly
invoked the mitigating circumstance of passion and obfuscation. But when, upon seeing his
brother Carlito dead, Marcelo went back to Jose, who by then was already prostrate on the
ground and hardly moving, hacking Jose again was a clear case of someone acting out of anger
in the spirit of revenge.

81. PEOPLE VS NIMUAN

Facts: Eulalia Garcia was tending hersari-sari store in La Union when the armed appellant and
Lamberttold her that they were going to kill the doctor. The two left when they saw the doctor
pass byen route to the poultry farm. Ten minutes later, Garcia heard two gunshots coming from thedirection of
the poultry farm.In the poultry farm, after the doctor had given medicines andbread to his
workers Manolong,Yaranon and Anasario, he went to another building. Said workers then
heard gunfire coming from the victim direction and went down to investigate. On the way, they met
theappellant and Lamberte, who threatened them with harm should they tell anyone that they
(theappellant and Lamberte) were responsible for the killing of the victim. The appellant
andLamberte left.The appellant denied any participation in the killing of the victim, and pointed
to Lamberte asthe person solely responsible. He claimed that he merely accompanied Lamberte
to thevictim farm when the latter suddenly shot the victim

Issues: Whether or notthere is conspiracy; WON there is evident premeditation.


Held: The prosecution has clearly proven that a conspiracy existed between appellant and
Lamberte,who had the common design of killing the victim. Both were armed and
both threatenedworkers Manolong, Yaranon and Anasario with harm should they tell anyone that
they (accused)had killed the victim.
It doesnt matter who actually shot the victim because of theconspiracy that existed. In
conspiracy, the act of one is the act of all; each of the accused isequally guilty of the crime
committed.The qualifying circumstance of treachery is present as the victim was shot at the
back. Theattack was deliberate, sudden and unexpected; it afforded the unsuspecting victim
noopportunity to resist or defend himself.The aggravating circumstance of evident premeditation
is not appreciated; there was no proof,as clear as the evidence of the crime itself, of (1) the time
when the offender determined tocommit the crime; (2) an act manifestly indicating that the
accused clung to his determination;and (3) a sufficient lapse of time between determination and
execution to allow himself time toreflect upon the consequences of his act. In this case, there is
little evidence when the accusedfirst conceived of killing the victim and that they were afforded
sufficient time to reflect on theconsequences of their contemplated crime before its final execution.
Moreover, the span of time(less than thirty minutes), from the time the accused showed their
determination to kill thevictim (when they told Garcia that the
y were going to kill the doctoru p t o t h e t i m e they shot the victim, could not have
afforded them full opportunity for meditation and reflectionon the consequences of the crime
they committed.Murder is killing qualified treachery, evident premeditation, among others, as
per Art. 248 of theRPC. Its punishment is reclusion perpetuato death under Article 248 of the
Revised Penal Code,as amended. Since neither aggravating nor mitigating circumstances
attended the commissionof the felony, the proper imposable penalty on the appellant is
reclusion perpetua

82. PEOPLE VS MAGILAN


Facts: Bernardo Bagabag was murdered in his own house in Abra on June 24, 1967 by
Talingdan, Tobias, Berras, Bides and Teresa Domogma, his alleged wife [whom cannot be
charged with parricide because no certificate or proof of marriage could be presented by the
prosecution]. The murder was witnessed by Corazon [12], the eldest child of Bernardo and
Teresa. She testified to the crime committed by the accused-appellants.

Story: [I want to include this so that you could get the whole thing; it is your option to write it or
not on your record books. This is the summary of Corazons testimony.] Prior to the violent
incident, Bernardo and Teresa have had several conflicts in their married life. She would often

withdraw from their house. The longest even for more than 3 weeks. It was suspected that Teresa
is having an illicit affair with Talingdan, a policeman who lives nearby. Two days before the
crime, Teresa was slapped several times by Bernardo after a violent quarrel. She sought the help
of Talingdan who challenged Bernardo to come down, but the latter refused. Then, Talingdan left
after shouting "If I will find you someday, I will kill you." Two days before the commission of
the crime, Corazon overheard her mothers meeting with the other accused-appellants about their
plot to kill her father as one of them said, Shall he elude a bullet? Corazon was then driven
away by her mother saying, You tell your father that we will kill him. On the night of the
murder, Corazon was cooking food for supper when she saw her mother talking with the other
accused-appellants in their batalan armed with long guns. After a while, Teresa went inside the
room to put her baby to sleep. After eating supper alone, Corazon told her father about the
persons outside but he ignored her. He went to the kitchen and sat on the floor near the door then
he was fired. Talingdan and Tobias fired their guns again. Bides threatened to kill Corazon if she
would ask for help. Corazon confessed to her fathers relatives the identities of the murderers
during his burial. The trial court found them guilty of the offense and so the five accused
appealed
to
their
conviction.

Contention of the Accused: According to Teresa, there was no illicit affair between her and
Talingdan. She loved her husband. Contrary to the testimony of Corazon, they never quarreled
nor did the former maltreat her. She never left home for so long. And she was cooking for supper,
and not Corazon, on the night of the murder. She contends that her in-laws used her daughter to
testify against her because they dont want Teresa from the start. She even added that Bernardo
had some enemies during his lifetime. Talingdan said that he escorted the Mayor as a bodyguard,
while the other three accused also claimed that they were at a certain Mrs. Bayongans house
during
the
night
of
the
murder.

Contention of the People: The sworn statement of the 13-year old Corazon was true. She knew
the accused because they live nearby their place. Besides, the accused-appellants testimonies are
indefensible and futile. Moreover, her mother claimed to have no suspect in mind during the
investigation in their house although she was in conspiracy with the other four accused.

Ruling: The court affirmed the decision held by the trial court with costs. There are two
aggravating circumstances present, treachery and evident premeditation, with no mitigating
circumstances to offset the accused-appellants. Talingdan, Tobias, Berras, and Bides are guilty
beyond reasonable doubt of murder and are sentenced to DEATH to be executed in accordance
with law. Teresa Domogma is guilty as accessory to the same murder, and is hereby sentenced to
suffer the indeterminate penalty of 5 years prision correccional as minimum to 8 years of prision
mayor as maximum, with the accessory penalties of the law.

83. PEOPLE VS CONCILLADO

Facts
Florencio, Eddie, Elynor, Franklin and Eric, all surnamed Agacer, are found guilty for the killing
of acommon relative, Cesario Agacer. The appellants surrounded the victim and one of them set
a fire tokeep Cesario from retreating. Franklin and Eric hit the deceased with stones, Florencio
induces thevictim to come closer, was hit with a gunshot from Eddie and was shot with a bow and arrow by
Elynor.They left the crime scene together, onboard a tractor and a tricycle.
Issues:1. Whether or not conspiracy was involved and if all appellants are liable for the murder
2. Whether or not appellants are guilty of the aggravating circumstance of treachery
Held:
Yes. In the case at bar, conspiracy is evident in the way the appellants surprised, surrounded,attacked and
abandoned the deceased together. Proof of previous agreement is not essentialbecause all acted in unison
pursuing one goal, which is to kill the victim. Distinguishing the fatalblow is immaterial in
indicting appellants for criminal liability; all are equally liable for murdersince conspiracy is
present.
Yes. Treachery was present, fulfilling the conditions that first, the victim was not given theopportunity to defend
himself and second, that the means of execution was deliberate (evidentin the fact that the accused carried
the weapons employed).
84. PEOPLE VS DWASTON

Facts:
Edgar Dawaton was found guilty by the trial court of murder qualified by treachery and
was sentenced to death. On 20 September 1998, Leonidas Lavares and several companions,
including Dawaton were drinking in the house of the accuseds uncle. Already drunk, Leonidas
Lavares decided to sleep while the accused and his companions continued drinking. Dawaton
awakened Lavares by stabbing him at the base of the neck. Dawaton continued stabbing Lavares
until the victim died. Dawaton then ran away to the house of his other relative, where he was
later on arrested by the police.

Issue:
Whether or not the penalty of death imposed by the trial court upon the accused was
correct?
Decision:
No. The Supreme Court held that the trial court erred in not considering the alternative
circumstance of intoxication in favor of the accused. Under Art. 15 of The Revised Penal Code,
intoxication of the offender shall be considered as a mitigating circumstance when the offender
commits a felony in a state of intoxication, if the same is not habitual or subsequent to the plan to
commit said felony. Otherwise, when habitual or intentional, it shall be considered as an
aggravating circumstance.The allegation that the accused was drunk when he committed the
crime was corroborated by the prosecution witnesses. The accused and his drinking companions
had consumed four (4) bottles of gin at the house of Esmeraldo Cortez, each one drinking at least
a bottle. It was also attested that while the four (4) shared another bottle of gin at the house of
Amado Dawaton, it was the accused who drank most of its contents. The Court further stated
that Under Art. 63, par. 3, of The Revised Penal Code, in all cases in which the law prescribes a
penalty composed of two (2) indivisible penalties, such as in this case, when the commission of
the act is attended by a mitigating circumstance and there is no aggravating circumstance, the
lesser penalty shall be applied. Since no aggravating circumstance attended the killing but there
existed the mitigating circumstance of intoxication, the accused should be sentenced only to the
lesser penalty of reclusion perpetua.

85. PEOPLE VS MONTINILLA

Facts:

On 18 November 1996, William Montinola, armed with an unlicensed Cal .380 Pistol
Llama deliberately, willfully and criminally with violence against or intimidation of persons,
with intent of gain, take and carry away cash amount of P67,500.00 belonging to Jose Eduardo
Reteracion. Montinola shot the victim on the neck, killing Reteracion. Two criminal cases were
filed against Montinola and he was later on sentenced to reclusion perpetua for robbery with
homicide and death for illegal possession of firearm.
Issue:
Whether the use of an unlicensed firearm on the killing perpetrated by reason or on
occasion of the robbery may be treated as a separate offense or as an aggravating circumstance in
the crime of robbery with homicide?
Decision:
Sec. 1 of P.D.1866 provides that if homicide or murder is committed with the use of an
unlicensed firearm, the penalty of death shall be imposed. Said Presidential Decree was however,
amended by R.A. 8294, while Montinolas case was still pending. R.A. 8294 provides that if
homicide or murder is committed with the use of an unlicensed firearm, such use of an
unlicensed firearm shall be considered as an aggravating circumstance. The Court held In recent
cases, we ruled that there could be no separate conviction for illegal possession of firearm if
homicide or murder is committed with the use of an unlicensed firearm; instead, such use shall
be considered merely as an aggravating circumstance in the homicide or murder committed.
Hence, insofar as the new law will be advantageous to WILLIAM as it will spare him from a
separate conviction for illegal possession of firearm, it shall be given retroactive effect.
Pursuant to the third paragraph of Section 1 of P.D. No. 1866, as amended by R.A. No. 8294, use
of an unlicensed firearm is a special aggravating circumstance in the homicide or murder
committed. At any rate, even assuming that the aggravating circumstances present in the
commission of homicide or murder may be counted in the determination of the penalty for
robbery with homicide, we cannot appreciate in this case the special aggravating circumstance of
use of an unlicensed firearm mentioned in the third paragraph of Section 1 of P.D. No. 1866, as

amended by R.A. No. 8294. Such law was not yet enacted when the crime was committed by
WILLIAM; it cannot, therefore, be given retroactive effect for being unfavorable to him. The
Court further held Under Article 294 of the Revised Penal Code, as amended by R.A. No. 7659,
robbery with homicide is punishable by reclusion perpetua to death, which are both indivisible
penalties. Article 63 of the same Code provides that in all cases in which the law prescribes a
penalty composed of two indivisible penalties, the greater penalty shall be applied when the
commission of the deed is attended by one aggravating circumstance. If we would apply
retroactively the special aggravating circumstance of use of unlicensed firearm under Section 1
of P.D. No. 1866, as amended by R.A. No. 8294, the imposable penalty would be death.
Conformably with our ruling in People v. Valdez, insofar as the new law would aggravate the
crime of robbery with homicide and increase the penalty from reclusion perpetua to death, it
would not be given retroactive application, lest it would acquire the character of an ex post facto
law. Hence, we shall not appreciate that special aggravating circumstance. There being no
modifying circumstances, the lesser penalty of reclusion perpetua shall be imposed upon
accused-appellant WILLIAM.

86. CANTA VS PEOPLE

Facts:
Narciso Gabriel owns a cow that was passed on from one person to another and each
person was responsible for the care and custody of the said cow. At the time the cow got lost, it
was under the care and custody of Gardenio Agapay. Agapay took the cow in the mountain of
Pilipogan, 40 meters away from his hut, at around 5:00 in the afternoon. When he came back to
get the cow at past 9 in the evening, the cow was gone. However, Aagapay saw footprints that
led to the house of Filomeno Vallejos. Vallejos told Agapay that Exuperancio Canta took the
cow.
Agapay and Maria were instructed by Narciso to get the cow and on their way to
Florenitno Cantas house, they saw Exuperancio. The latter told them that if it was really

Narciso who was the owner of the cow, he should get it himself. Exuperancia accompanied the
two to his fathers house and both recognized the cow but Florentino was not home.
Exuperancio told Maria and Agapay that he would call them the next day to talk about the matter
with his father. Exuperancio never called. The matter was reported to the police and Narciso
and Exuperancio were called for investigation. Exuperancio admitted taking the cow but claims
that he was the real owner of the cow and that it was lost on December 3, 1985. However,
Narciso presented a certificate of ownership issued on March 9, 1986, signed by the municipal
treasurer, in which the cow was described as two years old and female. Then, the petitioner also
presented a Certificate of Ownership of Large Cattle dated February 27, 1985 and a statement
executed by Franklin Telen, who was the janitor at the treasurer's office of the municipality, that
he executed the certificate of ownership in favor of Exuperancio. The trial court rendered its
decision finding petitioner guilty of the offense charged.

Exuperancio filed a Motion for

reconsideration but was denied by the Court of Appeals and affirmed the trial court's decision.
Issue:
Whether or not the lower courts were correct in sentencing Exuperancio to ten (10) years
and one (1) day of prision mayor, as minimum, to twelve (12) years, five (5) months, and eleven
(11) days of reclusion temporal medium, as maximum, and to pay the costs?
Decision:
No. The Supreme Court held that the trial court correctly found petitioner guilty of
violation of 2(c) of P. D. No. 533, otherwise known as the Anti-Cattle Rustling Law of 1974.
However, it erred in imposing the penalty of 10 years and 1 day of prision mayor, as minimum,
to 12 years, 5 months and 11 days of reclusion temporal medium, as maximum. The trial court
apparently considered P. D. No. 533 as a special law and applied 1 of the Indeterminate
Sentence Law, which provides that "if the offense is punished by any other law, the court shall
sentence the accused to an indeterminate sentence, the maximum term of which shall not exceed
the maximum fixed by said law and the minimum shall not be less than the minimum term
prescribed by the same." However, as held in People v. Macatanda, P. D. No. 533 is not a special

law. The penalty for its violation is in terms of the classification and duration of penalties
prescribed in the Revised Penal Code, thus indicating that the intent of the lawmaker was to
amend the Revised Penal Code with respect to the offense of theft of large cattle. In fact, 10 of
the law provides:
The provisions of Articles 309 and 310 of Act No. 3815, otherwise known as the Revised
Penal Code, as amended, pertinent provisions of the Revised Administrative Code, as
amended, all laws, decrees, orders, instructions, rules and regulations which are
inconsistent with this Decree are hereby repealed or modified accordingly.
There being one mitigating circumstance and no aggravating circumstance in the
commission of the crime, the penalty to be imposed in this case should be fixed in its minimum
period. Applying the Indeterminate Sentence Law, in relation to Art. 64 of the Revised Penal
Code, petitioner should be sentenced to an indeterminate penalty, the minimum of which is
within the range of the penalty next lower in degree, i. e., prision correccional maximum to
prision mayor medium, and the maximum of which is prision mayor in its maximum period.

87. PEOPLE VS NAVASCA

FACTS: The issue started when the Secretary of the Philippine Senate, Fernando Guerrero,
discovered that the documents regarding the testimony of the witnesses in an investigation of oil
companies had disappeared from his office. Then, the day following the convening of Senate, the
newspaper La Nacion edited by herein respondent Gregorio Perfecto published an article
against the Philippine Senate. Here, Mr. Perfecto was alleged to have violated Article 256 of the
Spanish Penal Code provision that punishes those who insults the Ministers of the Crown.
Hence, the issue.
ISSUE: Whether or not Article 256 of the Spanish Penal Code (SPC) is still in force and can be
applied in the case at bar?
HELD: No.

REASONING: The Court stated that during the Spanish Government, Article 256 of the SPC
was enacted to protect Spanish officials as representatives of the King. However, the Court
explains that in the present case, we no longer have Kings nor its representatives for the
provision to protect. Also, with the change of sovereignty over the Philippines from Spanish to
American, it means that the invoked provision of the SPC had been automatically abrogated. The
Court determined Article 256 of the SPC to be political in nature for it is about the relation of
the State to its inhabitants, thus, the Court emphasized that it is a general principle of the public
law that on acquisition of territory, the previous political relations of the ceded region are totally
abrogated. Hence, Article 256 of the SPC is considered no longer in force and cannot be applied
to the present case. Therefore, respondent was acquitted

88. PEOPLE VS MONAGA

FACTS:Petitioner Martin Monago (head of the counter-intelligence unit of the MPD),


havingreceived a reliable information that a certain shipment of personal effects wereallegedly
misdeclared and undervalued and were to be released from the customszone of the port of
Manila, conducted surveillance of said zone. With him werepetitioner Ricardo Papa, the Chief of
Police of Manila and a duly deputized memberof the BOC, and other elements of the counterintelligence unit. The informationwhich reached Alagao specified that said misdeclared and
undervalued items wereloaded on two trucks.2.The trucks leftthe gate where Alagaos group
conducted surveillance. However,such trucks were later intercepted. The load of the two trucks
consisted of 9 balesof goods.3.The cargo was owned by Remedios Mago while the truck was
owned by ValentinLanopa. In their petition in the CFI of Manila, they claimed that the MPD
seized thegoods without search warrant issued by a competent court, and that Papa denied
the request of Magos counsel that the bales be not opened and the goods not
examined.4. The respondent judge issued an order restraining petitioners from opening thenine
bales in question. However, some bales were already opened by examiners ofthe BOC when the
restraining order was received.5. Respondent contended that, since the inventory of the goods
seized did not showany article of prohibited importation, such articles should be released upon
herposting of the bond to be determined by court. Petitioners contended howeverthat most of the
goods, as shown in the inventory, were not declared and were
thus subject to
forfeiture. Respondent judge issued an order releasing the goodupon the filing of the bond in the
amount of Php 40,000.00 to which therespondent complied with.
Issue: Is there a need to procure a warrant before search be made?

Held:
The Bureau of Customs acquires exclusive jurisdiction over imported goods, for thepurposes of
enforcement of the customs laws, from the moment the goods areactually in possession or
control, even if no warrant of seizure or detention hadpreviously been issued by the Collector of
Customs in connection with seizure andforfeiture proceedings. In the case at bar, the moment the
BOC actually seized thegoods in question, the BOC acquired jurisdiction over the goods for the
purposes ofenforcement of the tariff and customs laws, to the exclusion of the regular courts.2.
Petitioner Alagao and his companion policemen had authority to effect the seizurewithout any
search warrant issued by a competent court. The Tariff and CustomsCode does not require said
warrant in the instant case. The Code authorizespersons having police authority under Section
2203 to enter, pass through orsearch any land, inclosure, warehouse, store or building, not being
a dwellinghouse; and also to inspect, search and examine any vessel or aircraft and any
trunk,package or envelope or any person on board, or to stop and search and examineany vehicle,
beast or person suspected of holding or conveying any dutiable orprohibited articles
89. PEOPLE VS PALAGANAS

Facts:
On January 16, 1998 brothers Servillano and Michael Ferrer went to Tidbits Videoke bar
singing and drinking beer. On the same evening Jaime Palaganas and Ferdinand Palaganas and
Virgilio Bautista arrived. The two groups occupied separate tables. After the Ferrers singing
Jaime Palaganas started singing and was joined by Tony Ferrer who sang loudly and in mocking
manner. This insulted Jaime and soon a fight ensued between Ferrers and Palaganas. Ferdinand
ran towards his house and sought help from his brother Fuijeric, the latter went outside however
he was stoned by the Ferrer brothers. As they were continuously stoned the appellant Ferdinand
suddenly pulled the trigger with the gun in his hands. The trial court rendered a decision finding
the petitioner guilty of the crime of Homicide and Frustrated homicide but not guilty of violation
of COMELEC RES. 2958.
Issue:

Whether or not violation of COMELEC RES. 2958 may be considered as Special


aggravating circumstances which will negate consideration of mitigating circumstances of
voluntary surrender?
Decision:
With the passage of Republic Act. No. 8294 on 6 June 1997, the use of an unlicensed
firearm in murder or homicide is now considered as a SPECIAL aggravating circumstance and
not a generic aggravating circumstance.68 Republic Act No. 8294 applies to the instant case since
it took effect before the commission of the crimes in 21 April 1998. Therefore, the use of an
unlicensed firearm by the petitioner in the instant case should be designated and appreciated as a
SPECIAL aggravating circumstance and not merely a generic aggravating circumstance.

90. PEOPLE VS EVINA

Facts:
Gerardo Gavina was serve sentence of Reclusion Pertpetua for raping certain Ms.
Maritess Catcharo. Based on the given facts, Gerardo took advantage of the time when
the victims mother was not around. He would likely forced Maritess to have carnal
knowledged against her will and even poked a knife at her while doing the deed in the
victims dwelling and threthened the victim to kill her family should she tell her parents
what happened. On November 13, 1991 when the appellant arrived at the Catcharro
residence he proceeded inside the bedroom of Maritess, the latter ran out of the bedroom
and told her mother not to leave her because her Papa Gerry might raped her again.
Surprised by what he heard, the following day Maritess was brought to Tacloban City
Medical Center for a check-up and found to have lacerations to the victims genitalia.
Contrary to the facts above, appellant claimed that the night of the incident he was
working as porter until 10 PM, thus it cannot be said that he committed the crime accused
of him. Based on the information submitted, aggravating circumstances of use of weapon
and dwelling were not alleged.

Issue:
Whether or not aggravating circumstances proved during trial but was not alleged
in the information may be considered?
Decision:
The supreme court held in the negative. Although the special aggravating
circumstance of the use of a weapon and the aggravating circumstance of dwelling were
proven, these aggravating circumstances cannot be considered in fixing the penalty
because they were not alleged in the information as mandated by Rule 110, Sections 8
and 9 of the Revised Rules of Criminal Procedure. Although the crimes charged were
committed before the effectivity of the said rule, nevertheless, the same should be applied
retroactively being favorable to the appellant.

91. PEOPLE VS MENDOZA

Facts:
Efren Mendoza was charged with the crime of murder for killing Anchito Nano. In this
case Efren alleged that Anchito Nano arrived at their house and upon arrival it started to destroy
the house and that the her wife was shouting for help. Efren immediately look for something to
protect his family but found a bolo. He approached Anchito but the latter tried to hacked him but
he was able to hacked him first on the right side of his neck resulting to the death of the victim.
Thereafter Mendoza went to Municipal Hall of Vinzon and voluntarily surrendered to the police.
He claimed that it was self defense. The autopsy revealed that location of the wounds found on
the body of the victim came from the back of the victims body. The court ruled rejecting
appellants self defense. This court finds that the accused was not in imminent danger of death
or great bodily harm, an attempt to defend himself by means which appeared unreasonable by
using a long bolo is unjustifiable. Hence this appeal.

Issue:
Whether or not voluntary surrender was offset by the aggravating circumstances of
treachery?
Decision:
The Supreme Court held in the negative. A qualifying circumstance changes the nature
of the crime. A generic aggravating circumstance, on the other hand, does not affect the
designation of the crime; it merely provides for the imposition of the prescribed penalty in its
maximum period. Thus, while a generic aggravating circumstance may be offset by a mitigating
circumstance, a qualifying circumstance may not. 32
Treachery in the present case is a qualifying, not a generic aggravating circumstance. Its presence
served to characterize the killing as murder; it cannot at the same time be considered as a generic
aggravating circumstance to warrant the imposition of the maximum penalty. Thus, it cannot
offset voluntary surrender.

92. PEOPLE VS VILLAMOR

Facts:
On November 25, 1995, brothers Jerry Velez and Jelord Velez were on their way home on
board a motorcycle. A motorcycle was speeding behind them and as they were about to cross the
bridge, they heard gun shots firing behind them. As they turned around, Jerry identified PO3
Renato Villamor and Jessie Maghilom riding the motorcycle behind them. Shots were fired at
them and Jerry sustained wounds on the abdomen and elbow while Jelord died on the spot. The
trial proceeded against PO3 Villamor while Maghilom was still at large. During trial, the Trial

Court found the PO3 Renato Villamor guilty of having commited Murder aggravated by the
circumstance of taking advantage of his public position.
Issue:
Whether or not the Trial Court properly applied the aggravating circumstance of taking
advantage of public position?
Decision:
The Supreme Court ruled that the aggravating circumstance of taking advantage of
public position under paragraph 1 of Article 14 of the Revised Penal Code was improperly
applied.
A public officer must use the influence that is vested in his office as a means to realize
the purpose of the crime to be appreciated as an aggravating circumstance. The question Did
the accused abuse his office to commit the crime must be asked in order to appreciate this
circumstance as an aggravating circumstance.
No proof was shown that Villamor took advantage of his position of being a policeman
when he shot Jelord Velez. Neither was his influence, prestige or ascendancy used in killing
Velez. Even without occupying a public position, the accused could have committed the crime.

93. FORTUNA VS PEOPLE


\
Facts:
On July 21, 1992, siblings Diosdada Montecillo and Mario Montecillo were standing at
the corner of Mabini and Harrison Streets. A mobile patrol car stopped in front of them and a
policeman alighted. The policeman frisked Mario and took Marios belt. He motioned Mario to

enter the car. Mario obeyed and was followed by Diosdada. While inside the car, the policemen
told Mario that he would be brought to the Bicutan police station where he would be
interrogated, mauled and heckled for carrying a deadly weapon. They told the Montecillos that
the bailbond for carrying a deadly weapon was P12,000. The Montecillos were asked how much
they had and then Diosdada was asked to alight from the car. The driver followed her, took
P1,500 from her wallet and instructed her to tell the others that she only had P3,500. Inside the
car, they were told to put all her money on the box. The Montecillos were told to get off at
Harrison Plaza. From there, they went home. The 3 policemen, Fortuna, Garcia, and Pablo, were
charged with robbery and were found guilty of having conspired in committing the crime with
intimidation of persons.
Issue:
Whether or not abuse of public position should be taken as an aggravating circumstance
by the mere fact that the accused were police officers?
Decision:
The Supreme Courted held that the lower courts failed to appreciate the aggravating
circumstance of abuse of public position.
Being police officers, it placed them in a position terrify the Montecillos to surrender their
money as bail. It was on the account of their authority that convinced the Montecillos that they
had committed a crime and that they would be taken to the police station. Had they not been
police officers, they would have not convinced the Montecillos into giving them their money.

94. PEOPLE VS DE MESA

Facts:

Barangay Chairman Patricio Motas of Sta. Cruz Putol, San Pablo City was pronounced
dead on arrival on October 15, 1996 at San Pablo City District Hospital. The autopsy report
showed that the cause of death was shock and hemorrhage due to gunshot wounds at the back of
the victim.
Hernando De Mesa was found guilty beyond reasonable doubt for the crime of murder by
the Regional Trial Court of San Pablo City. He was sentenced to suffer the penalty of Reclusion
Perpetua, pay the costs and to indemnify the heirs of the victim. Treachery, nighttime, in
contempt of or with assault to public authorities, were appreciated by the trial court as
aggravating circumstances attending the case thereby qualifying the crime committed to murder.
Issue:
Whether or not the trial court erred in determining the nature of the crime committed and
the corresponding penalty to be imposed?
Decision:
Yes. The prosecution failed to positively prove the presence of any qualifying
aggravating circumstance whereby the crime committed is only homicide for which the
imposable penalty provided by the Revised Penal Code is Reclusion Temporal.
Being the case, Indeterminate Sentence Law may now be applied and absent any
aggravating nor mitigating circumstance, the penalty that may be imposed is prision mayor in its
medium period as minimum to reclusion temporal in its medium period as maximum.

95. PEOPLE VS TACAN

Facts:

Renato Tac-an and Francis Escano were close friends being classmates in high school and
members of the local Bronx gang. Francis withdrew from the gang on the advice of his mother
who saw that Renato carried a handgun on his visits to their home. Things started turning sour
between the two, and came to a head on Dec 14, 1984. After an earlier altercation on that day,
Renato went home and got his gun. He entered the Mathematics class under Mr. Damaso
Pasilbas in Rm15 and shouted for Francis. After locating the victim he fired at him but missed.
He was later able to hit him in the head as he was running to the door with his classmates to
escape. After this, Renato paced outside in the hallway. A teacher unknowing that Renato was the
culprit, asked him for help unwittingly informing him that Francis was still alive. Renato
immediately re-entered the room and saying "So, he is still alive. Where is his chest?" Standing
over Francis sprawled face down on the classroom floor, Renato aimed at the chest of Francis
and fired once more. The bullet entered Francis' back below the right shoulder, and exited on his
front chest just above the right nipple.
Tac-an was charged with illegal possession of firearms under P.D. No. 1866. An amended
information for murder was subsequently filed aggravated by the use of illegal possession of
firearms.
Issues:
Whether or not illegal possession of a firearm is a special aggravating circumstance in
crimes of homicide and murder?
Decision:
No. Under an information charging homicide or murder, the use of an unlicensed firearm
is not an aggravating circumstance nor can it be used to increase the penalty for the second
offense of homicide or murder to death or reclusion perpetua. The character of the instrument
used in taking or destroying human existence is not one of those included in the enumeration of
aggravating circumstances under Article 14 of the Revised Penal Code.

On the other hand, under an information for unlawful possession of a firearm or


ammunition, P.D. 1866 authorizes the increase of the imposable penalty for unlawful possession
if the unlicensed firearm was used to destroy human existence. Though it is not one of the
enumerated aggravating circumstances in Article 14 of the Revised Penal Code, it may still be
considered to increase the penalty imposed because of the explicit provision of the said special
law.

96. PEOPLE VS REYES

FACTS:
On April 30, 1984, the appellant dragged the deceased towards the street and stabbed her in the
chest with a fan knife. Although the wound was just a slight one, it not having penetrated the
thoracic cavity, Fausta Tavera after running a bit, died.

ISSUE:
WON the accused is guilty of homicide although the wound is just superficial.
HELD:
Yes. A person is responsible for the consequences of his criminal act even if the deceased had
been shown to be suffering from a diseased heart, appellants assault being the proximate cause
of the death, he would be responsible. When a person stabs another with a lethal weapon, the
accused is presumed to have intended the natural consequences of the wrongful act.

97. PEOPLE VS MONTINILLA

Facts:

On 18 November 1996, William Montinola, armed with an unlicensed Cal .380 Pistol
Llama deliberately, willfully and criminally with violence against or intimidation of persons,
with intent of gain, take and carry away cash amount of P67,500.00 belonging to Jose Eduardo
Reteracion. Montinola shot the victim on the neck, killing Reteracion. Two criminal cases were
filed against Montinola and he was later on sentenced to reclusion perpetua for robbery with
homicide and death for illegal possession of firearm.
Issue:
Whether the use of an unlicensed firearm on the killing perpetrated by reason or on
occasion of the robbery may be treated as a separate offense or as an aggravating circumstance in
the crime of robbery with homicide?
Decision:
Sec. 1 of P.D.1866 provides that if homicide or murder is committed with the use of an
unlicensed firearm, the penalty of death shall be imposed. Said Presidential Decree was however,
amended by R.A. 8294, while Montinolas case was still pending. R.A. 8294 provides that if
homicide or murder is committed with the use of an unlicensed firearm, such use of an
unlicensed firearm shall be considered as an aggravating circumstance. The Court held In recent
cases, we ruled that there could be no separate conviction for illegal possession of firearm if
homicide or murder is committed with the use of an unlicensed firearm; instead, such use shall
be considered merely as an aggravating circumstance in the homicide or murder committed.
Hence, insofar as the new law will be advantageous to WILLIAM as it will spare him from a
separate conviction for illegal possession of firearm, it shall be given retroactive effect.
Pursuant to the third paragraph of Section 1 of P.D. No. 1866, as amended by R.A. No. 8294, use
of an unlicensed firearm is a special aggravating circumstance in the homicide or murder
committed. At any rate, even assuming that the aggravating circumstances present in the
commission of homicide or murder may be counted in the determination of the penalty for
robbery with homicide, we cannot appreciate in this case the special aggravating circumstance of

use of an unlicensed firearm mentioned in the third paragraph of Section 1 of P.D. No. 1866, as
amended by R.A. No. 8294. Such law was not yet enacted when the crime was committed by
WILLIAM; it cannot, therefore, be given retroactive effect for being unfavorable to him. The
Court further held Under Article 294 of the Revised Penal Code, as amended by R.A. No. 7659,
robbery with homicide is punishable by reclusion perpetua to death, which are both indivisible
penalties. Article 63 of the same Code provides that in all cases in which the law prescribes a
penalty composed of two indivisible penalties, the greater penalty shall be applied when the
commission of the deed is attended by one aggravating circumstance. If we would apply
retroactively the special aggravating circumstance of use of unlicensed firearm under Section 1
of P.D. No. 1866, as amended by R.A. No. 8294, the imposable penalty would be death.
Conformably with our ruling in People v. Valdez, insofar as the new law would aggravate the
crime of robbery with homicide and increase the penalty from reclusion perpetua to death, it
would not be given retroactive application, lest it would acquire the character of an ex post facto
law. Hence, we shall not appreciate that special aggravating circumstance. There being no
modifying circumstances, the lesser penalty of reclusion perpetua shall be imposed upon
accused-appellant WILLIAM.

98. PEOPLE VS TABOGA

Facts :Edralin Taboga was charged with Robbery with Homicide in an Information which
readsthat with intent to gain, and with violence against persons, entered the house ofone
Francisca Tubon and once inside, with treachery and abuse of superior strength,assault, attacked
and stabbed Tubon thereby inflicting upon her mortal wounds whichnecessarily caused the death
of said Tubon and took away several personalproperties belonging to Tubon. He was likewise
indicted for Arson for setting the victims house on fire. After finding the burnt house and
charred body of Tubon, Baranggay Captain Pagaoconfronted Taboga, and the latter readily
admitted that he killed Tubon and set herhouse on fire, causing the whole house, including the
dead body of the old woman, tobe burned.Taboga was brought to the police station for further
investigation. Mr. Mario Contaoi, aradio announcer of DZNS, went to Police Station to interview
the suspect. Again,Taboga admitted killing the deceased and setting her and her house on
fire.Upon arraignment, accused-appellant entered separate pleas of "Not Guilty" to thecrimes
charged and interposed an alibi. Accused-appellant also claimed that he wasmaltreated by the
policemen and forced to admit the crime. Regarding his admission toradio announcer Contaoi, he

narrated that the interview was held inside theinvestigation room of the police station
wherepolicemen were present and that thereporter acted as an agent for the prosecution
.Thus, he had to admit the crimesbecause he was afraid of the policemen.The RTC rendered
judgment finding him guilty beyond reasonable doubt of both crimes.
Issue: Whether or not confession made by the accused to a radio reporter, a privateperson, can be
admitted as evidence against him.
Held :Yes. There is nothing in the record to show that the radio announcer colluded withthe
police authorities to elicit inculpatory evidence against accused-appellant.Neither is there
anything on record which even remotely suggests that the radioannouncer was instructed by the
police to extract information from him on the

99. PEOPLE VS EVANGLIO

Facts:
Sentenced to life imprisonment and a fine of P20,000.00 by the Regional Trial Court of Manila was
appellant Yolanda Velasco y Pamintuan, after having been found guilty of unlawfully selling shabu,
in violation of Section 15 of Article III in relation to Section 2(e-2), (f), (m), and (o) of Article 1 of The
Dangerous Drugs Act of 1972 (R.A. 6425).
Velasco was apprehended in a buy-bust operation in the afternoon of June 28, 1991. Velasco was
caught in flagrante delicto as she was handing shabu to a designated poseur-buyer. Five more decks
were found in her pockets.
Appellant argues that the court erred in admitting the said decks of shabu as evidence against her
since those were acquired through a warrantless arrest. Hence, its inadmissibility. Secondly,
appellant questions the RTCs jurisdiction over the case given the quantity allegedly obtained in her
possession.
ISSUES:
1.) Whether or not the decks of shabu are inadmissible as evidence for having been acquired
through a warrantless arrest.
2.) Whether or not the RTC has jurisdiction over the case.
Held

1.) Yes. Section 5(a) of Rule 113 of the Rules on Criminal Procedure provides that an arrest when
done lawfully either by a peace officer or any private person may be done if the person to be arrested
is actually committing, has committed or attempting to commit an offense.
Appellant was caught in flagrante delicto thus her denial and defense of frame-up cannot be
justified under the said provision. Moreover, appellant failed to establish that the members of the buybust team are policemen engaged in mulcting or other unscrupulous caprice when they entrapped
her.

2.) Yes. The enforcement of R.A. 7659, which amended the penalty provided for in R.A. 6425,
agrees with the appellants argument that under the foregoing directive, since the amount of
shabu involved in the instant case is only 0.8020 gram, the proper imposable
component penalty is prision correccional to be applied in its medium period, in the
absence of any mitigating or aggravating circumstances. Applying the indeterminate
Sentence Law, the maximum shall be taken from the medium of prision correccional,
which is two (2) years, four (4) months and one (1) day, to four (4) years and two (2)
months, while the minimum shall be taken from the penalty next lower in degree, which
is arresto mayor, the range of which is one (1) month and one (1) day to six (6)
months.
R.A. 7691 expanded the jurisdiction of the Metropolitan Trial Courts, Municipal Trial Courts, and
Municipal Circuit Trial Courts. The said act vested these courts with exclusive original jurisdiction
over all offenses punishable with imprisonment not exceeding six years. However, R.A. 7691 shows
that retroactive provisions apply only to civil cases that have not yet reached the pre-trial stage.
Neither from an express proviso nor by implication can it be understood as having retroactive
application to criminal cases pending or decided by the Regional Trial Courts prior to its effectivity.
RTCs jurisdiction to proceed to the final determination of the cause is not affected by the new
legislation.
At the time that the case against appellant was filed, the Regional Trial Court had jurisdiction over
the offense charged in as much as Section 39 of R.A 6425. In fine, the jurisdiction of the trial court
(RTC) over the case of the appellant was conferred by the aforecited law then in force (R.A. 6425
before amendment) when the information was filed. Jurisdiction attached upon the commencement
of the action and could not be ousted by the passage of R.A. 7691 reapportioning the jurisdiction of
inferior courts, the application of which to criminal cases is, to stress, prospective in nature.

100.

PEOPLE VS AGCANAS

101.

PEOPLE VS COLANGUL

102.

PEOPLE VS MELENDRES

103.

PEOPLE VS ANCHETA\

104.

PEOPLE VS EVANGLIO

105.

PEOPLE VS VILLANUEVA

106.

PEOPLE VS SILVA

107.

PEOPLE VS LIBRENDO

108.

PEOPLE VS DACILLO

109.

PEOPLE VS MAINGAN

110.

PEOPLE VS COMADRE

111.

PEOPLE VS PALING

112.

PEOPLE VS SEGOBRE

113.

PEOPLE VS LABUGEN

114.

PEOPLE VS LAOG

115.

PEOPLE VS AMBROCIO

116.

PWEOPLE VS CALPITO

117.

PEOPLE VS YANSON

118.

PEOPLE VS PILIIN

119.

PEOPLE VS VILLARICO

120.

PEOPLE VS AVILES

121.

PEOPLE VS ESCOTE

122.

PEOPLE VS TEEHANKEE

123.

PEOPLE VS CACHOLA

124.

PEOPLE VS BUNIDANG

125.

PEOPLE VS JOSE

126.

PEOPLE VS MALLARI

127.

PEOPLE VS ENGUINTO

128.

SIMANGAN VS PEOPLE

129.

PEOPLE VS GUERRERO

130.

PEOPLE VS WHISENHUNT

131.

PEOPLE VS TARUC

132.

PEOPLE VS REBUCAN

133.

PEOPLE VS NIMUON

134.

UCTAYO VS PEOPLE

135.

PEOPLE VS FRUGANTE

136.

PEOPLE VS CALONGUI

137.

PEOPLE VS MARCOS

138.

PEOPLE VS VASQUEZ

139.

PEOPLE VS DACILLLO

140.

PEOPLE VS DELATORRE

141.

PEOPLE VS TOMAS SR.

142.

Abarquez v. People (G.R. No. 150762)

Facts:
The prosecution charged Abarquez with the crimes of homicide and attempted homicide alleging
in the two informations filed that said accused was conspiring and confederating with one
Alberto Almojuela in the killing of Ricardo Quejong Bello, by stabbing him twice with a bladed
weapon and hitting him with a gun at the back.
The trial court found Abarquez guilty beyond reasonable doubt as an accomplice in the
crime of homicide. Abarquez filed an appeal to the Court of Appeals. However the Court of
Appeals rejected Abarquezs allegation that he was merely at the crime scene to pacify the
quarreling parties.
Abarquez alleges that the prosecutions evidence does not satisfy the test of moral
certainty and is not sufficient to support his conviction as an accomplice. He further alleges that
there was a misapprehension of facts and that the trial court and the Court of Appeals reached
their conclusion based entirely on speculation, surmises and conjectures. Abarquez also assails
the credibility of the witnesses against him.
Issue:
Whether or not there is sufficient evidence to prove that fact that Abarquez was an
accomplice in the killing of Ricardo Bello?

Decision:
No. Two elements must concur before a person becomes liable as an accomplice: (1)
community of design, which means that the accomplice knows of, and concurs with, the criminal
design of the principal by direct participation; and (2) the performance by the accomplice of
previous or simultaneous acts that are not indispensable to the commission of the crime. Mere
commission of an act, which aids the perpetrator, is not enough.
The cooperation that the law punishes is the assistance knowingly rendered, which cannot
exist without the previous cognizance of the criminal act intended to be executed. It is therefore
required in order to be liable as an accomplice; that the accused must unite with the criminal
design of the principal by direct participation.
The court held in one case that the mere presence of the accused at the crime scene
cannot be interpreted to mean that he committed the crime charged. In convicting Abarquez in
this case, the trial court and the Court of Appeals relied mainly on the testimony of Paz. Paz
testified that he was held by Abarquez on the shoulders, thus preventing him from helping
Quejong who was grappling with Almojuela.
Pazs testimony does not show that Abarquez concurred with Almojuelas criminal
design. "Tumigil" literally means "stop." Clearly, Abarquez was trying to stop Paz from joining
the fray, not from helping Quejong. Paz claims that he was only trying to talk to Almojuela.
However, Paz could not have been merely talking to Almojuela, as he tried to portray, because
Almojuela was already grappling with Quejong at that time. Paz interpreted Abarquezs action as
an attempt to prevent him from helping Quejong. His interpretation was adopted by the trial
court and sustained by the Court of Appeals. Yet, in his testimony, Paz admitted that while
restraining him, Abarquez was scolding or reprimanding him and telling him to stop. It was not
shown that Abarquez was stopping Paz from helping Almojuela. It is more likely that Abarquez
was trying to stop Paz from joining the fight. Abarquezs act of trying to stop Paz does not
translate to assistance to Almojuela.
143.
Facts:

Brillante v CA (G.R. Nos. 118757 & 121571)

Roberto Brillante, then a candidate for the position of councilor in Makati City held a
press conference where he accused Jejomar Binay, a candidate for mayoralty in Makati, and
Nemesio Prudente of plotting an assassination plot against Augusto Syjuco, another mayoral
candidate in Makati. Several journalists wrote articles regarding the same and an open letter was
published as well. Later, Binay and Prudente both filed libel charges against Brillante. The trial
court found Brillante guilty of four counts of libel, which decision the CA affirmed.
Issue:
Whether or not the act of libel charged against petitioner has prescribed when the
Information was filed before the trial court
Held:
No. Article 90 of the Revised Penal Code provides that the crime of libel shall prescribe
within one year. In determining when the one year prescriptive period should be reckoned,
reference must be made to Article 91 of the same code which sets forth the rule on the
computation the computation of the prescriptive periods of offenses which states that period of
prescription shall be interrupted by the filing of the complaint or information. In the case at bar, a
proceeding in the prosecutors office may terminate without conviction or acquittal.

144.

Chang v. People (G.R. No. 165111)

Facts:
Chang was the Municipal Treasurer of Makati who was tasked to examine or investigate
tax returns of private corporations and companies operating within Makati and determine the
sufficiency and insufficiency of the income tax assessed on them and collect payments, San
Mateo was

the Chief Operations, Business revenue Examination, Audit division, Makati

Treasurers office.
The examiners found that Group Developers, Inc. (GDI) incurred a tax deficiency
inclusive of penalty in the amount of P494,601.11. The assessment notice was received by Mario
Magat, Chief Operating Officer of GDI. Magat was later able to talk to San Mateo via phone. On
May 15, 1991, Magat and San Mateo met for lunch at the Makati Sports Club. Chang later joined
the two, the three agreed that if GDI could pay P125,000 by the end of May 1991, the assessment
would be resolved.

On June 6, 1991, Magat met again for lunch with San Mateo and Chang at the Makati
Sports Club. Magat tried to convince the two that GDI wanted to pay the correct amount of tax to
the municipality. He was advised by San Mateo and Chang, however, that GDI had only two
options: Pay the P494,601.11 to the municipality or P125,000 to them.
On June 12, 1991, Magat met with the NBI Deputy Director Epimaco Velasco who
advised him to file a complaint with the NBI. Magat thus gave a sworn statement. After several
days, Magat contacted San Mateo and asked him if their position was still the same to which the
latter said yes. Magat thereafter told San Mateo that he would deliver the P125,000 on June
19,1991 at the Makati Sports Club.
On June 19, 1991, Magat informed the NBI that payment was to be made that day around
lunchtime. The NBI formed a team to conduct an entrapment. The genuine money as well as the
boodle money and the envelope where the money was placed were then laced with fluorescent
powder.
San Mateo arrived and joined Magat at his table. Chang arrived and joined the two. Magat told
Chang and San Mateo that GDI was ready to pay and asked them if they could give him the
Certificate of Examination showing that GDI has no more tax liability. Chang handed the
Certificate while Magat gave the brown envelope. At that instant, the NBI agents announced that
they were being arrested. The Sandiganbayan convicted San Mateo and Chang of violation of sec
3(b) of R.A. No. 3019, otherwise known as Anti- Graft and Corrupt Practices Act. Hence, this
appeal.
Issue:
Whether or not there was a valid entrapment operation?
Decision:
Petitioners were undisputedly public officers at the time of the commission of the
offense. The prosecution, not only established creditably how the offense charged was
committed. It is established just as creditably how petitioners conspired to commit the crime.
There is entrapment when law officers employ ruses and schemes to ensure the
apprehension of the criminal while in the actual commission of the crime. There is instigation
when the accused is induced to commit the crime. The difference in the nature of the two lies in

the origin of the criminal intent. In entrapment, the mens reoriginates from the mind of the
criminal. The idea and the resolve to commit the crime comes from him. In instigation, the law
officer conceives the commission of the crime and suggests to the accused who adopts the idea
and carries it into execution.
From the evidence of the prosecution, it was clearly established that the criminal intent
originated from the mind of the petitioner. Even before the June 19, 1991 meeting took place,
petitioners already made known to Magat that GDI only had two options to prevent the closure
of the company, either to pay the assessed amount of P494,601.11 to the Municipality or to pay
the amount of P125,000 to them.

145.

De Guzman vs People (413 SCRA 215)

Facts:
The petitioner in this case is appealing from the decision rendered by Court of Appeals ordering
said petitioner to pay for the theft of jewelries amounting to P!, 500, 000 and moral damages of
P100,000 for the fear and trauma caused to the complainant.
The Court required the Office of the Solicitor General to comment. In the
manifestation it filed it stated that the petitioner passed away during the pendency of the appeal.

Issue:
What is the effect of the death of the on the petition?

HELD:
The Court cited Article 89 of the Revised Penal Code which states:
How criminal action is totally extinguished. Criminal liability is totally extinguished;
1. By death of the convict as to the personal penalties; and as to pecuniary penalties liability
therefore is extinguished only when the death of the offender occurs before the final
judgment;

That upon the death of the accused pending appeal of his conviction, the criminal action is
extinguished in as much as there is no defendant to stand as the accused. The civil action
instituted for the recovery of the civil liability ex delicto is extinguished grounded as it is on the
criminal.
83. Jovendo Del Castillo vs Hon. Rosario Torrecampo,
G.R. No. 139033. December 18, 2002.
Facts:
In Barangay Ombao, Camarines Norte the accused unlawfully conducted himself in a disorderly
manner, by striking the electric bulb and two (2) kerosene petromax lamps lighting the room
where voting center no. 24 is located, during the counting of the votes in said voting center
plunging the room in complete darkness, thereby interrupting and disrupting the proceedings of
the Board of Election Tellers.
He was declared guilty by the RTC. Petitioner appealed his conviction to the Court of Appeals
which eventually affirmed the decision of the trial court. Execution of judgment was scheduled
on October 14, 1987. His counsel motioned to reset the execution of judgment which was denied.
During the execution petitioner failed to appear. The court was prompted to issue an order for his
arrest. He was never apprehended and he remained at large.
Ten years later petitioner filed before the trial court a motion to quash the warrant issued for his
arrest on the ground of prescription of the penalty imposed upon him. He asserts that that the
period of prescription shall commence to run from the date when the culprit should evade the
service of his sentence. The Court of Appeals, in its interpretation of the said provision, engaged
in judicial legislation when it added the phrase by escaping during the term of the sentence.
Issue:
Whether the penalty imposed upon the petitioner has prescribed?
Held:
The Court held, that in order for prescription to take place the following elements must concur:
1.) That the penalty is imposed by final sentence; 2.) That the convict evaded the service of the
sentence by escaping during the term of his sentence; 3.) That the convict who escaped from
prison has not given himself up, or been captured, or gone to a foreign country with which we
have no extradition treaty or committed another crime; 4.) That the penalty has prescribed,
because of the lapse of time form the date of the evasion of the service of the sentence by the
convict.
From the elements, it is clear that the penalty imposed has not prescribed because the
circumstances of the case failed to satisfy the second element, That the convict evaded the
service of the sentence by escaping during the service of his sentence. As a matter of fact, the
petitioner never served a single minute of his sentence.
146.

Nueva Espana v. People (460 SCRA 547)

Facts:
Petitioner Nueva Espana was found guilty of reckless imprudence resulting into double
homicide when the passenger bus he was driving rammed into a Honda motorcycle driven by
Reynard So with Nilo Castro as passenger resulting into the death of both. An aggravating
circumstance was also imposed as Espana also left the scene of the crime without lending
assistance to the victims.
During trial, the father of So and the mother of Castro were both called on to testify as to
the earning capacity of the two. Sos father claimed that his son was earning P80,000 a month
while Castros mother said that his son was bringing in P8,000 a month. Sos father additionally
testified that the funeral expenses incurred by them was P87,000 while Castros mom stated that
they spent P30,000 for the funeral.
As a result the trial court, besides imprisonment, awarded the following amounts to the
heirs of the victims:

TO THE HEIRS OF THE VICTIM REYNARD SO


1)
P2,997,000.00 indemnity for loss of earning capacity of victim
2)
14,200.00 for expenses of the wake
3)
20,000.00 for funeral parlor
4)
12,000.00 for the tomb
5)
53,000.00 for cost of burial site
6)
30,000.00 for attorneys fees
7)
200,000.00 for moral damages
8)
100,000.00 for exemplary damages
P3,429,200.00 TOTAL AMOUNT

TO THE HEIRS OF VICTIM NILO CASTRO

1)
P1,728,000.00 indemnity for loss of earning capacity
2)
20,000.00 for funeral expenses
3)
200,000.00 for moral damages
4)
50,000.00 for exemplary damages
P1,998,000.00 TOTAL AMOUNT

The court based the amount of loss of earning capacity based on the formula used by the
Supreme Court as illustrated:
As to the civil liability, particularly the indemnity for the loss of the earning capacity of
the victims, the formula last enunciated by the Supreme Court is:

Net earning capacity (x) = life expectancy x gross-living expenses annual


(50% of gross annual income)
Thusly, since the victim Reynard So was earning P80,000 a month at the time of his death
when he was thirty (30) years old, his lost earning capacity should be computed as follows:
x = 2 (80 30) x [P960,000.00 P480,000.00)
3
x = 33.4 x P480,000.00
x = P16,032,000.00
With respect to the victim Nilo Castro, he was earning P8,000.00 a month when he died
at the age of twenty-six (26). His lost earnings were:
x = 2 (80 26) [P96,000.00 P48,000.00]
3
x = 36 x P48,000.00
x = P1,728,000.00
As a result, petitioner appeals to the CA but the appellate court affirmed the decision of the
trial court regarding the damages, Consequently, the CA declared that Vallacar Transit Inc.,
should not yet be held subsidiary liable for the liability of the petitioner as its driver. Thus, this
petition for review with the SC.
Issue:
Was the award of damages amounting to P8 million proper?
Decision:
The SC modifies the award of damages mostly to the fact that loss of earning capacity
should be properly adduced and supported by competent evidence to prove the same. This rule
also applies to the funeral and burial expenses. In the case at bar, the lower courts based their
award for damages solely on the testimony of SOs father and Castros mother, even though both
of them never substantiated the amounts claimed with receipts, papers and other evidence. And
so the award is modified as follows: To summarize, the heirs of the deceased Reynard So are
entitled to the following:

P 50,000 civil indemnity ex delicto


73,000 actual damages
25,000 temperate damages
50,000 moral damages
25,000 exemplary damages
30,000 attorneys fees
P 253,000 TOTAL
The heirs of Nilo Castro are also entitled to the following:
P 50,000 civil indemnity ex delicto
50,000 temperate damages
50,000 moral damages
25,000 exemplary damages
30,000 attorneys fees
P 205,000 TOTAL
The SC meanwhile adopts the pronouncement of the Court of Appeals regarding the
subsidiary liability of petitioners employer, Vallacar Transit Inc., under Article 103 of the
Revised Penal Code. An employer may be subsidiarily liable for the employees civil liability in
the criminal action if it can be shown that: (1) the employer is engaged in any kind of industry;
(2) the employee committed the offense in the discharge of his duties and (3) the accused is
insolvent. However, subject to prevailing jurisprudence, the subsidiary liability may be enforced
only upon a motion for subsidiary writ of execution against Vallacar Transit, Inc. and upon proof
that petitioner is insolvent.

147.

People of the Philippines vs Abungan

Facts:
Pedro Abungan was convicted for the crime of murder in the Regional Trial Court of VIllasis
Pangasinan for the murder of Camilo Dirilo. Through appellants counsel a notice of appeal was
filed while he was at the same time committed in the National BIlibid Prison. Pending the appeal
of his case the assistant director of the Bureau of Corrections informed the court that appellant
had died in the National Bilibid Prison Hospital.
Issue:

What is the effect of death of appellant Abungan on the case and on the appeal?
Held:
The consequence of appellants death is provided in the Revised Penal Code in Article 89. By
death of the convict before final judgment, as to personal liabilities and as to pecuniary liabilities
are extinguished.
The death of the appellant would result in the dismissal of the case against him. Necessarily, the
lower courts decision finding him guilty and sentencing him to suffer imprisonment and to
indemnify the heirs of the deceased becomes ineffectual.

148.

People v. Callet (G.R. No. 135701)

Facts:
Elbert S. Callet was charged and found guilty of the crime of Murder in the death of
Alfredo Senador. Callet used a 9-inch hunting knife in stabbing the latter on the left shoulder
near the base of the neck causing Senadors death shortly thereafter.

Callet appealed his

conviction claiming that the Regional Trial Court of Negros Oriental, Dumaguete City (Branch
30) gravely erred in failing to consider the mitigating circumstance of the fact that he had no
intention to commit so grave a wrong therefore his liability should be mitigated.
Issue:
Whether or not the criminal liability of Callet be mitigated in that he had no intention to
commit so grave a wrong?
Decision:
The Supreme Court ruled in the negative. The lack of intent to commit a wrong so
grave is an internal state. It is weighed based on the weapon used, the part of the body injured,
the injury inflicted and the manner it is inflicted. The fact that the accused used a 9-inch hunting
knife in attacking the victim from behind, without giving him an opportunity to defend himself,
clearly shows that he intended to do what he actually did, and he must be held responsible
therefore, without the benefit of this mitigating circumstance.

149.

People v Dansico (644 SCRA 151)

Facts:
Sometime on September 7, 1998 at about 4:30 o'clock in the afternoon, at Brgy. MayAnao, Tigaon, Camarines Sur, Romeo Dansico a.k.a. Lamyak and Augusto Cuadra conspired
and confederated with one another to attain a common purpose which is to feloniously without
authority of law to sell and deliver one piece of marijuana brick wrapped in newspaper weighing
approximately (900) grams for and in consideration of five thousand pesos.

Issue:
Were the accused instigated to sell marijuana?

Held:
No. Instigation means luring the accused into a crime that he, otherwise, had no intention
to commit, in order to prosecute him. On the other hand, entrapment is the employment of ways
and means in order to trap or capture a lawbreaker. Instigation presupposes that the criminal
intent to commit an offense originated from the inducer and not the accused who had no
intention to commit the crime and would not have committed it were it not for the initiatives by
the inducer. In entrapment, the criminal intent or design to commit the offense charged originates
in the mind of the accused; the law enforcement officials merely facilitate the apprehension of
the criminal by employing ruses and schemes.
In instigation, the law enforcers act as active co-principals. Instigation
leads to the acquittal of the accused, while entrapment does not bar prosecution and conviction.
150.
People of the Philippines vs Orlando Guerrero Sr., and Orlando Guerrero Jr.
(389 SCRA 389)
Facts:
From the decision of the Regional Trial Court of La Union the accused Orlando Guerrero
Jr. a.k.a. Pablo appeals before the Court in its findings in rendering him guilty beyond
reasonable for the crime of murder. Sometime in January 1997 when appellant was on his way
home from work he saw Ernesto Ocampo. At around 9 a.m. he had breakfast with his family then
slept being still tired from the night shifts work.

He was roused from his sleep by someone calling his name outside the kitchen door. He rose and
proceeded to the door. Then suddenly, the door was kicked open, and there he saw his godfather,
Ernesto Ocampo a.k.a. George. Ernesto immediately put his left foot inside appellants house.
Ernesto wanted to know the whereabouts of appellants sister(Georges mistress), Nora.
Appellant replied that he did not know where she was. Ernesto then warned that if ever appellant
could not present Nora to him, blood will be spilled around their house.
Ernesto suddenly rushed towards appellant and lunged at him and while doing so, drew
out a knife from his waist. Appellant immediately reached for a wooden club they kept beside the
door, usually used to secure the door at night, as its knob had long ago been broken. Using the
club, appellant struck Ernesto on the head, which sent Ernesto reeling and caused him to step
backward. Appellant struck again, this time causing Ernesto to fall down on the porch of the
house. After Ernesto fell, appellant got Ernestos knife and used it to slash his neck, to the point
of completely severing the head from the body. He then proceeded to cut off Ernestos penis.
Pablo placed Ernestos head beside his body, and then went out of the house.
The information alleged the qualifying circumstances of treachery and evident
premeditation. It also stated that there was cruelty in the perpetration of the crime, where there
was deliberate and inhuman suffering of the victim and the offender had scoffed at the victims
corpse.
Issue:
Whether the trial court erred in appreciating the circumstance cruelty and scoffing at and
outraging or outraging at the victims court?
Held:
Based on the sequence of events, cruelty has to be ruled out for it connotes an act of deliberately
and sadistically augmenting the wrong by causing another wrong not necessary for its
commission, or inhumanely increasing the victim's suffering. Ernesto died as a result of his head
being severed. No cruelty is to be appreciated where the act constituting the alleged cruelty in the
killing was perpetrated when the victim was already dead.

151.

People of the Philippines v Pacana (345 SCRA 72)

Facts:
All the accused in this case were charged for frustrated murder of Raul Leyson and the murder of
Felizardo de Solo. The trial court rendered a decision convicting all of them. In view of such
decision all of the accused appealed.
During the pendency of their appeal, appellants

Vicente, Bernardo and Virgilio, who are confined at the National Bilibid Prison, filed a motion to
withdraw their appeal. After confirming the voluntariness of their withdrawal of appeal, accusedappellant Bernardo, on the other hand, was required to confirm the voluntariness of his motion to
withdraw appeal. However, a return from the Bureau of Corrections shows that he died.
Issue:
What is the effect of the death of the accused on his liability?
Held:
The death of the accused extinguishes his criminal liability even if his death should occur during
the pendency of his appeal. Bernardos death not only extinguished his criminal liability
concerning the personal penalties but also whatever pecuniary penalties have been imposed on
him, considering that he died before final judgment.
Article 89 (1) of the Revised Penal Code states that: Criminal liability is totally
extinguished by the death of the convict, as to the personal penalties; and as to pecuniary
penalties, liability therefor is extinguished only when the death of the offender occurs before
final judgment.
152.

People of the Philippines vs Bayotas

Facts:
Rogelio Bayotas was charged with rape and was eventually convicted. Pending appeal of his
conviction Bayotas died in the National Bilibid Prison Hospital. Consequently the Supreme
Court dismissed the criminal aspect of the appeal. Court required the OSG to comment with
regard to the civil liability of Bayotas arising from his commission of the offense charged.
The OSG invoked People vs Sendaydiego, wherein the
Court ruled that civil liability of the therein accused survives as a result of his commission of the
offense charged. On the other hand, counsel for the accused-appellant Bayotas invokes People v
Castillo-Ocfemia where the Court held that upon death of the accused before final judgment civil
liability arising from the criminal offense is extinguished.

Issue:
Does death of the accused pending appeal of his conviction extinguish his civil liability?

Held;
Yes. The death of the appellant extinguished his criminal and civil liability based on the act
complained of, i.e. rape. Consequently the appeal is dismissed. Explaining that reliance on the
decision of the Court in People vs Sendaydiego is misplaced. There is a strained implication
therefrom that where the civil liability instituted together with the criminal liabilities had already
passed beyond the judgment of the Court of First Instance, that Court of Appeals can continue to
exercise appellate jurisdiction thereover despite the extinguishment of the component criminal
liability of the deceased.
153.

Panaguiton v. DOJ (G.R. No. 167571)

Facts:
Cawili and his business associate Tongson borrowed from Panaguiton (petitioner) sums
amounting to 1,979,459. They issued checks signed by both of them to Panaguiton but these were
dishonored upon presentation. Panaguiton made demands to pay but to no avail. He formally filed
a complaint on August 24, 1995 for violating BP 22 before the City Prosecutors Office.
Tongson moved to drop his name from the case as his signatures were allegedly
falsified. Case against him was dismissed but afterwards upon finding that Tongson might have
indeed signed the checks, the chief state prosecutor directed the city prosecutor to conduct a
reinvestigation. Tongson moved for reconsideration but denied.
In 1999 assistant prosecutor dismissed the complaint for the action has prescribed pursuant
to Act 3326, which provides for the prescriptive periods of statutes without their own (4 years for
BP22). She claims that the filing of the complaint on August 24, 1995 did not interrupt the running
of the period as the law refers to judicial and not administrative proceedings.
Issue:
Whether or not the filing of the complaint in the prosecutors office tolled the prescriptive
period?
Decision:
Yes. Filing of the complaint in the prosecutors office tolls the prescriptive period for
violations of BP22. When Act 3326 was passed into law, preliminary investigation of cases was
done by the justices of peace, and not by agents of the executive department (i.e. prosecutors).

Thus, the prevailing rule at that time is that prescription is tolled once filed with the justice of
peace (a judicial process). However, since then, the conduction of a preliminary investigation
was moved to the function of the executive department. Today, the term proceedings must be
understood to mean either executive or judicial proceedings. With this interpretation, any type of
investigation may ultimately lead to sufficiently toll prescription. To rule otherwise would
deprive the injured party the right to obtain vindication on account of delays not under his
control. As seen in this case, various conflicting opinions of the DOJ delayed his cause.
Aggrieved parties who do not sleep on their right should not be allowed to suffer simply because
of circumstances beyond their control.

154.

Pangan v Gatbatile

Facts:
On September 26, 1987 the petitioner was convicted of the offense charged and was
sentenced to serve a penalty of the offense charged and was sentenced to serve a penalty of two
month is and one day of arresto mayor. On appeal, the Regional Trial Court, affirmed the
decision of the MTC. Petitioner never got to serve his sentence and hid for nine years.
Then, on January 20, 2000 petitioner was apprehended and detained at the
Mabalacat Detention Cell. Four days thereafter, he filed a petition for a writ of habeas courpus at
the RTC of Angeles City, impleading the acting chief of police of Mabalacat, Pampanga.
Petitioner contended that his arrest was illegal and the straight penalty of two months and one
day of arresto mayor prescribes in five years under Article 93 of the RPC and having been able to
continue to evade service of sentence for almost nine years, his criminal liability has long been
extinguished under Article 89 of the RPC. The petition for writ of habeas corpus was denied
since there was no evasion of service of sentence. Evasion presupposes escape during the service
of the sentence consisting in deprivation of liberty.
Issue:
Whether or not the penalty prescribed?
Held:
No. The period of prescription of penalties in Article 93 of the RPC provides that it shall
commence to run from the date when the culprit should evade service of his sentence. Article
157 discussed how evasion of service of sentence was perfected. It is provided therein that the
penalty of prision correccional in in it medium and maximum periods shall be imposed upon any
convict who shall evade service of his sentence by escaping during the term of his imprisonment
by reason of final judgment. To consider properly the meaning of evasion of service of sentence,

its elements: (1) the offender is convict by final judgment; (2) he is serving his sentence which
consists in deprivation of liberty; and (3)he evades service of sentence by escaping during the
term of his imprisonment by reason of final judgment.
That escape should take place while serving sentence, is emphasized by the
second sentence of Article 157. It provides a higher penalty if such evasion or escape shall have
taken place by means of unlawful entry, by breaking doors, windows, gates, walls, roofs or
floors, or by using picklocks, false keys, disguise, deceit, violence or intimidation or through
connivance with other convicts or employees of the penal institution. Indeed, evasion of sentence
is but another expression of the term jail breaking. As pointed out by the Solicitor General,
escape in legal parlance and for purposes of Articles 93 and 157 of the RPC means the
unlawful departure of prisoner from the limits of his custody. Clearly, one who has not
committed cannot be said to have escaped therefrom.
155.

People of the Philipiines vs Rex Nimuan (650 SCRA 597)

Facts:
Alfredo Nimuan was on his way home walking along a path inside a mango plantation in
the barangay, he spotted the appellant and the victim about 30 meters ahead of him, walking in
the same trail leading to their respective houses. Unaware of his presence, the appellant who
was walking a meter behind the victim suddenly hacked the latter with a bolo. Alfredo ran
away to seek help when he saw the victim fall to the ground after the attack.
Nimuan claimed that between 3:00 and 5:00 p.m. of July 22, 2004, he was watching
television at the house of his uncle, when a certain Barangay Captain Cario, along with a
barangay kagawad, arrived and informed him that he was a suspect in the death of the victim.
The appellant and his mother went with the barangay officials to the police station.
The RTC declared Nimuan guilty of the
crime of murder appreciating treachery because the victim was hacked by surprise leaving the
latter no opportunity to defend himself and appreciated the mitigating circumstance of voluntary
surrender. On appeal the CA affirmed the decision of the trial court however it found that latter
erred in appreciating the mitigating circumstance of voluntary surrender because the appellant
went with the barangay officials not to admit the alleged crime or to voluntarily surrender to the
authorities, but only for verification purposes.
Issue:
Whether mitigating circumstance of voluntary surrender be appreciated against
appellant?

Held:
No. The mitigating circumstance of voluntary surrender cannot be appreciated in his
favor; the records indicate that the appellant did not intend to assume responsibility for the death
of the victim when he and his mother went with the barangay officials to the police station.
156.

People of the Philippines vs Alfredo Taruc (171 SCRA 75)

Facts:
Federico Sanchez a.k.a. Manuela was on his way home from a gay dinner dance
when Alfredo Taruc asked him to come with to his house. He was surprised when he saw Efe
the fellow accused in the original case strangling one Luis Martinez with a cord and was forcing
banana peelings into his mouth, looking helpless and was not offering much of a struggle. Efe
demanding for the whereabouts of Boy Baba who had stabbed him a few weeks ago.
Next thing or before Manuela knew, he was likewise being strangled with a cord but
when Taruc saw that Efe needed help being weak from the wounds he sustained a few weeks ago
went to assist Efe. Then, Manuela took his chance to escape.
Issue:
Whether the accused is guilty of Murder?
Held:
Yes. The Court ruled with the qualifying circumstance of evident premeditation, which
has been established. Martinez was undoubtedly lured into Taruc's house with the intention of
strangling him with the nylon cord prepared for the purpose. The crime was attended by the
generic aggravating circumstance of cruelty as manifested by the stab wounds inflicted on
Martinez and the perversion of forcing banana peelings into his mouth while he was being
strangled.
157.

People of the Philippines vs Whisenhunt (358 SCRA 586)

Facts:
The accused in this case Stephen Whisenhunt is the lover of the victim Elsa Castillo.
Both are married and estranged to their respective spouses. Likewise, both are colleagues in
Apex Motor Incorporated. Sometime in September 23,1993 the accused instructed his driver to
fetch Elsa in their house in Blumentritt, Manila. The driver named Demetrio Ravelo found Elsa

standing at a corner near her parents house. He brought her to the condominium of the accused.
The next day, the housemaid, Lucy, was looking for the kitchen knife and asked the driver where
it was, the accused overheard and answered that it was in his room.
On September 25, 1993, Demetrio asked for permission from the accused that he will
follow up his salary in Apex office. When he returned to the condominium the accused instructed
him to send Lucy to their house to get her clothes because they are going to the Bagac, Bataan
the next day.
The following day while Demetrio was waiting at the servants quarters the accused
asked him how long did he want to work for him, to which he answered with forever and
expressed his full trust in him. Upon hearing that the accused said that he had a problem, that
Elsa was dead due to hemorrhage in her pancreas or bangungot.
Demetrio suggested to have the body autopsied but the accused refused. Instead they
went to Shopesville, Greenhills and bought a big bag with a zipper and rollers wherein they
wrapped dismembered hands, feet, trunk and head of a woman which Demterio noticed as Elsas
face. Then they loaded the bags in the car and drove to Batangas and passed by the South
Expressway and Laguna when the accused instructed the driver to dump the bag in the roadside.
En route to Bagac, Bataan they made several stops where other paraphernalias of the victim
were dumped. The information alleged abuse of superior strength , treachery, and cruelty.
Issue:
Whether the accused is guilty of the crime of murder?
Held:
Yes. Even if treachery was not present in this case, the crime would still be murder because of
the dismemberment of the dead body. One of the qualifying circumstances of murder under
Article 248, par.6, of the Revised Penal Code is outraging or scoffing at person or corpse of the
victim. However, the other circumstance of outraging and scoffing at the corpse of the victim
was correctly appreciated. The mere decapitation of the victims head constitutes outraging or
scoffing at the corpse of the victim, thus qualifying the killing to murder. In this case, accused
not only beheaded Elsa. He further cut up her body like pieces of meat. Then, he strewed the
dismembered parts of her body in a deserted road in the countryside, leaving them to rot on the
ground.

158.

People V. Antonio

Facts:
The
facts as presented by the prosecution show that at 1:00 in the early morning of October 11, 1998,
the victim, Jomar Ephan, was engaged in a drinking session with Reynaldo Ephan and Roselito
Dacillo in front of a store in Barangay Pakna-an, Mandaue City. Accused-appellant arrived and
bought cigarettes. Then, he ordered Jomar, Reynaldo and Roselito to count the cigarettes he
bought, but the three told accused-appellant to let the storekeeper do the counting. Rebuked,

accused-appellant left the store. He returned minutes later and suddenly stabbed the victim at the
back, after which he immediately fled. The victim was rushed by his companions to the hospital
but died the following day.
Meanwhile,
Eduardo Juban, a Barangay Tanod, was awakened by one of his neighbors and was told that there
was trouble at a nearby store. When Eduardo went out, he saw accused-appellant being chased
by a crowd who were shouting, "thief." The group mauled accused-appellant when they caught
up with him. Eduardo, however, pacified the mob and brought accused-appellant to the barangay
hall. Eduardo later learned from the group that accused-appellant had stabbed somebody.
On the other hand, the defense tried to prove that: at around
1:00 a.m. of October 11, 1998, accused-appellant was in the house of his friend, Fernando Gelig,
at Pakna-an, Mandaue City. While they were drinking liquor, accused-appellant went out and
bought cigarettes from a store across the street. As a token of friendship, accused-appellant
offered the cigarettes to the people in front of the store, but nobody accepted his offer. Accusedappellant went back to the house of his friend. After a short while, he went back to the same store
to buy "pulutan." For no reason at all, somebody struck him with a stool hitting him on the left
eyebrow. Accused-appellant fell on the ground but the group of the deceased, who were then in
front of the store, ganged up on him. The deceased attempted to hit accused-appellant but
because the former was very drunk, he missed and fell on his belly. It was at this point when
accused-appellant got hold of a knife he saw under the table and stabbed the deceased at the
back. Thereafter, accused-appellant immediately fled but the crowd chased and mauled him.
Fortunately, a Barangay Tanod came and stopped the mob. The RTC ruled in favor of the victim
and found the accused guilty of murder. Hence, he appealed the said decision.
Issue:
Whether the lower court failed to appreciate the justifying circumstance of self-defense in
favor of the accused
Held:
In the case at bar, even if the Court will sustain the version of accused-appellant
that the initial act of aggression came from the group of the deceased, still we cannot uphold his
plea of self-defense. As testified by accused-appellant himself, the deceased who was at that time
very drunk tried to hit him but missed and fell on the ground. At that point, unlawful aggression
ceased and it was no longer necessary for him to stab the deceased. It was accused-appellant,
therefore, who became the aggressor when he, despite the condition of the deceased, proceeded
to stab the latter at the back. His act can no longer be interpreted as an act of self-preservation
but a perverse desire to kill. Hence, he cannot successfully claim the benefit of self-defense.
Furthermore, if it were true that the companions of the deceased ganged up on him, his attack
should have been directed against them and not against the deceased who was already
defenseless and lying on the ground.

159.

Facts:

People v. Bates (G.R. No. 139907)

Around 2:00 in the afternoon of November 28, 1995, Edgar Fuentes, Simon Fuentes and
Jose Boholst left Barangay Esperanza, Ormoc City to deliver copra to a certain Fely Rodado at
Barangay Green Valley, Ormoc City. After delivering copra around 5:00 in the afternoon, the
three men headed back to Barangay Esperanza. While they were along a trail leading to the
house of Carlito Bates, the latter suddenly emerged from the thick banana plantation surrounding
the trail, aiming his firearm at Jose Boholst who was then walking ahead of his companions.
Jose grabbed Carlitos right hand and elbow and tried to wrest possession of the firearm. While
the two were grappling for possession, the gun fired, hitting Carlito who immediately fell to the
ground. At that instant, Marcelo Bates and his son Marcelo Bates, Jr., brother and nephew of
Carlito, respectively, emerged from the banana plantation each brandishing a bolo. They
immediately attacked Jose hacking him several times. Jose fell to the ground and rolled but
Marcelo and his son kept on hacking him. Marcelo, then, turned to Simon and Edgar and
shouted huwes de kutsilyo. Upon hearing the same, Simon and Edgar ran. Upholding the
prosecution evidence, the trial court rendered its Judgment, finding Marcelo Bates guilty beyond
reasonable doubt of the crime of Murder.

Issue:
Whether or not Marcelo could validly invoke the mitigating circumstance of passion and
obfuscation?

Decision:
Passion and obfuscation may not be properly appreciated in favor of appellant. To be
considered as a mitigating circumstance, passion or obfuscation must arise from lawful
sentiments and not from a spirit of lawlessness or revenge or from anger and resentment. In the
present case, clearly, Marcelo was infuriated upon seeing is brother, Carlito, shot by Jose.
However, a distinction must be made between the first time that Marcelo hacked Jose and the
second time that the former hacked the latter. When Marcelo hacked Jose right after seeing the
latter shoot at Carlito, and if appellant refrained from doing anything else after that, he could
have validly invoked the mitigating circumstance of passion and obfuscation. But when, upon
seeing his brother Carlito dead, Marcelo went back to Jose, who by then was already prostrate on
the ground and hardly moving, hacking Jose again was a clear case of someone acting out of
anger in the spirit of revenge.
160.

People v. Calongui (G.R. No. 170566)

Facts:
Calonqui was found guilty for two counts of rape. On January 1, 1998 about 2 am in
Tagbong, Camarines Sur, Calonqui was able to rape the 13 year old girl Maricel in the latters
house. On September 26, 1998 at about three in the morning, the accused again raped the victim.
Both rape incidents were witnessed by the brother of Maricel.
Issue:
Whether or not the aggravating circumstances of dwelling and relationship be appreciated
against Calonqui and the latter circumstance as an alternative circumstance?

Decision:
Calonqui and Maricel live under the same shelter as they are first cousins. At the time of
the incident, both are living in the same house and in the same room. Therefore, the supposed
aggravating circumstance of dwelling cannot be appreciated as there was no trespass to the
sanctity of the house of the victim on the part of Calonqui, while the aggravating circumstance of
relationship is likewise cannot go against Calongui, even as an alternative circumstance, as being
first cousins is not within the concept contemplated in Article 15 of the Revised Penal Code.
However, his conviction is nonetheless affirmed.

161.

People v. Concepcion (386 SCRA 74)

Facts:
The accused Concepcion is a police officer charged with murder for the killing of one
Lorenzo Galang. According to testimonies of both parties witnesses, Lorenzo Galang was
brought to the barangay hall because he was so drunk and unruly at the town plaza and was
continually disturbing the peace.

The accused then came to the barangay hall apparently to question Galang. But herein
lies the differences in the testimonies. The prosecution witnesses (2 of them) testified that while
interrogating Galang, Concepcion suddenly fired two shots past the ear of the victim without
injuring him. But later on, he hit the victim in the abdomen and fired a shot which wounded
Galang in the thigh and then Concepcion fired three more shots which hit the victim in the chest
and killed him.
But according to the accused, he was merely pacifying Galang when the victim became
so unruly that the accused fired two warning shots. But instead of scaring Galang, the latter tried
to grab the gun from the accused. Two shots were accidentally fired which hit Galang thus
causing his death.
The accused claims that he should be exempted because he was just performing his
lawful duty as a police officer and that the shooting was purely accidental. The trial court found
Concepcion guilty.
Issue:
Should the accused be exempted from criminal liability due to accident?
Decision:
Well settled is the rule in criminal cases, that the prosecution has the burden of proof to establish
the guilt of the accused. However, once the defendant admits the commission of the offense
charged, but raises an exempting circumstance as a defense, the burden of proof is shifted to
him. By invoking mere accident as a defense, appellant now has the burden of proving that he is
entitled to that exempting circumstance under Article 12 (4) of the Code.
Unfortunately for the accused, his testimony was too full of inconsistencies which failed to
discharge the burden . For one, Concepcion claims that when the victim tried to grab his gun,
said rifle was hanging on his shoulder on a swivel. But then he claimed that Galang tried to rest
the rifle away by grabbing the BARREL OF THE GUN. This was very inconceivable.
Furthermore, it was not believable that a person so drunk would try to take away a rifle from a
police officer who also had a handgun tucked by his waist.
Lastly, the prosecution witness categorically testified that he saw Concepcion shoot the victim
with the M-16 rifle. And so, the finding of guilt by the lower court was proper.

162.

People v. Dacillo (G.R. No. 149368)

Facts:
Appellant was convicted by the trial court of the crime of murder for the death of
Rosemarie Tallada, with aggravating circumstance of recidivism with no mitigating circumstance
to offset the same, and sentenced to the extreme penalty of death.
In his defense, appellant admitted complicity in the crime but minimized his
participation. Appellant alleged that he only held down Rosemaries legs to prevent her from
struggling and, after the latter was killed by another man he identified as Joselito Pacot, he
encased the corpse in cement.
He claimed that Pacot, a co-worker at Davao Union Cement Corporation (DUCC), was
looking for a house where he and his girlfriend Rosemarie could spend the night. He offered his
brothers house which was under his care. In the evening of February 6, 2000, he and Joselito
Pacot brought Rosemarie to the house at Purok No. 3, New Society Village, Ilang, Davao City.
After accompanying the couple there, he went home to take supper. Later that evening, he
returned to the house with the bottle of Sprite Pacot had ordered. When he arrived, Pacot and
Rosemarie were already grappling with each other and Pacot was strangling the girl. He told
Pacot to stop but instead of heeding him, the latter ordered him to close the door. Pacot told
appellant that he was going to be implicated just the same so he closed the door as ordered and
helped Pacot "(hold) the feet of the woman" as "her feet kept hitting the walls."
The two men stopped only when Rosemarie was already motionless. Pacot wanted to
dump the body into the sea but appellant told him it was low tide. Appellant then suggested that
they entomb the body in cement for which Pacot gave appellant P500. Pacot left the house at
dawn the following day, February 7, 2000. At past 10:00 a.m., appellant brought the concrete
mixture and cast the dead body in cement.

Issue:
Whether or not appellant is liable as a principal?
Decision:
The rule is that any admission made by a party in the course of the proceedings in the
same case does not require proof to hold him liable therefor. Such admission may be contradicted
only by showing that it was made through palpable mistake or no such admission was in fact
made. There was never any such disclaimer by appellant.
Moreover, despite appellants self-serving, exculpatory statement limiting his
involvement in the crime, all circumstances pointed to his guilt. Assuming for the sake of
argument that Pacot was the mastermind, appellants admission that he participated in its
commission by holding Rosemaries legs made him a principal by direct participation.
Two or more persons taking part in the commission of a crime are considered principals
by direct participation if the following requisites are present: 1.) they participated in the criminal
resolution and 2.) they carried out their plan and personally took part in its execution by acts
which directly tended to the same end.
Both requisites were met in this case. Two or more persons are said to have participated
in the criminal resolution when they were in conspiracy at the time of the commission of the
crime. To establish conspiracy, it is not essential that there be proof of the previous agreement
and decision to commit the crime, it being sufficient that the malefactors acted in concert
pursuant to the same objective.
It is well-settled that a person may be convicted for the criminal act of another where,
between them, there is conspiracy or unity of purpose and intention in the commission of the
crime charged. Conspiracy need not be proved by direct evidence of prior agreement on the
commission of the crime as the same can be inferred from the conduct of the accused before,
during, and after the commission of the crime showing that they acted in unison with each other
pursuant to a common purpose or design.
163.

Facts:

People V. Dequina

Accused-appellants Dequina et al were charged before the RTC-Manila with violations


of the offense of illegal transport of marijuana under the Dangerous Drugs Act of 1972.
Police Officer III Wilfredo Masanggue testified that at about 6:00 a.m., of September
29,1999, he and SPO1 Anthony Blanco were instructed by their superior to proceed at the corner
of Juan Luna and Raxabago Sts., Tondo, Manila, where, according to the report given by the
informant, three persons a male and two female[s] would be coming from Baguio City to deliver
unknown quantity of marijuana.
At around 9:00 a.m., they noticed a taxi cab coming from Yuseco St. heading towards the
direction of the pier. From it emerged three passengers a man and two women each one of them
carrying a black travelling bag.
As the trio started walking towards the western portion of Raxabago St., they drove and trailed
them. As the patrol car got closer behind them, [Dequina] noticed its presence. She started
walking in a more hurried pace. While trying to get away, [Dequina] dropped the bag she was
carrying. As a result, the zipper of the bag gave way. Bundles of dried leaves wrapped in
transparent plastic bags case into view. Thus, they arrested the three accused.
During the investigation, it was discovered that each of the three black travelling bags
confiscated from the three accused contained eleven bricks of marijuana. Accused-appellants
assail their conviction, asserting that their arrests were illegal. They were not doing anything
illegal that would have justified their warrantless arrest, much less a warrantless search of their
persons and belongings. A search made without a warrant cannot be justified as an incident of
arrest unless the arrest itself was lawful.
In order to exonerate herself from criminal liability, Dequina contends that she
transported the marijuana under the compulsion of an irresistible fear. Jundoc and Jingabo, on
the other hand, claim that they went along to accommodate Dequina, a trusted childhood friend.
Issue:
Whether or not the accused should be acquitted based on a compulsion of an irresistible
fear.
Held:
A person who acts under the compulsion of an irresistible force, like one who acts under
the impulse of an uncontrollable fear of equal or greater injury, is exempt from criminal liability
because he does not act with freedom. Actus me invito factus non est meus actus. An act done
by me against my will is not my act. The force contemplated must be so formidable as to reduce
the actor to a mere instrument who acts not only without will but against his will. The duress,
force, fear or intimidation must be present, imminent and impending, and of such nature as to
induce a well-grounded apprehension of death or serious bodily harm if the act be done. A threat
of future injury is not enough. The compulsion must be of such a character as to leave no
opportunity for the accused for escape or self-defense in equal combat. Here, Dequinas version
of events that culminated with her and Jundoc and Jingabos arrests on September 29, 1999 is
implausible. Equally far-fetched is Jundoc and Jingabos assertion of blind trust in Dequina and
total ignorance of the transportation of marijuana.

164.

People v. Marcos (G.R. No. 132392)

Facts:
Prosecution, with the testimony of a lone eye-witness, who happened to be the son of the
victim, along with the admission of guilt, found Cesar Marcos guilty beyond reasonable doubt of
Murder for the killing of his elder brother, Virgilio, as aggravated by the qualifying circumstance
of evident premeditation.
During Appeal, the Solicitor General insisted that since the accused is a brother of the victim,
the alternative circumstance of relationship must be considered in determining the imposable
penalty.
Issue:
Whether or not the alternative circumstance of relationship shall be considered in the
imposition of the proper penalty?
Decision:
In order that the Alternative Circumstance of relationship may be taken into consideration
in the imposition of the proper penalty, Paragraph 2 of Article 15 of the Revised Penal Code
provides that the offended party must either be the (a) spouse, (b) ascendant, (c) descendant, (d)
legitimate, natural or adopted brother or sister, or (e) relative by affinity in the same degree, of
the offender.
Relationship is Mitigating in Crimes against Property, while it must be considered as
Aggravating in Crimes against Persons where the offended party is a relative of a higher degree
than the offender or when in the same degree or level, as in brothers. Therefore, the Alternative
Circumstance of relationship shall be considered as Aggravating.

165.

People v. Mariano (G.R. No. L-40527)

Facts:
That on or about and during the period from May 11 and June 8, 1971, in San Jose del
Monte, Bulacan, the said accused Hermogenes Mariano, being then appointed as Liaison Officer
by the then incumbent Municipal Mayor, Constantino Nolasco, acting for and in behalf of the
municipality and authorized to receive and be receipted for US excess property of USAID/NEC
for the use and benefit of said municipality, received from the said USAID/NEC the following
items with a total value of $717.50 or P4,797.35, involving the duty of making delivery of said
items to the said Municipal Mayor, but the said accused Hermogenes Mariano once in possession
of the said items and far from complying with his aforesaid obligation and in spite of repeated
demands, did then and there wilfully, unlawfully and feloniously, with grave abuse of confidence
and with deceit, misappropriate, misapply and convert to his own personal use and benefit the
said items valued at $717.50 or P4,797.35, belonging to the said USAID/NEC, to the damage
and prejudice of the said owner in the said sum of $717,50 or P4,797.35. On February 19, 1975,
Hermogenes Mariano thru his counsel Filed a motion to quash the Information on the following
grounds:
1. That the court trying the cause has no jurisdiction of the offense charged or of the
person of the defendant; 2. That the criminal action or liability has been extinguished; 3. That it
contains averments which , if true, would constitute a legal excuse or justification.
In his motion to quash, Mariano claimed that the items which were the subject matter of
the Information against him were the same items for which Mayor Constantino A. Nolasco was
indicted before a Military Commission under a charge of malversation of public property, and for
which Mayor Nolasco had been found guilty and that inasmuch as the case against Mayor
Nolasco had already been decided by the Military Tribunal, the Court of First Instance of
Bulacan had lost jurisdiction over the case against him.
On March 14, 1975 respondent Judge issued an Order granting the motion to quash on
the ground of lack of jurisdiction reasoning as follows: Considering that the Military
Commission had already taken cognizance of the malversation case against Mayor Nolasco
involving the same subject matter in its concurrent jurisdiction with this Court, the case

involving the subject properties had already been heard and decided by a competent tribunal, the
Military Commission, and as such this Court is without jurisdiction to pass upon anew the same
subject matter. (pp. 30-31, rollo, emphasis supplied) Respondent Judge issued an order granting
the motion to quash on the ground of lack of jurisdiction but did not rule on the other grounds
invoked in the motion to quash.

Issue:
Whether or not Mariano can be held liable for estafa?
Decision:
The Supreme Court ruled that Respondent court gravely erred when it ruled that it lost
jurisdiction over the estafa case against respondent Mariano with the filing of the malversation
charge against Mayor Nolasco before the Military Commission. Estafa and malversation are two
separate and distinct offenses and in the case now before the SC the accused in one is different
from the accused in the other.
The conferment of jurisdiction upon courts or judicial tribunals is derived exclusively
from the constitution and statutes of the forum. Thus, the question of jurisdiction of respondent
Court of First Instance over the case filed before it is to be resolved on the basis of the law or
statute providing for or defining its jurisdiction. That, We find in the Judiciary Act of 1948 where
in its Section 44 (f) it is provided that Courts of First Instance shall have original jurisdiction In
all criminal cases in which the penalty provided by law is imprisonment for more than six
months,or a fine of more than two hundred pesos.The offense of estafa charged against
respondent Mariano is penalized with arresto mayor in its maximum period to prision
correccional in its minimum period, or imprisonment from four (4) months and one (1) day to
two (2) years and four (4) months. By reason of the penalty imposed which exceeds six (6)
months imprisonment, the offense alleged to have been committed by the accused, now
respondent, Mariano, falls under the original jurisdiction of courts of first instance.
The above of course is not disputed by respondent Judge; what he claims in his Order is
that his court exercises concurrent jurisdiction with the military commission and because the
latter tribunal was the first to take cognizance of the subject matter, respondent court lost

jurisdiction over it .That statement of respondent court is incorrect. In People vs. Fontanilla, this
Court speaking through then Justice now Chief Justice Fred Ruiz Castro, categorically reiterated
the settled rule that the jurisdiction of a court is determined by the statute in force at the time of
the commencement of the action. In the case at bar, it is rightly contended by the Solicitor
General that at the time Criminal Case No. SM-649 was filed with the Court of First Instance of
Bulacan, that was December 18, 1974, the law in force vesting jurisdiction upon said court was
the Judiciary Act of 1948, the particular provision of which was not affected one way or the other
by any Presidential issuances under Martial Law. The Military Commission is not vested with
jurisdiction over the crime of estafa.

166.

People v. Pacifador (G.R. No. 139405)

Facts:
On October 27, 1988, Arturo F. Pacificador then Chairman of the Board of the National
Shipyard and Steel Corporation, a government-owned corporation, and therefore, a public officer
was charged before the Sandiganbayan with the crime of violation of Republic Act No. 3019, as
amended, otherwise known as the Anti-Graft and Corrupt Practices Act.

The crime was

committed from December 6, 1975 to January 6, 1976, in Metro Manila by Pacificador. After his
arraignment, the respondent filed a Motion to Dismiss on the ground of prescription of the
offense.

Sandiganbayan on November 10, 1998 dismissed the Information against the

respondent on the ground of prescription.

The Urgent Motion for Reconsideration of the

Solicitor General was denied by the Sandiganbayan. Republic Act No. 3019 provides for its own
prescriptive period. Section 11 of R.A. No. 3019, as amended by B.P. Blg. 195, provides that the
offenses committed under the said statute shall prescribe in fifteen (15) years. It appears
however, that prior to the amendment of Section 11 of R.A. No. 3019 by B.P. Blg. 195 which
was approved on March 16, 1982, the prescriptive period for offenses punishable under the said
statute was only ten (10) years.
Issue:

Whether or not the crime had not yet prescribed as the special law governing the same have been
amended increasing the prescriptive period from ten (10) to fifteen (1) years.
Decision:
The longer prescriptive period of fifteen (15) years, as provided in Section 11 of R.A. No. 3019
as amended by B.P. Blg. 195, does not apply in this case for the reason that the amendment, not
being favorable to the accused, cannot be given retroactive effect. Hence the crime prescribed on
January 6, 1986 or ten (10) years from January 6, 1976. The crime had already prescribed when
the Information in this case was filed with the Sandiganbayan on October 27, 1988. It bears
emphasis, as held in a number of cases that in the interpretation of the law on prescription of
crimes, that which is more favorable to the accused is to be adopted. The said legal principle
takes into account the nature of the law on prescription of crimes which is an act of amnesty and
liberality on the part of the state in favor of the offender.

167.

People v. Patriarcha (G.R. No. 135457)

Facts:
On August 16, 1990, an Information for murder was filed against Jose
Patriarca, Jr., alias "Ka Django," "Carlos Narra", "Ka Jessie," et al., for killing
Alfredo Arevalo. Accused-appellant Jose Patriarca, Jr. was also charged with
Murder for the killing of one Rudy de Borja and a certain Elmer Cadag under
Informations docketed as Criminal Cases Nos. 2665 and 2672, respectively.
On January 20, 1998, the lower court rendered its decision convicting the
herein accused-appellant. Thus, Accused-Appellant filed his appeal. However,
while his appeal was pending, he applied for amnesty under Proclamation No. 724
amending Proclamation No. 347, dated March 25, 1994, entitled "Granting
Amnesty to Rebels, Insurgents, and All Other Persons Who Have or May Have
Committed Crimes Against Public Order, Other Crimes Committed in
Furtherance of Political Ends, and Violations of the Article of War, and Creating a
National Amnesty Commission." His application was favorably granted by the
National Amnesty Board.

After a careful verification and evaluation on the claims of the applicant,


the Local Amnesty Board concluded that his activities were done in the pursuit of
his political beliefs. It, thus, recommended on 20 May 1998 the grant of his
application for amnesty. The Commission, in its deliberation on the application
on 22 October 1999, resolved to approve the recommendation of the Local
Amnesty Board.
The Office of the Solicitor General, in its letter dated June 23, 2000 to the
National Amnesty Commission, requested information as to whether or not a
motion for reconsideration was filed by any party, and the action, if there was any,
taken by the NAC. In his reply dated June 28, 2000, NAC Chairman Tadiar
wrote, among other things, that there has been no motion for reconsideration filed
by any party.
Accused-appellant Jose N. Patriarca, Jr. was granted amnesty under
Proclamation No. 724 on May 17, 1996.

Issue:
Whether or not the grant of amnesty in favor of Jose Patriarca, Jr. - while
the various criminal cases filed against him were pending - shall completely
extinguished his criminal liability?

Decision:
Amnesty commonly denotes a general pardon to rebels for their treason or
other high political offenses, or the forgiveness which one sovereign grant to the
subjects of another, who have offended, by some breach, the law of nations.
Amnesty looks backward, and abolishes and puts into oblivion, the offense itself;
it so overlooks and obliterates the offense with which he is charged, that the
person released by amnesty stands before the law precisely as though he had
committed no offense.

Paragraph 3 of Article 89 of the Revised Penal Code provides that criminal


liability is totally extinguished by amnesty, which completely extinguishes the
penalty and all its effects.
In the case of People vs. Casido, the difference between pardon and
amnesty is given: "Pardon is granted by the Chief Executive and as such it is a
private act which must be pleaded and proved by the person pardoned, because
the courts take no notice thereof; while amnesty by Proclamation of the Chief
Executive with the concurrence of Congress, is a public act of which the courts
should take judicial notice. Pardon is granted to one after conviction; while
amnesty is granted to classes of persons or communities who may be guilty of
political offenses, generally before or after the institution of the criminal
prosecution and sometimes after conviction. Pardon looks forward and relieves
the offender from the consequences of an offense of which he has been convicted,
that is, it abolishes or forgives the punishment, and for that reason it does 'not
work the restoration of the rights to hold public office, or the right of suffrage,
unless such rights be expressly restored by the terms of the pardon,' and it 'in no
case exempts the culprit from the payment of the civil indemnity imposed upon
him by the sentence' (Article 36, Revised Penal Code). While amnesty looks
backward and abolishes and puts into oblivion the offense itself, it so overlooks
and obliterates the offense with which he is charged that the person released by
amnesty stands before the law precisely as though he had committed no offense."
This Court takes judicial notice of the grant of amnesty upon accusedappellant Jose N. Patriarca, Jr. Once granted, it is binding and effective. It serves
to put an end to the appeal.

168.

Facts:

People v. Pilola (405 SCRA 134)

Accused Edman Aguilos, Odilon Lagliba and Rene Pilola were charged with murder for
the death of Joselito Capa. Rene Pilola devised stabbing the victim and interposed the defense of
alibi. The trial count found all the accused guilty and sentenced them to reclusion perpetua. Rene
Pilola appealed the decision by contending that there was no conspiracy and he may not be held
criminally liable as principal by direct participation. He argued that the prosecution failed to
prove that he conspired with the others in stabbing the victim to death. He asserts that he is
merely an accomplice.
Issue:
Whether or not the appellant may be held criminally liable as principal by direct
participation in the absence of proof of conspiracy?
Decision:
The court in applying paragraph 1, Article 4 of the Revised Penal Code ruled that even if
two or more offenders do not conspire to commit homicide or murder, they may be held
criminally liable as principals by direct participation if they perform overt acts which
immediately cause or accelerate the death of the victim. They are all criminally liable although
the wrongful act done be different from that which he intended by reason of their individual and
separate overt criminal acts.

169.

People v. Sta. Maria (G.R. No. 171019)

Facts:
On November 27, 2002, at around 10:00 oclock in the morning, P/Chief Insp. Noli
Pacheco, Chief of the Provincial Drug Enforcement Group of the Bulacan Provincial Office
based at Camp Alejo Santos, Malolos, Bulacanreceived an intelligence report about the illegal
drug activities in Sitio Gulod, Barangay Pantubig, San Rafael, Bulacan of a certain "Fael," who
later turned out to be appellant Rafael Sta. Maria. P/Chief Insp. Pacheco formed a surveillance
team to look for a police asset to negotiate a drug deal with appellant.
In the morning of November 29, 2002, the surveillance team reported to P/Chief Insp.
Pacheco that a confidential asset found by the team had already negotiated a drug deal for the
purchase of P200 worth of shabu from appellant at the latters house at No. 123 Sitio Gulod,
Barangay Pantubig, San Rafael, Bulacan between7:00 and 7:30 in the evening of November 29,

2002. The surveillance team then prepared for a buy-bust operation, with PO3 Enrique Rullan as
team leader, andPO1 Rhoel Ventura, who was provided with two (2) marked P100-bills, as
poseur-buyer. At the appointed time and place, PO1 Ventura and the confidential informant
proceeded to appellants house and knocked at the door. Appellant opened the door and the
confidential informant introduced to him PO1 Ventura as a prospective buyer. PO1 Ventura later
handed the two (2) marked P100-billsto appellant who, in turn, gave him a plastic sachet of
shabu.
Thereupon, PO1Ventura sparked his cigarette lighter, which was the pre-arranged signal
to the other members of the buy-bust team that the sale was consummated. Appellant was
arrested and the two marked P100-bills recovered from him. Also arrested on that occasion was
one Zedric dela Cruz who was allegedly sniffing shabu inside appellants house and from whom
drug paraphernalia were recovered. Upon laboratory examination of the item bought from
appellant, the same yielded positive for methylampetamine hydrochloride or shabu weighing
0.041 gram. The accused was charged of violation of Section 5, Article II of R.A. No.9165,
otherwise known as the Comprehensive Dangerous Drugs Act of 2002.
The trial court found appellant guilty beyond reasonable doubt of the offense charged. The Court
of Appeals promulgated the assailed decision denying the appeal.
Issue:
Whether or not instigation was the act which preceded Sta. Marias arrest?
Held:
In entrapment, the entrapper resorts to ways and means to trap and capture a lawbreaker
while executing his criminal plan. In instigation, the instigator practically induces the would-bedefendant into committing the offense, and himself becomes a co-principal. In entrapment, the
means originates from the mind of the criminal. The idea and the resolve to commit the crime
come from him. In instigation, the law enforcer conceives the commission of the crime and
suggests to the accused who adopts the idea and carries it into execution. The legal effects of
entrapment do not exempt the criminal from liability. Instigation does. It is no defense to the
perpetrator of a crime that facilities for its commission were purposely placed in his way, or that
the criminal act was done at the "decoy solicitation" of persons seeking to expose the criminal, or
that detectives feigning complicity in the act were present and apparently assisting its
commission. Especially is this true in that class of cases where the offense is one habitually
committed, and the solicitation merely furnishes evidence of a course of conduct.
The solicitation of
drugs from appellant by the informant utilized by the police merely furnishes evidence of the
course of conduct. The police received an intelligence report that appellant has been habitually
dealing in illegal drugs. They duly acted on it by utilizing an informant to effect a drug
transaction with the appellant. There was no showing that the informant induced appellant to sell
illegal drugs to him

170.

People v. CA and Tangan (G.R. No. 103613)

Facts:
On December 1, 1984, Navy Captain Eladio C. Tangan was driving alone on Roxas
Boulevard heading south and Generoso Miranda was driving his car in the same direction with
his uncle, Manuel Miranda. Generoso was moving ahead of Tangan. Suddenly, firecrackers were
thrown in Generoso's way, causing him to swerve to the right and cut Tangan's path. Tangan blew
his horn several times. Generoso, slowed down to let Tangan pass. Tangan accelerated and
overtook Generoso, but when he got in front, Tangan reduced speed. Generoso tried four or five
times to overtake on the right lane but Tangan kept blocking his lane. When Tangan slowed down
to make a U-turn, Generoso passed him, pulled over and got out of the car with his uncle. Tangan
also stopped his car and got out. Generoso and Tangan then exchanged expletives. Then Tangan
went to his car and got his .38 caliber handgun on the front seat.
According to the prosecution witnesses, Mary Ann Borromeo, Rosalia Cruz and Manuel
Miranda, the accused pointed his gun at Generoso Miranda and when Manuel Miranda tried to
intervene, the accused pointed his gun at Manuel Miranda, and after that the accused pointed
again the gun to Generoso Miranda, the accused shot Generoso Miranda at a distance of about a
meter. The shot hit the stomach of Generoso Miranda causing the latter to fall. Manuel Miranda
grappled for the possession of the gun and during their grappling, Rosalia Cruz intervened and
took hold of the gun and after Rosalia Cruz has taken hold of the gun, a man wearing a red Tshirt took the gun from her. The man in T-shirt was chased by Manuel Miranda who was able to
get the gun where the man in red T-shirt placed it.
On the other hand, the defense, particularly the accused and his witness by the name of
Nelson Pante claimed that after the gun was taken by the accused from inside his car, the
Mirandas started to grapple for possession of the gun and during the grappling, and while the two
Mirandas were trying to wrest away the gun from the accused, they fell down at the back of the
car of the accused. The accused lost the possession of the gun after falling at the back of his car
and as soon as they hit the ground, the gun fell, and it exploded hitting Generoso Miranda.
Tangan ran away while Generoso lay on the ground bloodied. Manuel looked for the gun
and ran after Tangan. Tangan found a policeman who allowed him to enter his patrol car. Manuel
arrived and told the policeman that Tangan had just shot his nephew. Manuel went back to where
Generoso lay and there found two ladies, Mary Ann Borromeo and Rosalina Cruz, helping his

nephew board a taxi. Manuel suggested that Generoso be brought to the hospital in his car. He
was rushed to the Philippine General Hospital but he expired on the way.
Tangan was charged with the crime of murder with the use of an unlicensed firearm.
However, the information was amended to homicide with the use of a licensed firearm, and he
was separately charged with illegal possession of unlicensed firearm. Tangan entered a plea of
not guilty in the homicide case, but moved to quash the information for illegal possession of
unlicensed firearm on various grounds. The motion to quash was denied, whereupon he filed a
petition for certiorari with this Court. On November 5, 1987, said petition was dismissed and the
joint trial of the two cases was ordered.
After trial, the lower court acquitted Tangan of illegal
possession of firearm, but convicted him of homicide. The privileged mitigating circumstance of
incomplete self-defense and the ordinary mitigating circumstances of sufficient provocation on
the part of the offended party and of passion and obfuscation were appreciated in his
favor; Tangan was released from detention after the promulgation of judgment and was allowed
bail in the homicide case.
Tangan appealed to the Court of Appeals, which affirmed the judgment of the trial court
but increased the award of civil indemnity to P50,000.00. His subsequent motion for
reconsideration and a motion to cite the Solicitor General in contempt were denied by the Court
of Appeals.

The Solicitor General,

on behalf of the prosecution, alleging grave abuse of discretion, filed a petition for certiorari
under Rule 65, naming as respondents the Court of Appeals and Tangan, where it prayed that the
appellate court's judgment be modified by convicting accused-appellant of homicide without
appreciating in his favor any mitigating circumstance.
Issue:
Whether or not Tangan acted in incomplete self-defense?
Decision:
Incomplete self-defense is not considered as a justifying act, but merely a mitigating
circumstance; hence, the burden of proving the crime charged in the information is not shifted to
the accused. In order that it may be successfully appreciated, however, it is necessary that a

majority of the requirements of self-defense be present, particularly the requisite of unlawful


aggression on the part of the victim. Unlawful aggression by itself or in combination with either
of the other two requisite suffices to establish incomplete self-defense. Absent the unlawful
aggression, there can never be self-defense, complete or incomplete, because if there is nothing
to prevent or repel, the other two requisites of defense will have no basis.
The element of unlawful aggression in self-defense must not come from the person
defending himself but from the victim. A mere threatening or intimidating attitude is not
sufficient. The exchange of insulting words and invectives between Tangan and Generoso
Miranda, no matter how objectionable, could not be considered as unlawful aggression, except
when coupled with physical assault. There being no lawful aggression on the part of either
antagonists, the claim of incomplete self-defense falls.

171.

People v. Tolentino (G.R. No. 139179)

Facts:
On February 28, 1996 appellant Jonathan Fabros and his cousins, Sheila Guilayan and
Merwin Ledesma, were at their house in Luyahan, Pasonanca, Zamboanga City when their
neighbor Wilfredo Tolentino called them. When asked what it was all about, Wilfredo simply
motioned to them to come to his house located just across the road. Once they were inside the
house, Wilfredo immediately revealed his plan to kill Hernan Sagario, Sheila's stepfather.
Wilfredo explained that it was the only way to free Sheila's mother - appellant's aunt - of the
sufferings being caused by Hernan. Wilfredo then instructed Merwin to go back to the house and
get the bolo of Hernan. Merwin obliged, got the bolo, and gave it to Wilfredo. Thereafter, they
were told by Wilfredo to go home and wait for Hernan.
Around 8:30 in the evening, Hernan arrived. He went directly to the kitchen and fixed the
bag of rice he was carrying. Jonathan together with Sheila and Merwin, just stayed quiet in the
living room.Later, Wilfredo with a 2"x2" piece of wood in his hand entered the house. He then
followed Hernan towards the kitchen. When about an armslength away from Hernan, Wilfredo,
immediately walloped Hernan on the right side of the neck sending the latter unconscious and
falling face down to the ground. Wilfredo immediately instructed appellant and Merwin to help

him bring Hernan out of the house. Lifting Hernan out of the house, Wilfredo held him by the
neck while both appellant and Merwin grasped his feet. They then carried Hernan towards the
creek, upon reaching the creekside, the three stopped, then Wilfredo successively stabbed Hernan
on different parts of the body causing the latter's instant death. After throwing the victim's lifeless
body in the creek, the three immediately left. Tolentino called Jonathan, Sheila and Merwin and
warned them that if they will tell other people, he will kill them. Out of fear, they just followed
whatever Tolentino told them.
On 01 March 1996, however, Jonathan was arrested for the death of Hernan Sagario.
Accused Jonathan Fabros and Wilfredo Tolentino both denied killing the victim. Instead, they
pointed to each other as the one who killed Hernan Sagario. Fabros pointed to Tolentino as the
assailant and the latter also fingered the former as the killer of Sagario.
However, on 14 July 2000, long after the trial court's decision had become final and
executory on his part, Wilfredo Tolentino, apparently conscience-stricken, executed an affidavit
admitting sole responsibility for the death of Hernan Sagario and retracted his testimony
implicating accused-appellant Jonathan Fabros.
The trial court held that the prosecution's evidence positively identified Wilfredo
Tolentino as the person who had hit the victim with a piece of wood and later stabbed him with a
bolo. It also ruled that the killing was qualified by treachery and attended by the aggravating
circumstance of dwelling.
The court a quo observed that overt and positive acts of appellant (Jonathan Fabros)
manifested his approval of the killing and the concurrence of his acts with those of the other
accused.8 Thus, the RTC concluded that Fabros was a co-conspirator and should be held equally
responsible for the murder. Hence, this appeal.
Issue:
Whether or not appellant (Jonathan Fabros) should be convicted as an accessory?
Decision:
Appellant cannot be convicted as an accessory. Article 19 of the Revised Penal Code
defines an accessory as one who had knowledge of the commission of the crime and did not
participate in its commission as principal or accomplice, yet took part subsequent to its

commission by any of three modes: (1) profiting oneself or assisting the offender to profit by the
effects of the crime; (2) concealing or destroying the body of the crime, or the effects or
instruments thereof, in order to prevent its discovery; and (3) harboring, concealing, or assisting
in the escape of the principals of the crime, provided the accessory acts with abuse of his public
functions or when the offender is guilty of treason, parricide, murder, or an attempt to take the
life of the Chief Executive, or is known to be habitually guilty of some other crime. To convict
an accused as an accessory, the following elements must be proven: (1) knowledge of the
commission of the crime and (2) subsequent participation in it by any of the three above-cited
modes.
Under paragraph 2 of said codal provision, the concealment or the destruction of the body
of the crime or of the effects or the instruments thereof must have been done in order to prevent
the discovery of the crime. That, precisely, is wanting in the present case.
In his testimony, appellant stated that because he was afraid his co-accused would hurt
him if he refused, he agreed to assist the latter in carrying the victim towards the river. The fact
that appellant left thereafter likewise indicated his innocence of the charge. Verily, he adequately
explained his conduct prior to the stabbing incident as one born of fear for his own life. It is not
incredible for an eyewitness to a crime, especially if unarmed, to desist from assisting the victim
if to do so would put the former's life in peril. The presumption of innocence in favor of
appellant has not been overcome by proof beyond reasonable doubt. Thus, he must be acquitted.

172.

People v. Torpio (G.R. No. 138984)

Facts:
In the evening of October 11, 1997, Anthony went to the house of Dennis and invited the
latter for a drinking spree. Afterwards both left the house of Dennis and went to a nearby store
and started drinking with a companion named Porboy Perez. The three proceeded to Shoreline.
In a cottage, Anthony tried to let Dennis drink gin and as the latter refused, Anthony bathed
Dennis with gin and mauled him several times. Dennis crawled beneath the table and Anthony
tried to stab him with a 22 fan knife but did not hit him. Dennis got up and ran towards their
home.

Upon reaching home, he got a knife. Alarmed by the action of Dennis, his mother
shouted. Manuel, his father, tried to scold his son and confiscate from him the knife but failed to
do so, resulting to Manuels incurring a wound in his hand. He went back to the cottage. Upon
seeing Dennis, Anthony ran towards the creek but Dennis blocked him and stabbed him. When
he was hit, Anthony ran but got entangled with fishing net and fell on his back. Dennis then
mounted on him and continued stabbing him resulting to the latters death. After stabbing,
Dennis left and went to Camp Downes and slept there. The next morning, Dennis voluntarily
surrendered himself to Boy Estrera, a well-known police officer.
The trial court rendered a judgment convicting Dennis for the crime of Murder qualified
by treachery or evident premeditation and appreciating three mitigating circumstances. His father
Manuel was acquitted. Not satisfied with the judgment, Dennis appealed his case.
Issue:
Whether or not the mitigating circumstance of having acted in the immediate vindication
of a grave offense is appreciated?
Decision:
The Supreme held that the mitigating circumstance of having acted in the immediate
vindication of a grave offense was properly appreciated. Dennis was humiliated, mauled and
almost stabbed by the Anthony. Although the unlawful aggression had ceased when Dennis
stabbed Anthony, it was nonetheless a grave offense for which the Dennis may be given the
benefit of a mitigating circumstance.
However, the mitigating circumstance of sufficient provocation cannot be considered
apart from the circumstance of vindication of a grave offense. These two circumstances arose
from one and the same incident, i.e., the attack on the appellant by Anthony, so that they should
be considered as only one mitigating circumstance.

173.

Facts:

People v. Vasquez (G.R. No. 123939)

The appellant drove the passenger jeepney with his cohorts on board looking for Luable
and Geronimo. When the appellant saw the two going in the opposite direction, the appellant
drove the vehicle and sideswiped Geronimo. And when Geronimo fled, the appellant, armed with
a bolo, pursued him. When the appellant failed to overtake the victim, he returned to the
passenger jeepney and drove it to where his cohorts ganged up on the victim. The appellant
urged them on to kill Geronimo. Thereafter, he left the scene along with his cohorts, leaving the
hapless Geronimo mortally wounded.
After trial, the court rendered judgment acquitting Ramon, but convicting the appellant of
murder for the killing of Geronimo, and attempted homicide for attempting to kill Luis.
The appellant avers that he and his brother Ramon had no motive to kill Geronimo. The
appellant contends that the witnesses for the prosecution were not in agreement as to who killed
Geronimo. The appellant noted that according to the testimony of the witness, the appellant
stayed in the jeepney and merely yelled to his companions who ganged up on Geronimo, "Sige
patayin ninyo, patayin ninyo na, at huwag ninyong iwanang buhay!"
The appellant further posits that the prosecution witnesses were not even in accord as to
where Geronimo was stabbed to death. The appellant argues that because of the inconsistencies
in the testimonies of the witnesses of the prosecution, it failed to prove his guilt beyond
reasonable doubt of the crimes charged. Hence, he should be acquitted of the said charges.
Issue:
Whether the trial court erred in convicting the appellant when the witnesses testimony
didnt confirm who chased and stabbed the victims?
Decision:
Whether Domingo Vasquez chased the deceased with a bolo was averred by Luis Luable
or whether the accused merely incited his companions in the jeepney to kill the deceased as
averred by Luisa Abellanosa, is immaterial in the determination of his liability because a
conspiracy among the occupants of the jeepney has been established.
In order to hold an accused guilty as co-principal by reason of conspiracy, it must be
established that he performed an overt act in furtherance of the conspiracy, either by actively
participating in the actual commission of the crime, or by lending moral assistance to his co-

conspirators by being present at the scene of the crime, or by exerting moral ascendancy over the
rest of the conspirators as to move them to executing the conspiracy."
The Supreme Court, likewise, stressed that where there are several accused and
conspiracy has been established, the prosecution need not pinpoint who among the accused
inflicted the fatal wound. Where conspiracy has been established, evidence as to who among the
accused rendered the fatal blow is not necessary. All the conspirators are liable as co-principals
regardless of the intent and character of their participation because the act of one is the act of all.
Article 8 of the Revised Penal Code provides that there is conspiracy when two or more
persons agree to commit a felony and decide to commit it. Conspiracy need not be proven by
direct evidence. It may be inferred from the conduct of the accused before, during and after the
commission of the crime, showing that they had acted with a common purpose and design.
Conspiracy may be implied if it is proved that two or more persons aimed by their acts towards
the accomplishment of the same unlawful object, each doing a part so that their combined acts,
though apparently independent of each other were, in fact, connected and cooperative, indicting a
closeness of personal association and a concurrence of sentiment. Conspiracy once found,
continues until the object of it has been accomplished and unless abandoned or broken up. To
hold an accused guilty as a co-principal by reason of conspiracy, he must be shown to have
performed an overt act in pursuance or furtherance of the complicity. There must be intentional
participation in the transaction with a view to the furtherance of the common design and purpose.
Each conspirator is responsible for everything done by his confederates which follows
incidentally in the execution of a common design as one of its probable and natural
consequences even though it was not intended as part of the original design.
Responsibility of a conspirator is not confined to the accomplishment of a particular
purpose of conspiracy but extends to collateral acts and offenses incident to and growing out of
the purpose intended. Conspirators are held to have intended the consequences of their acts and
by purposely engaging in conspiracy which necessarily and directly produces a prohibited result,
they are, in contemplation of law, chargeable with intending that result. Conspirators are
necessarily liable for the acts of another conspirator unless such act differs radically and
substantively from that which they intended to commit. When a conspirator embarks upon a
criminal venture of indefinite outline, he takes his chances as to its content and membership, so
be it that they fall within the common purposes as he understands them."

All the foregoing constitutes evidence beyond cavil of conspiracy between the appellant
and the principals by direct participation. The appellant is, thus, criminally liable for the death of
the victim, although there is no evidence that he did not actually stab the latter.

174.

People v. Verzola (G.R. No. L-35022)

Facts:
On September 28, 1969, Bernardo Molina was clubbed to death by Ricardo Verzola in the
presence of appellant Josefina Molina inside Molina's house at Barrio Lipcan, Bangued, Abra.
The body of the victim was subsequently carried by the two appellee to the ground and left at the
foot of the stairs. Appellant Verzola then went to his house, changed his clothes and threw his
bloodstained sweater undershirt and underwear, including the piece of wood be used in clubbing
the deceased, inside their toilet. Afterwards, he went to the municipal building and reported to
the police authorities that Bernardo had died in an accident. The police authorities together with
the Municipal Health Officer, the Municipal Judge and a photographer went to Lipcan to conduct
the investigation. They found the body of the deceased Bernardo Molina sprawled at the foot of
the bamboo ladder. Blood had oozed from the mouth, nose and ears. There were bloodstains on
the floor of the bedroom of the house, on the mat, as well as on the beddings of the deceased.
The bloodstains led to the bamboo ladder where some of the stains could be found on the steps of
the ladder. When questioned by the police, Josefina revealed that the assailant of her husband
was Ricardo Verzola.
Upon her request, she was brought to the Office of the Chief of Police of Bangued, where
at about 2:00 o'clock in the morning of September 29, 1969 she gave a written statement
narrating the circumstances surrounding the incident in question and pointing to appellant
Verzola as the assailant of her husband. In that extra-judicial statement, she stated that
immediately after 10:00 o'clock in the evening of September 28. 1969, appellant Ricardo Verzola
went to their house in Barrio Lipcan, Bangued Abra entered the room where she was sleeping
with her husband, Bernardo Molina, woke her up and had carnal knowledge of her; that when
Bernardo Molina woke up and attempted to rise from the floor, that was the moment when
Verzola clubbed Bernards, hitting him on the head several times that afterwards, she heard the

sound of a body being dragged downstairs and the voice of Verzola saying that he was leaving
and warning her not to say anything about the incident. She looked out of the door and saw her
husband already lying prostrate at the foot of the stairs. This statement was sworn to by her
before Municipal Judge Francisco T. Valera.
On that same morning, appellant Verzola was picked up by the police and brought to the
municipal building, and there he also executed a written statement admitting that he clubbed the
victim several times. Both appellants admit that it was appellant Verzola who inflicted the fatal
blows on the victim.
The trial court convicted Verzola as principal and Josefina Molina as an accessory to the
crime of murder.

Issue:
Whether or not assisting the principal in bringing the body of the deceased to the ground
will make one an accessory to the crime?
Decision:
An accessory does not participate in the criminal design, nor cooperate in the commission
of the felony, but with knowledge of the commission of the crime, he subsequently takes part in
three (3) ways: (a) by profiting from the effects of the crime; (b) by concealing the body, effects
or instruments of the crime in order to prevent its discovery; and (c) by assisting in the escape or
concealment of the principal of the crime, provided he acts with abuse of his public functions or
the principal is guilty of treason, parricide, murder, or an attempt to take the life of the Chief
Executive or is known to be habitually guilty of some other crime.
Even if she assisted her co-appellant without duress, simply assisting Verzola in bringing
the body down the house to the foot of the stairs and leaving said body for anyone to see, cannot
be classified as an attempt to conceal or destroy the body of the crime. The concealing or
destroying of the body of the crime, the effects or instruments thereof, must be done to prevent
the discovery of the crime. In the case at bar, the body was left at the foot of the stairs at a place
where it was easily visible to the public. Under such circumstances there could not have been

any attempt on the part of Josefina to conceal or destroy the body of the crime. Thus, Josefina
Molina is acquitted.

175.

People vs Sales (658 SCRA 367)

Facts:
On September 19, 2002, brothers Noemar and Junior, then nine and eight years old, respectively,
left their home to attend the fluvial procession of Our Lady of Peafrancia without the permission of their
parents. They did not return home that night. When their mother, Maria Litan Sales (Maria), looked for
them the next day, she found them in the nearby Barangay of Magsaysay. Afraid of their fathers rage,
Noemar and Junior initially refused to return home but their mother prevailed upon them. When the two
kids reached home a furious appellant confronted them. Appellant then whipped them with a stick which
was later broken so that he brought his kids outside their house. With Noemars and Juniors hands and
feet tied to a coconut tree, appellant continued beating them with a thick piece of wood.
When the beating finally stopped, the three walked back to the house, Noemar collapsed and lost
consciousness. Maria then told appellant to call a quack doctor. He left and returned with one, who told
them that they have to bring Noemar to a hospital. Appellant thus proceeded to take the unconscious
Noemar to the junction and waited for a vehicle to take them to a hospital. As there was no vehicle and
because another quack doctor they met at the junction told them that Noemar is already dead, appellant
brought his son back to their house.
Appellant denied that his son died from his beating since no parent could kill his or her child. He
claimed that Noemar died as a result of difficulty in breathing. In fact, he never complained of the
whipping done to him. Besides, appellant recalled that Noemar was brought to a hospital more than a
year before September 2002 and diagnosed with having a weak heart.
On the other hand, Maria testified that Noemar suffered from epilepsy. Whenever he suffers from
epileptic seizures, Noemar froths and passes out. But he would regain consciousness after 15
minutes. His seizures normally occur whenever he gets hungry or when scolded. The trial court charged
the accused guilty of parricide and slight physical injuries.
Issue:
Whether or not the accused is guilty of the crimes charged.
Held:
Yes. All the elements of the crime of parricide is present in this case. Parricide is committed
when: (1) a person is killed; (2) the deceased is killed by the accused; (3) the deceased is the father,
mother, or child, whether legitimate or illegitimate, or a legitimate other ascendant or other descendant, or
the legitimate spouse of accused.
In the case at bar, there is overwhelming evidence to prove the first element, that is, a person was
killed. There is likewise no doubt as to the existence of the second element that the appellant killed the

deceased. It is sufficiently established by the positive testimonies of Maria and Junior. As to the third
element, appellant himself admitted that the deceased is his child.
With regard to charge of Physical injuries, the victim himself, Junior testified that he, together
with his brother Noemar, were beaten by their father, herein appellant, while they were tied to a coconut
tree. He recalled to have been hit on his right eye and right leg and to have been examined by a physician
thereafter. Maria corroborated her sons testimony.

176.

People vs Tomas (G.R. No. 192251)

Facts:
The victim in this case is Estrella. She was with her mother. Damiana Doctor and
caretakers Liezl Toledo and Angelita Duque. They were traversing the road towards her house in
Barangay Baybayaos, Mayantoc, Tarlac after she had parked her rented car at the house of
Liezls mother in law. Estrella was walking slightly ahead of her mother and Angelita when
appellants,Tomas, Sr., Doctor and Gatchalian suddenly came out from the side of the road.
Thereupon, without saying anything, Tomas, Sr. drew a gun and shot Estrella twice. Gatchalian,
without a gun, allegedly supported Tomas, Sr. by standing in a blocking position along the road,
while Doctor positioned himself at the back of Damiana and Angelita and poked a handgun at
them, telling them to lie face down on the ground, though they did not totally drop on the road
but were in a kneeling position.
Accused Tomas Sr. denied involvement in the incident. The two others, Doctor and Gatchalian
both asserted that they were sleeping in their respective homes when the incident occurred. The
trial court and Appellate Court found the accused-appellants in conspiracy for the murder of the
victim. Hence, the petition before the Supreme Court raising presence of conspiracy among the
appellants.
Issue:
Whether there is conspiracy?
Held:
Yes. Conspiracy exists when two or more persons come to an agreement concerning the
commission of a crime and decide to commit it. It may be proved by direct or circumstantial
evidence consisting of acts, words or conduct of the alleged conspirators before, during and after
the commission of the felony to achieve a common design or purpose. In the instant case, the
ascertained facts of the shooting to death of Estrella with treachery established beyond
reasonable doubt the commission of the crime of murder.
Tomas, Sr.s guilt has been proved beyond reasonable doubt. To be equally guilty for
murder, it must be shown that Doctor and Gatchalian conspired with Tomas, Sr., for in a
conspiracy, every act of one of the conspirators in furtherance of a common design or purpose of
such a conspiracy is the act of all. It has been duly established that Doctors contemporaneous act

was made in furtherance of the common purpose of killing Estrella and ensuring impunity from
the act. Indeed, Doctors cooperation in the shooting of Estrella ensured its accomplishment and
their successful escape from the crime scene. Doctor is, thus, equally guilty and liable for the
murder of Estrella on account of conspiracy. It appears that Gatchalian is a party to the
conspiracy. He also fled together with them. However, Gatchalian was unarmed and did not say
anything or commit any overt act to externally manifest his cooperation with the shooting of
Estrella. On the other hand, Gatchalian never attempted to stop the shooting, which tends to
show that he was aware of the plan and intent to kill Estrella or, at the very least, that he
acquiesced to the shooting of Estrella. Thus, with his lack of overt acts manifestly contributing
to the accomplishment of the common design to shoot Estrella, there is some doubt if he indeed
conspired with the two accused-appellants.
The fact that Gatchalian appeared together with the other accused- appellants and fled
with them, while not constitutive of proof beyond reasonable doubt of conspiracy, still proves a
certain degree of participation and cooperation in the execution of the crime. Consequently, in
line with the principle that whatever is favorable to an accused must be accorded him, Gatchalian
is guilty as an accomplice only.

177.

Quinto vs Andres, 453 SCRA 511

Facts:
Dante Andres and Randyver Pacheco were by the mouth of a drainage culvert. Andres
and Pacheco invited Wilson and Garcia to go fishing with them inside the drainage culvert.
Wilson assented. When Garcia saw that it was dark inside, he opted to remain seated in a grassy
area. After a while, respondent Pacheco, who was holding a fish, came out of the drainage
system and left without saying a word. Respondent Andres also came out, went back inside, and
emerged again, this time, carrying Wilson who was already dead.
The respondents filed a demurer to evidence which
the trial court granted on the ground of insufficiency of evidence. It also held that it could not
hold the respondents liable for damages because of the absence of preponderant evidence to
prove their liability for Wilsons death. The petitioner appealed the order to the Court of Appeals
insofar as the civil aspect of the case was concerned. The CA ruled that the acquittal in this case
is not merely based on reasonable doubt but rather on a finding that the accused-appellees did not
commit the criminal acts complained of. Thus, pursuant to the above rule and settled
jurisprudence, any civil action ex delicto cannot prosper. Acquittal in a criminal action bars the
civil action arising therefrom where the judgment of acquittal holds that the accused did not
commit
the
criminal
acts
imputed
tothem.
Issue:

Whether or not the extinction of respondents criminal liability carries with it the
extinctionoftheircivilliability.
Held:
The Court held when a criminal action is instituted, the civil action for the recovery of
civil liability arising from the offense charged shall be deemed instituted with the criminal action
unless the offended party waives the civil action, reserves the right to institute it separately or
institutes
the
civil
action
prior
to
the
criminal
action.
The prime purpose of the criminal action is to punish the offender in order to deter him and
others from committing the same or similar offense, to isolate him from society, to reform and
rehabilitate him or, in general, to maintain social order. The sole purpose of the civil action is the
restitution, reparation or indemnification of the private offended party for the damage or injury
he sustained by reason of the delictual or felonious act of the accused.
The extinction of the penal action
does not carry with it the extinction of the civil action. However, the civil action based on delict
shall be deemed extinguished if there is a finding in a final judgment in the criminal action that
the act or omission from where the civil liability may arise does not exist. In this case, the
petitioner failed to adduce proof of any ill-motive on the part of either respondent to kill the
deceased and as held by the the trial court and the CA, the prosecution failed to adduce
preponderant evidence to prove the facts on which the civil liability of the respondents rest, i.e.,
that the petitioner has a cause of action against the respondents for damages.
178.

Recebido v. People (346 SCRA 881)

Facts:
Sometime in April 1985, Caridad Dorol mortgaged her property--an agricultural land
located in Bacon, Sorsogon to her cousin Recebido. Dorol and Recebido did not execute any
mortgage document, but instead, the former gave to the latter a copy of the Deed of Sale dated
June 16, 1973 which was done by Juan Dorol (father of Caridad). On September 9, 1990,
Caridad Dorol went to the house of Recebido to redeem such property, wherein Recebido refused
to allow claiming that Dorol has already sold to him the land on 1979. Dorol, on the other hand,
insisted that the transaction between them was not a sale, but a mere mortgage.
Caridad Dorol, then, went to the Office of the Assessor in Sorsogon and
verified the existence of a file Deed of Sale dated August 13, 1979 in which she knew that the
property was already registered in Recebidos name. A comparison of the specimen signatures of
Caridads other documents and that in the questioned Deed of Sale was done, and NBI Document
Examiner Antonio Magbojas found out that in the latters signature was falsified.
The Office of the Provincial

Prosecutor of Sorsogon filed the information indicting Recebido for Falsification of Public
Document with the Regional Trial Court of Sorsogon. The trial court rendered the decision
convicting the petitioner of the crime and sentenced to an indeterminate penalty of one (1) year
to three (3) years and six (6) months of prision correccional as maximum and to pay a fine of
Three Thousand (P3,000.00) Pesos, with subsidiary imprisonment. The defense of prescription
was raised only during the motion for reconsideration of the Court of Appeals.

Issue:
Whether or not the crime charged had already prescribed at the time the information was
filed?

Decision:
No. Prescription, although not invoked in the trial, may, as in this case, be invoked on
appeal. Hence, the failure to raise this defense in the motion to quash the information does not give
rise to the waiver of the petitioner-accused to raise the same anytime thereafter including during
appeal. Nonetheless, we hold that the crime charged has not prescribed. The petitioner is correct in
stating that whether or not the offense charged has already prescribed when the information was
filed would depend on the penalty imposable therefore, which in this case is prision correccional
in its medium and maximum periods and a fine of not more than 5,000.00 pesos.
Under the Revised Penal Code, said penalty is a correctional penalty in the same
way that the fine imposed is categorized as correctional. Both the penalty and fine being
correctional, the offense shall prescribe in ten years. The issue that the petitioner has missed,
however, is the reckoning point of the prescriptive period. The petitioner is of the impression that
the ten-year prescriptive period necessarily started at the time the crime was committed. This is
inaccurate. Under Article 91 of the Revised Penal Code, the period of prescription shall
commence to run from the day on which the crime is discovered by the offended party, the
authorities, or their agents.

179.

Romera vs. People of the Philippines [G.R. No. 151978]

Facts:
Petitioner stabbed the victim in the stomach as a result of an altercation. After the stabbing,
petitioner surrendered to the CAFGU. He was then charged with frustrated murder with the

regional trial court, where he alleged that it was the victim who hacked the walls of his house
with a bolo and thrust the same at him, after which, they grappled with the bolo where the
petitioner then stabbed the victim. He was then convicted, but he was credited with the
mitigating circumstance of voluntary surrender in the imposition of his sentence. The Court of
Appeals then affirmed the trial courts judgment.

Issue:
Whether or not the mitigating circumstances of provocation and passion and obfuscation are
present in this case.

Held:
Yes. The Court held that thrusting his bolo at petitioner, threatening to kill him, and hacking the
bamboo walls of his house are sufficient provocation to enrage any man, or stir his rage and
obfuscate his thinking, more so when the lives of his wife and children are in danger. Petitioner
stabbed the victim as a result of those provocations, and while petitioner was still in a fit of rage.
But, it is stressed that provocation and passion or obfuscation are not two
separate mitigating circumstances. Well-settled is the rule that if these two circumstances are
based on the same facts, they should be treated together as one mitigating circumstance. From
the facts established in this case, it is clear that both circumstances arose from the same set of
facts aforementioned. Hence, they should not be treated as two separate mitigating
circumstances.
Nonetheless, since the mitigating circumstance of voluntary surrender is
also present, Article 64 (5) of the Revised Penal Code should be applied. The penalty for
frustrated homicide, pursuant to Article 50[8] of the Revised Penal Code, is the penalty next
lower in degree than that prescribed by law for consummated homicide. The penalty for
consummated homicide is reclusion temporal, hence the penalty next lower in degree is prision
mayor. There being two mitigating circumstances and no aggravating circumstance, pursuant to
Article 64 (5) of the Revised Penal Code, the next lower penalty, prision correccional, is the next
statutory penalty. But following the Indeterminate Sentence Law herein applicable, the minimum
term of the penalty that should be imposed on petitioner for frustrated homicide should be within
the range of arresto mayor in any of its periods or from one (1) month and one (1) day to six (6)
months, while the maximum term should be within the range of prision correccional in its
medium period or two (2) years, four (4) months and one (1) day to four (4) years and two (2)
months.

180.

Simangan v. People (434 SCRA 38)

Facts:
On February 10, 1980 at about 8 pm, Simangan and four other men wearing fatigues
knocked on the door of the store owned by Ernesto and Sofronia. The couple was having dinner
with their daughter Lorna. Simangan asked Ernesto to guide them on the road as they were not
familiar. Ernesto agreed, he then ordered his houseboy Romeo to accompany him in guiding the
group of Simangan. The next morning, Romeo reported to Sofronia that Ernesto is dead. Ernesto
was found near a creek, he sustained 10 stab wounds.

Issue:
Whether or not Simangan is guilty beyond reasonable doubt.

Decision:
Yes. The testimonies of Romeo and Sofronia are credible. Thus, Simangans conviction is
affirmed. It is found that Simangan stabbed Ernesto 10 times, three of which were fatal. But the
number of stab wounds does not qualify as an aggravating circumstance against Simangan for it
must be proven that Simangan intended to exacerbate the suffering of Ernesto. Nigh time is also
not appreciated as it was included in the original information.

181.

Toledo v. People (439 SCRA 94)

Facts:
The accused Toledo was charged with homicide for the killing of one Ricky Duarte.
Toledo insisted that when he killed the victim, the same was purely accidental. He claimed that
the victim was so drunk that the same charged at the door of his house. This prompted the
accused to get his bolo and when he tried to prevent Ricky from entering, he accidentally hit the
latter whereby killing him.

But still the RTC and the Ca found him guilty. And so, the accused goes to the SC
wherein this time, he claims that his actions were purely on self-defense. It was done when the
victim attacked him and in trying to defend himself, he accidentally killed Duarte.
Issue:
Should the Court find his actions exempting and/or justifying?

Decision:
The SC ruled that there is no such thing as accidental self-defense. The accused cannot claim the
death purely accidental and when the findings of the lower courts were unfavorable, later on
change his defense by alleging that what happened was purely self-defense.
The two defenses perpetuated by the accused are totally inconsistent with each other.
Although in the justifying circumstance of self-defense, an accused is excused because of
DELIBERATELY trying to repel an unlawful aggression which could have killed or injure him.
And so, such acts are not in tune with ACCIDENT which presupposes an act which was not even
contemplated or planned but purely accidental.

182.

Ty v. People (G.R. No. 149275)

Facts:
This case stemmed from the filing of 7 Informations for violation of B.P. 22 against Ty
before the RTC of Manila. The said accused drew and issue to Manila Doctors Hospital to apply
on account or for value to Editha L. Vecino several post-dated checks. The said accused well
knowing that at the time of issue she did not have sufficient funds in or credit with the drawee
bank for payment of such checks in full upon its presentment, which check when presented for
payment within ninety (90) days from the date hereof, was subsequently dishonored by the
drawee bank for Account Closed and despite receipt of notice of such dishonor, said accused
failed to pay said Manila Doctors Hospital the amount of the checks or to make arrangement for
full payment of the same within five (5) banking days after receiving said notice.
Ty claimed that she issued the checks because of an uncontrollable fear of a greater
injury. She claims that she was forced to issue the checks to obtain release of her mother whom

the hospital inhumanely and harshly treated, and would not discharge unless the hospital bills are
paid.
The trial court rendered judgment against Ty. Ty interposed an appeal with the CA and
reiterated her defense that she issued the checks under the impulse of an uncontrollable fear of a
greater injury or in avoidance of a greater evil or injury. The appellate court affirmed the
judgment of the trial court with modification. It set aside the penalty of imprisonment and
instead sentenced Ty to pay a fine of sixty thousand pesos P 60,000.00 equivalent to double the
amount of the check, in each case.

Issue:
Whether or not the defense of uncontrollable fear is tenable to warrant her exemption
from criminal liability?

Decision:
NO. Uncontrollable fear - For this exempting circumstance to be invoked successfully,
the following requisites must concur: (1) existence of an uncontrollable fear; (2) the fear must be
real and imminent; and (3) the fear of an injury is greater than or at least equal to that committed.
In the instant case, the evil sought to be avoided is merely expected or anticipated. If the evil
sought to be avoided is merely expected or anticipated or may happen in the future, this defense
is not applicable.
It must appear that the threat that caused the uncontrollable fear is of such gravity and
imminence that the ordinary man would have succumbed to it. It should be based on a real,
imminent or reasonable fear for ones life or limb. A mere threat of a future injury is not
enough. It should not be speculative, fanciful, or remote. A person invoking uncontrollable fear
must show therefore that the compulsion was such that it reduced him to a mere instrument
acting not only without will but against his will as well. It must be of such character as to leave
no opportunity to the accused for escape.

The fear harbored by Ty was not real and imminent. Ty claims that she was compelled to
issue the checks, a condition the hospital allegedly demanded of her before her mother could be
discharged, for fear that her mothers health might deteriorate further due to the inhumane
treatment of the hospital or worse, her mother might commit suicide. This is speculative fear; it
is not the uncontrollable fear contemplated by law.

183.

US v Insierto (15 Phil 358)

Facts:
The defendant Ramon Insierto inflicted upon his niece, Marcelina Cainela, a girl of
twelve years of age, three wounds, which required a little over a month to cure, without medical
attendance, and of which on the day of the trial nothing but scars appeared. The victim declared
that she lived with her aunt and uncle, Olivia Insierto and Ramon Insierto, the accused; That the
latter was her teacher and that he had been teaching for a long time and did no beat her when she
took lessons, and that he beat her on that occasion only because she was unable to answer his
question.

Issue:
Whether or not defendant is exempt from criminal liability on account of his relationship
with the victim?

Held:
No. The defendant cannot apply the defense of relationship to be exempted from criminal
liability nor be allowed to use it as mitigating circumstance.
The fact is that the circumstance of relationship can be considered, neither as an
aggravating or as a mitigating one. The relationship which might aggravate or mitigate criminal
liability is defined by Article 10 of the RPC: When the injured person is the spouse, ascendant,
descendant, legitimate, natural, or adopted brother or sister or relative by affinity in the same
degrees of the offender. Marcelina does not come within any of the above degrees of
relationship with respect to the defendant, who is simply her uncle.
184.

Fragrante v People (624 SCRA 566)

Facts:
AAA was the third biological child borne of the marriage of Ernesto Fragrante and
CCC. Many times in the period of April 1993 to September 1995 and September 1997 to
October 1997, Ernesto Fragrante acted lasciviously with his biological minor daughter. Likewise,
sometime in 1995 of September, he had carnal knowledge of said daughter against her will.
Both trial court and appellate court convicted
appellant Ernesto Fragante of nine (9) counts of acts of lasciviousness and one (1) count of rape,
all committed against his minor daughter, AAA. RTC found Ernesto liable for all charges
while CA acquitted Ernesto for the crime of rape.

Issue:
Whether or not Fragrante is guilty of rape and acts of lasciviousness

Ruling:
Yes. It was held by the Court several times that actual force or intimidation need not be
employed in incestuous rape of a minor because the moral and physical dominion of the father is
sufficient to cow the victim into submission to his beastly desires. When a father commits the
odious crime of rape against his own daughter, his moral ascendancy or influence over the latter
substitutes for violence and intimidation. The absence of violence or offer of resistance would
not affect the outcome of the case because the overpowering and overbearing moral influence of
the father over his daughter takes the place of violence and offer of resistance required in rape
cases committed by an accused who did not have blood relationship with the victim.
As for the cases for acts of lasciviousness, the Supreme Court held that the
alternative circumstance of relationship is an aggravating circumstance. In the instant case, the
information expressly state that AAA is appellants daughter, and appellant openly admitted this
fact.

185.

Licyayo v. People (547 SCRA 598)

Facts:
After attending the same wedding, two groups of men went to the store of Kiangan Public
Market. One group is Aron Licyayo and two friends. The other group is Rufino and friends.
After their drinking sessions, the two groups also went to same sari-sari store owned by Larry
Famorca. A brawl suddenly occurred between Rufinos companion and Arons companion. While
police officers off duty rushed to the scene, it did not repel Roberto Licyayos act of stabbing
Rufino, while the latter was wrestling with Aron.
A criminal case was filed before the RTC charging petitioner, his brother Aron Licyayo
(Aron), Paul Baguilat (Paul) and Oliver Buyayo (Oliver) with Homicide under Article 249 of the
Revised Penal Code, of the killing of one Rufino Guay. This was later on consolidated with the
Direct Assault case related to such incident. The RTC convicted Licyayo. The CA affirmed the
decision of the RTC.

ISSUE:
Whether Roberto is entitled to the mitigating circumstance of Intoxication

Ruling:
No. Roberto clearly knows what he is doing during that time. There is no plausible
evidence showing that the quantity of liquor taken by petitioner was of quantity as to affect his
mental faculties. For intoxication to be considered as a mitigating circumstance, it must be
shown that the willpower of the accused the accused was impaired and that he did not know what
he was doing or could not comprehend the wrongfulness of his acts. The person pleading
intoxication must prove that he took such quantity of alcoholic beverage, prior to the commission
of the crime, as would blur his reason.
The fact that petitioner could recall the details that transpired during and after his
drinking session with friends is the best proof that he knew what he was doing during the
incident. His vivid narration that he had a confrontation with Rufino, Jeffrey and Joel during the
drinking session; that Daniel approached and told him that Aron was being mauled; that he
immediately went to the scene and saw Aron being beaten by Rufino and Jeffrey; that he pushed
Jeffrey away from Aron; that he was allegedly beaten by the companions of Jeffrey; and that he
fought back but was allegedly overpowered all point to the conclusion that petitioner had
complete control of his mind during the incident. It must be clear and convincing proof such fact
and not merely rely on the testimony of the prosecution witnesses.
186.

Pangonorom v. People (455 SCRA 211)

Facts:
Pangonorom was the driver of a passenger bus owned and operated by MMTC which
collided with a Gemini Isuzu car driven by Carlos Berba, resulting into the damage of the car and
physical injuries obtained by Berba.
Pangonorom was found guilty of reckless imprudence resulting into damage to property
and physical injuries. The trial court awarded damages amounting to P42,000 but was silent as
to the subsidiary liability of MMTC. Consequently, the CA affirmed the decision of the trial
court and also found MMTC subsidiary liable for the amount notwithstanding the fact that the
judgment of the trial court was silent as to said matter.

Issue:
Did the CA err in not holding MMTC not subsidiary liable despite the fact that the RTC
did not mention anything to that effect?

Decision:
The SC ruled that even when the dispositive portion of an RTC decision does not
expressly pronounce subsidiary liability of the employer, they are deemed written into the
judgment whenever applicable.
But, he subsidiary liability of the employer arises only after conviction of the employee
in the criminal action. In the present case, there exists an employer-employee relationship
between petitioners, the MMTC is engaged in the transportation industry, and Olimpio has been
adjudged guilty of a wrongful act and found to have committed the offense in the discharge of
his duties. However, there is no proof here of Olimpios insolvency. The judgment of conviction
against Olimpio has not attained finality. This being so, no writ of execution can issue against

him to satisfy his civil liability. Only after proof of the accused-employees insolvency may the
subsidiary liability of his employer be enforced.
In short, there is as yet no occasion to speak of enforcing the employers subsidiary civil
liability unless it appears that the accused-employees primary liability cannot in the first
instance be satisfied because of insolvency. This fact cannot be known until sometime after the
verdict of conviction shall have become final. And even if it appears prima facie that execution
against the employee cannot be satisfied, execution against the employer will not issue as a
matter of course.
The procedure for the enforcement of a judgment will have to be followed. Once the
judgment of conviction against Olimpio becomes final and executory, and after the writ of
execution issued against him is returned unsatisfied because of his insolvency, only then can a
subsidiary writ of execution be issued against the MMTC after a hearing set for that precise
purpose. It is still too early to hold the MMTC subsidiarily liable with its accused-employee
considering that there is no proof yet of Olimpios insolvency.
187.

People of the Philippines v Genosa, 419 SCRA 537

Facts:
Ben Genosa was killed by his wife Marivic Genosa the herein appellant. During their first year
of marriage. Marivic and Ben lived happily but later on Ben changed and the couple would
always quarrel and sometimes their quarrels would become violent. Appellant testified that every
time her husband came home, he was drunk and he would provoke her and sometimes beat her.
Whenever beaten by her husband, she consulted medical doctors who testified during their trial.
On the night of the killing, the appellant and the victim quarreled and the victim beat the
appellant. However, appellant was able to run to another room. Appellant admitted having killed
the victim with the use of a gun. The information for parricide against the appellant, however,
alleged that the cause of death of the victim was by beating through the use of lead pipe.
Appellant invoked self defense and defense of her unborn child. The trial court found her guilty
for the crime of parricide with an aggravating circumstance of treachery and imposed the penalty
of death.
On review by the
Supreme Court, appellant filed an urgent omnibus motion praying that the Court allow the

exhumation of Ben Genosa and the reexamination of the cause of death; The examination of
Marivic Genosa by qualified psychologists and psychiatrists to determine her state of mind at the
time she killed her husband; and the inclusion of the said experts reports in the records of the
case for purposes of automatic review or in the alternative a partial re-opening of the case a quo
to take the testimony of said experts opinion on the battered woman syndrome plea. Dr.
Pajarillo and Dra. Dayan were presented and admitted by the trial court and subsequently
submitted to the Supreme Court as part of the records.

Issue:
Whether the appellant can validly invoke the battered woman syndrome as constituting selfdefense

Held:
A battered woman has been defined as woman who is repeatedly subjected to any forceful
physical or psychological behavior by a man in order to coerce her to do something he wants her
to do without concern for her rights. Battered woman in any form in intimate relationship with
men. Furthermore, in order to be classified as a battered woman, the couple must go through the
battering cycle at least twice. Any woman may find herself in an abusive relationship with a man
once. If it occurs a second time and she remains in the situation she is defined as battered
woman.
More graphically, the battered woman syndrome is characterized by the so-called
cycle of violence, which has three phases: (1) tension building phase; (2) the acute battering
incident; and (3) the tranquil, loving phase.
The
Court,
however, is not discounting the possibility of self-defense arising from the battered woman
syndrome. First, each of the phases of the cycle of violence must be proven to have characterized
at least two battering episodes between the appellant and her intimate partner. Second, the final
acute battering episode preceding the killing of the batterer must have produced in the battered
persons mind an actual fear of an imminent harm from her batterer and an honest belief that she
needed to use force in order to save her life. Third, at the time of the killing, the batterer must
have posed probable -- not necessarily immediate and actual grave harm to the accused, based
on the history of violence perpetrated by the former against the latter. Taken altogether, these
circumstances could satisfy the requisites of self-defense. Under the existing facts of the present
case, however, not all of these elements were duly established. The defense fell short of proving
all three phases of the cycle of violence supposedly characterizing the relationship of Ben and
Marivic. No doubt there were acute battering incidents but appellant failed to prove that in at
least another battering episode in the past, she had gone through similar pattern. Neither did
appellant proffer sufficient evidence in regard to third phase of the cycle. In any event, the
existence of the syndrome in a relationship does not in itself establish the legal right of the

woman to kill her abusive partner. Evidence must still be considered in the context of selfdefense. Settled in our jurisprudence, is the rule that one who resorts to self-defense must face a
real threat on ones life; and the peril sought to be avoided must be imminent and actual, not
merely imaginary. Thus, the Revised Penal Code provides that the following requisites of selfdefense must concur: (1) Unlawful aggression; (2) reasonable necessity of the means employed
to prevent or repel it; and (3) lack of sufficient provocation on the part of the person defending
himself.
Unlawful aggression is the most essential element
of self-defense. It presupposes actual, sudden and unexpected attack or an imminent danger
thereof on the life or safety of a person. In the present case, however, according to the
testimony of Marivic herself, there was a sufficient time interval between the unlawful
aggression of Ben and her fatal attack upon him. She had already been able to withdraw from his
violent behavior and escape to their childrens bedroom. During that time, he apparently ceased
his attack and went to bed. The reality or even the imminence of the danger he posed had ended
altogether. He was no longer in a position that presented an actual threat on her life or safety.
The mitigating factors of psychological paralysis
and passion and obfuscation were, however, taken in favor of the appellant. It should be clarified
that these two circumstancespsychological paralysis as well as passion and obfuscation did
not arise from set of facts. The first circumstance arose from the cyclical nature and the severity
of the battery inflicted by the batterer-spouse upon appellant. That is, repeated beatings over a
period of time resulted in her psychological paralysis, which was analogous to an illness
diminishing the exercise of her will power without depriving her of her consciousness of her
acts.
As to the extenuating circumstance of having acted
upon an impulse so powerful as to naturally produce passion or obfuscation, it has been held that
this state of mind is present when a crime is committed as a result of an uncontrollable burst of
passion provoked by prior unjust or improper acts or by a legitimate stimulus so powerful as to
overcome reason. To appreciate this circumstance, the following requisites should concur (1)
there is an act, both unlawful and sufficient to produce such a condition of mind; and (2) this act
is not far removed from the commission of the crime by a considerable length of time, during
which the accused might recover her normal equanimity.

188.

People v Dela Torre (419 SCRA 18)

Facts:
Husband and wife are the accused in this case for nine counts of crime of rape against Baby Jane
their house help. The initial rape occurred sometime in September 1992.
Accused Butchoy and Fe Dela Torre were sleeping on the same floor as Baby Jane but in
a separate bedroom. Fe de la Torre woke Baby Jane and her husband Butchoy. Baby Jane was
surprised to see that Fe was holding a lighted kerosene lamp and a scythe. Fe ordered her

husband to transfer and lie beside Baby Jane. As appellant Butchoy did not comply, Fe herself
transferred so that Baby Jane was between her and Butchoy. Fe put down the scythe and the lamp
and proceeded to take Butchoys clothes off and then Baby Janes. Butchoy offered no resistance
but Baby Jane objected and cried to no avail. The rape was repeated once a week from
September until October. However, she was again raped sometime in December and to this she
said this would be the last.
When she was left alone in the house she took the chance to escape and went to her godmother
barangay captain in Langongan. While Baby Jane was at the barangay captains home, Fe arrived
and told Baby Jane not to report the matter and threatened to twist the facts about the rape, that
she caught Baby Jane and her husband in the act and would file a complaint against them.
The spouses argue that it is unnatural for a wife like Fe to intensely desire that her husband have
sex with their maid.

Issue:
Whether accused Fe Dela Torre is guilty as principal?
Held:
Yes. Under the Revised Penal Code, an accused may be considered a principal by direct
participation, by inducement, or by indispensable cooperation. This is true in a charge of rape
against a woman, provided of course a man is charged together with her. Thus, the woman as a
principal by direct participation, since it was proven that she threatened and intimidated the
complainant in order to help her co-accused spouse consummate the offense.

189.

People v. Cui (G.R. No. 121982)

Facts:
In the evening of December 5, 1990, ten (10) armed robbers raided the compound of
Johnny and Rose Lim on Edison Street, Lahug, Cebu City. The Lims, their three (3) children, and
the employees of the family-owned business, were able to see the faces of the leader
Wilfredo alias "Toto" Garcia and two of his men, Mawe Garcia and a certain Edgar. The other
robbers could not be identified as they had flour sacks over their heads. The robbers carted away
cash and jewelries worth twenty thousand pesos (P20,000.00). They also blindfolded and
forcibly abducted seventeen (17) year old Stephanie, the youngest daughter of the Lims. They
demanded a ransom of one million pesos (P1,000,000.00) for her release.Johnny Lim turned over
to Toto Garcia the ransom amount in the afternoon of the next day at an arranged meeting place.
Stephanie, in turn, was released to her father.

Initially, the Lims kept the crime a secret. But on the third day, they reported the
kidnapping to the Philippine National Police Cebu Metropolitan District Command (Cebu
Metrodiscom). The Metrodiscom Intelligence Security Team (MIST) conducted an investigation
and Johnny Lim identified one of the suspects as Toto Garcia.
Toto Garcia was known as the leader of a group of armed robbers called the Baong Gang.
The gang's base of operation was pinpointed at Quiot, Pardo, Cebu. When the police learned that
Eduardo Basingan, hailed from Quiot, Pardo, Cebu City, they decided to interrogate him.
Upon Basingan's interrogation, he identified Toto Garcia, Mawe Garcia and Edgar as the
three (3) who did not wear masks, Sadam and Rey as the two (2) who held him and the Lims at
gunpoint, and Tata Garcia, Yul Alvarez, a certain Benjie, a certain Leos and a certain Laring as
the look-outs who stayed outside the Lim compound. He named Toto Garcia as the chief plotter
of the crime at bar, and revealed that his neighbor and close family friends, the spouses Leonilo
and Beverly Cui, participated in the plan. Basingan said he was asked to join the plot and was
assured that he would not be under suspicion because he would be placed at gun point together
with the other members of the Lim household when the crime is committed. However, he refused
to join the plot during the December 2, 1990 meeting of the group at the residence of the Cuis in
Quiot, Pardo, Cebu City. Leonilo Cui even invoked their close ties as godfathers of each other's
children but he was unmoved. At the meeting were Toto Garcia, Mawi Garcia, Edgar, Rey,
Sadam and the Cuis.
On December 18, 1990, Basingan executed a sworn statement reiterating these
revelations in writing. Johnny and Rose Lim then formalized their complaint by executing a Joint
Affidavit. Assistant Prosecutor Bienvenido N. Mabanto, Jr. filed an information for Kidnapping
with Ransom against Basingan, the Cuis, and the members of the group of Toto Garcia as
identified by Basingan in his sworn statement.On the same day, Basingan and Leonilo Cui were
arrested.
On March 14, 1991, Joselito "Tata" Garcia, Hilaria Sarte and her live-in partner, Luis
Obeso, referred to by Basingan as "Laring" and "Leos", respectively, were arrested in the
neighboring Negros Island. The next day, however, Tata Garcia died due to "hemorrhage, severe,
secondary to gunshot wounds." Upon presentation of his death certificate, the trial court ordered
his name deleted from the information.

After preliminary investigation, Prosecutor Manuel J. Adlawan found that the


participation of the Cuis was only that of accomplices amended the Information downgrading the
charge against the Cuis as mere accomplices in the kidnapping with ransom of Stephanie Lim.
On May 15, 1991, Basingan, the Cuis, Obeso and Sarte were arraigned and they all
pleaded not guilty. On June 27, 1991, Basinga escaped from the prison. Trial on the merit ensued
against the Cuis, Obeso and Sarte. Basingan was tried in absentia.
On February 13, 1992, Beinvenido Nacario, alias "Rey Nacario", was arrested. On
arraignment on April 13, 1992, he pleaded not guilty. However, on May 5, 1991, he, too, escaped
from detention and remains at large to this date.
On August 18, 1992, the prosecuting fiscal manifested before the trial court that, per newspaper
report, Toto Garcia had been killed in Davao. Thus, on December 6, 1993, the trial court
convicted the Cuis, Obeso, Sarte, Basingan and Nacario. Obeso and Sarte filed their Notice of
Appeal on May 19, 1994. The Cuis filed theirs on May 31, 1994.
Issue:
Whether or not the Cuis are liable as accessories?
Decision:
Art. 19 of the Revised Penal Code, as amended, penalizes as accessories to the crime
those who, subsequent to its commission, take part therein by profiting themselves or assisting
the offenders to profit by the effects of the crime, without having participated therein, either as
principals or accomplices. Conviction of an accused as an accessory requires the following
elements: (1) that he has knowledge of the commission of the crime; and (2) that he took part in
it subsequent to its commission by any of the three modes enumerated in Article 19 of the
Revised Penal Code, as amended. These twin elements are present in the case of the Cuis, and
indubitable proof thereof is extant in the records of the case. The Court held that the Cuis
profited from the kidnapping of Stephanie Lim and are liable as accessories.

You might also like